Вы находитесь на странице: 1из 201

2006

Oral and Maxillofacial Surgery


Self Assessment Tool (OMSSAT)

Administration Dates: April 17 28, 2006


The American Board of Oral and Maxillofacial Surgery
2005-2006 Oral and Maxillofacial Surgery Self
Assessment Tool (OMSSAT) Committee
Patrick J. Louis, DDS, MD, chair

Dentoalveolar Surgery

David A. Stanton, DMD, MD,


section editor

Nabil Abaza
Darren Cross
Richard DInnocenzo
Sam Farish
David Grogan
Deepak Kademan
Vasiliki Karlis
Lawrence Levin
Rawle Philbert
Anthony Pitrowski
Louis Rafetto
Kevin Rieck
Eric Rippert
Rory Sadoff
Vernon Sellers
Coleman Spector

Medical Assessment and Management of the Surgical Patient

Patrick J. Vezeau, DDS, MS,


section editor
Shawn Bailey Kurt Westlund
Jeffrey Bennett William Wood
Lionel Candelaria
Karen Crowley
Kirk Engel
Godfrey Funari
Andrew Hartwig
Robert Hinkle
Timothy Holton
John Kelly
John McCabe
Teresa Morgan
Noah Sandler
Daniel Sarasin
Paul Smith
William Synan
Patrick Vezeau
Larry Weeda

The American Board of Oral and Maxillofacial Surgery 2


Cosmetic Surgery

David Cottrell, DMD, section editor

Jeffrey Almony Julie Ann Smith


James Boyle Michael Will
Kamal Busaidy
Guillermo Chacon
Matthew Dennis
Stephaine Drew
David Emanuel
David Harmatz
Holly Hatt
Kathleen Herb
Pamela Hughes
Kurt Hummeldorf
Antilano Leon
PushkaMehra r
Daniel Mobati
Michael Peleg
Charles Repa

Trauma

Rich Haug, DDS, section editor

Jon Bradrick
Vito Cardo
Radhika Chigurupati
Karel deLeeuw
Robert Diecidue
Edward Ellis
Tirbod Fattahi
Rui Fernandes
Edwin Granite
Steven Kaltman
TakashiKoyama
Steward Lazow
Vincent Perciaccante
Gary Schween
Robert Strauss
Joseph Van Sickels
Brent Ward
Dennis-Duke Yamashita
Malcolm Zola

The American Board of Oral and Maxillofacial Surgery 3


1. Alignment of which of the following is the most reliable for proper reduction of the
zygomaticomaxillary complex fracture?

A. Frontozygomatic suture
B. Sphenozygomatic suture
C. Infraorbital rim
D. Medial orbital rim

Answer: B

Rationale:
The sphenozygomatic suture area has been previously analyzed and shown to be an area for
confirmation of alignment of the zygomatic arch and the zygomatic complex (ZMC). This has
also been shown to key point for fixation thru biomechanical studies.

The sphenozygomatic suture is a broad area along the greater wing of the sphenoid and can be
approached along the internal aspect of the lateral orbit. Even in severe midface fractures the
greater wing of the sphenoid is intact thus acting as a key landmark for proper reduction of the
ZMC fracture.

Reduction of the frontozygomatic suture or the infraorbital rim alone can result in errors due
to the small surface area. The medial orbit is generally not involved in a ZMC fracture.

Reference:
Rohner D, Tay A, Meny CS, Hutmacker DW, Hammer B.: The sphenozygomatic suture as a
key site for osteosynthesis of the orbitozygomatic complex in panfacial fractures: A
biomechanical study in human cadavers based on clinical practice. Plast Reconstr Surg 110:
1463, 2002.

Manson PN, Clark N, Robertson B, et al. Subunit principles in midface fractures: the
importance of sagittal buttresses, soft tissue reductions and sequencing treatments of
segmental fractures. Plast Reconstr Surg 103: 1287, 1999.

The American Board of Oral and Maxillofacial Surgery 4


2. When the medial canthal ligament is attached to a bony segment in naso-orbito-ethmoidal(NOE)
fracture repair the transcanthal wire is best placed:

A. after all soft tissue injuries have been addressed.


B. anterior to the original insertion of the canthal ligament.
C. posterior and inferior to the original insertion.
D. posterior and superior to the original insertion.

Answer: D

Rationale:
The purpose of the trans-canthal wire is to secure the canthal ligament and boney segment in
the pretraumatic position. Pull of the soft tissues displaces the bone and canthal ligament in an
anterior and inferior direction. Therefore a wire placed posterior and superior to the original
insertion provides a vector whose resistance to displacement is most ideal and provides the
best alignment.

Reference:
OMS Knowledge Update, Volume three, Section 6. Abubaker AO and Strauss RA, eds. p
TRA 75-76. Classification D - Trauma - Soft tissue Oral and Maxillofacial Surgery In-
Training Examination (OMSITE) questions for the Trauma Section.

The American Board of Oral and Maxillofacial Surgery 5


3. A 21-year-old female is an unrestrained driver involved in a MVA. She suffers a scalp laceration
and is noted to have lost 1000mL of blood at the scene. You would expect her vital signs to be
consistent with:

A. Pulse rate >100, normal systolic blood pressure, decreased pulse pressure, respiratory rate
of 20-30, urinary output of 20-30mL/hr.
B. Pulse rate <100, normal systolic blood pressure, normal or increased pulse pressure,
respiratory rate of 14-20, urinary output of >30mL/hr.
C. Pulse rate >120, decreased systolic blood pressure, decreased pulse pressure, respiratory
rate of 30-40, urinary output of 5-15mL/hr.
D. Pulse rate >140, decreased systolic blood pressure, decreased pulse pressure, respiratory
rate of >35, urinary output thats negligible.

Answer: A

Rationale:
These findings are consistent with a Class II hemorrhage, 750-1500ml, The vitals signs or
such a blood loss are consistent with those in response A. Response D reflects the vital signs
of a Type IV blood loss, Response C a Type III and Response B a Type I.

Reference:
1997 ATLS for Doctors, Sixth Edition.

The American Board of Oral and Maxillofacial Surgery 6


4. A 65-year-old man fell down the stairs. Upon examination of him, you notice that he opens his
eyes to speech, localizes pain, and mutters inappropriate words. You assess his Glasgow coma
scale (GCS) to be:

A. 13
B. 11
C. 9
D. 7

Answer: B

Rationale:
According to the Glascow Coma Scale, the patient can open his eyes in response to
commands speech, (3 out of 4); localizes pain, (5 out of 5); yet produces inappropriate words,
(3 out of 6); for a Glascow coma score of 11.

Reference:
1997 ATLS for Doctors, Sixth Edition

The American Board of Oral and Maxillofacial Surgery 7


5. A 79-year-old white male presents to your office for removal of carious teeth. Medical history
review reveals chronic obstructive pulmonary disease (COPD), hypertension, peptic ulcer
disease, athlerosclerosis with occasional angina, and osteoarthritis. Daily medications include
isosorbide dinitrate, furosemide, and acetaminophen. After conscious sedation with midazolam
and local anesthesia with prilocaine, you note that in recovery he has slowly become ashen
looking and the pulse oximetry reading has fallen to 85%. Which of the following measures is
most appropriate?

A. Intubation and hyperventilation with 100% oxygen


B. Titrated administration of 0.4 mg flumazenil IV
C. Methylene blue administration 1 mg/kg IV
D. Assisted ventilation by face mask with room air.

Answer: C

Rationale:
This situation may appear to be pulmonary in origin, but in fact represents acquired
methemoglobinemia. This condition can be precipitated by nitrates, (such as isosorbide
dinitrate) acetaminophen, prilocaine, articaine, and a number of other medications, especially
in genetically susceptible individuals. The oxidized (ferric) state of the methemoglobin
molecule cannot be reversed by increasing the FIO2, which also may decrease the respiratory
drive in COPD. Sedation reversal by flumazenil will have no effect on the condition.
Cautious administration of methylene blue will reduce methemoglobin back to a ferrous state,
normalizing the oxygen binding/delivering capacity of hemoglobin.

Reference:
Benumof JL Anesthesia & Uncommon Diseases, 4th ed. WB Saunders, 1998 pp288-9

The American Board of Oral and Maxillofacial Surgery 8


6. When performing a z-plasty to remove a prominent labial frenum the secondary incisions are
made at an angle approximately 60 degrees to allow the main limb to be rotated:

A. 33 degrees
B. 45 degrees
C. 60 degrees
D. 90 degrees

Answer: D

Rationale:
A z-plasty is designed to rotate the frenum or scar 90 degrees. Secondary incisions made at
other angles may not allow as great a rotation of the main limb (in this case, the main frenum
incision) as those made at 60 degrees tothe main limb.

Reference:
Fonseca, RJ, Oral and Maxillofacial Surgery Vol 7, Reconstruction and Implant Surgery, WB
Saunders 2000

The American Board of Oral and Maxillofacial Surgery 9


7. When performing a floor-of-the-mouth lowering procedure, it is necessary to:

A. perform a subperiosteal dissection


B. cover the denuded region with a soft tissue graft
C. avoid altering muscle attachments in patients diagnosed with retrolingual sleep apnea
D. detach all muscle attachments at the genial tubercle

Answer: C

Rationale:
Patients with suspected or diagnosed obstructive sleep apnea should not have muscle
attachments altered in floor-of -mouth lowering procedures because this may worsen or create
obstruction. Supraperiosteal dissections are performed and the incision margin is sutured to
the periosteum at the depth of the vestibule. It is not necessary to place a soft tissue graft over
the denuded periosteum as this may be allowed to secondarily epithelialize. The genioglossus
muscle attachments at the genial tubercle may be partially removed to increase the lingual
sulcus, but approximately of the genioglossus attatchment should remain intact to ensure
proper tongue function.

Reference:
Fonseca RJ Oral and Maxillofacial Surgery Vol. 7 p. 49WB Saunders2000

The American Board of Oral and Maxillofacial Surgery 10


8. A mandibular angle fracture with comminution, infection, or loss of bone buttressing is best
treated with which type of fixation:

A. miniplate.
B. dynamic compression plate (DCP).
C. reconstruction plate.
D. wire osteosynthesis.

Answer: C

Rationale:
The need for absolute stability for these types of fractures negates wire or miniplate fixation.
DCP causes interfragmentary compression with possible bone devitalization and necrosis. The
reconstruction plate is specifically indicated in these instances.

Reference:
Ellis- Treatment of mandible angle fractures using AO reconstruction plates.JOMS-
1993;51(3):250-254.

The American Board of Oral and Maxillofacial Surgery 11


9. Which of the following medications may trigger asthmatic symptoms?

A. Atropine
B. Ipatropium
C. Valdecoxib
D. Isoetharine

Answer: C

Rationale:
Valdecoxib (Bextra) is a cyclo-oxygenase-2 inhibitor. Any inhibitor of prostaglandin
synthesis (such as nonsteroidal anti-inflammatory drugs) can cause an increase in
leukotrienes which cause bronchoconstriction.

Atropine, being an anticholinergic, was formerly used to decrease bronchoconstriction in


asthma but is no longer used because of its systemic side effects. Ipatropium
bromide(Atrovent) is an inhaled anticholinergic used in chronic refractory asthma and in
chronic obstructive pulmonary disease. Isoetharine (Bronkosol) is an inhaled B2 agonist used
for bronchodilation as a nebulized solution.

Reference:
Washington Manual of Medical Therapeutics 28th ed., Little Brown, 1995 pp. 238-242

The American Board of Oral and Maxillofacial Surgery 12


10. Which of the following would be observed in a patient with an isolated C4-5 spinal cord injury?

A. Disturbance of heart rate


B. Apnea
C. A major loss of diaphragmatic function
D. An air embolism

Answer: C

Rationale:
The phrenic nerve innervates the diaphragm and arises from cervical segments 3, 4, and 5.
Therefore, a cervical spine injury at this level would likely cause severe ventilatory
dysfunction, affecting tidal volume, vital capacity, or FEV1.
A is incorrect, since parasympathetic innervation to the heart arises from Cranial Nerve X and
the sympathetic innervation arises from the cervical sympathetic trunk, and both join to form
the cardiac plexus.
B is incorrect, since the injury only involves C4-5 (and not C3); the phrenic nerve would
probably still be somewhat functional, and some ventilation would be expected.
D is incorrect. This is a completely random answer, not necessarily related to this injury.

Reference:
Hollingshead's Manual of Practical Anatomy

The American Board of Oral and Maxillofacial Surgery 13


11. A 16-year-old male is involved in a motor vehicle accident. He is found unresponsive at the
scene and is intubated and brought to the emergency room. On arrival he both opens his eyes and
withdraws only to painful stimulus. What is his Glascow Coma Scale classification?

A. 4T
B. 5T
C. 6T
D. 7T

Answer: C

Rationale:
Intubated patients receive a 'T' since they are unable to verbally respond

Best Eye Response. (4)


No eye opening.
Eye opening to pain.
Eye opening to verbal command.
Eyes open spontaneously.
Best Verbal Response. (5)
1. No verbal response
2. Incomprehensible sounds.
3. Inappropriate words.
4. Confused
5. Orientated
Best Motor Response. (6)
1. No motor response.
2. Extension to pain.
3. Flexion to pain.
4. Withdrawal from pain.
5. Localising pain.
6. Obeys Commands.

Reference:
ATLS Manual, 1997

The American Board of Oral and Maxillofacial Surgery 14


12. When utilizing the Champy technique for fixation of a mandibular angle fracture, the plate used
for fixation is:

A. an eccentric dynamic compression plate.


B. placed in the zone of compression.
C. placed in the zone of neutrality.
D. placed in the zone of tension.

Answer: D

Rationale:
The principle of the Champy technique is for fixation in the zone of tension with natural
functional forces aiding in the approximation of the fracture in the area of compression.

Reference:
Champy M, Lodde JP, Scmitt R, et al.
Mandibular osteosynthesis by miniature screwed plates via a buccal approach.
Journal of Maxillofacial Surgery 1978; 6:14-9.

The American Board of Oral and Maxillofacial Surgery 15


13. When utilizing a retromandibular approach to a subcondylar fracture, which of the following is
true?

A. The dissection can go behind or through the parotid gland


B. The dissection goes between the temporal and zygomatic branches of the facial nerve
C. The approach is best for subcondylar fractures that are high
D. The superior extent of the incision begins 2 cm below the earlobe

Answer: A

Rationale:
Dissections posterior to the parotid and through the parotid have been described. Classically
the dissection carried between the cervicofacial trunk and the temporofacial trunk of the facial
nerve. The dissection may be on either side of the marginal mandibular branch. This
approach is best for lower subcondylar fractures and more limited for higher fractures. The
incision begins approximately 0.5 cm below the earlobe.

Reference:
Hinds EC: Correction of prognathism by subcondylar osteotomy. J Oral Maxillofac Surg
16:209, 1958
Ellis E III, Zide MF: Surgical Approaches to the Facial Skeleton. Baltimore. Lippincott
Williams & Wilkins. 1995

The American Board of Oral and Maxillofacial Surgery 16


14. When performing a Risdon approach to a mandibular angle fracture:

A. posterior to the facial artery the marginal mandibular branch of the facial nerve is always
located below the inferior border of the mandible.
B. posterior to the facial artery the marginal mandibular branch of the facial nerve may drop
2.5cm below the inferior border of the mandible.
C. anterior to the facial artery the marginal mandibular branch of the facial nerve is found
below the inferior border of the mandible less than 10% of the time.
D. anterior to the facial artery the marginal mandibular branch of the facial nerve usually
has only one branch.

Answer: C

Rationale:
The two classic articles on this subject quote incidences of 0% and 6% frequency of the
marginal mandibular branch of the facial nerve being found below the inferior border of the
mandible anterior to the facial artery. Posterior to the facial artery, 19% of the time the
marginal mandibular branch of the facial nerve may be located below the inferior border of
the mandible up to 1 (or1.2) cm. Anterior to the facial artery the marginal mandibular branch
of the facial nerve has one branch only 21% of the time.

Reference:
Ziarah HA. Atkinson ME. The surgical anatomy of the mandibular distribution of the facial
nerve. [Journal Article] British Journal of Oral Surgery. 19(3):159-70, 1981 Sep
Dingman R, Grabb W: Surgical anatomy of the mandibular ramus of the facial nerve based on
the dissection of 100 facial halves. Plast Reconstr Surg 29:266-272, 1962

The American Board of Oral and Maxillofacial Surgery 17


15. When treating uncomplicated, compound mandible fractures, the current recommendation for
antibiotics is that:

A. antibiotic coverage is not needed.


B. perioperative antibiotic use is indicated.
C. pre-op antibiotics with a 7 day post-operative course are indicated.
D. antibiotics are only useful for open reductions.

Answer: B

Rationale:
Compound or open mandible fractures have been shown to have reduced infection rates when
perioperative antibiotics have been used. The use of postoperative courses of antibiotics has
not been shown to affect the rate of infection.

Reference:
Zallen RD, Curry JT:A study of antibiotic usage in compound mandibular fractures. J Oral
Surg; 33:431, 1975.
Abubaker AO, Rollert MK:Postoperative antibiotic prophylaxis in mandibular fractures:A
preliminary randomized double-blinded and placebo controlled clinical study. J Oral
Maxillofac Surg; 59:1415, 2001.

The American Board of Oral and Maxillofacial Surgery 18


16. Surgical exposure of which of the following areas would require soft tissue re-suspension to re-
establish facial form?

A. Midface exposure via an oral vestibular incision


B. Symphysis exposure via submental incision
C. Zygomatic exposure via coronal incision
D. Orbital floor exposure via transconjunctival incision.

Answer: C

Rationale:
Extended exposure of the facial skeleton is indicated for management of many complex facial
fractures. Re-suspension of the soft tissue is important to establish proper facial form.
Midface exposure via vestibular incision avoids key suspensory areas, thus simple closure is
usually indicated. Symphysis exposure via a submental incision leaves the origin of the
mentalis muscle attached to the surround soft tissue and/or bone. Re-suspension via this
approach is not indicated. With exposure of the orbital floor via a transconjunctival incision
simple closure is all that is indicated. When the zygomatic process is exposed via a coronal
approach the superficial layer of the temporalis fascia is incised and periosteum overlying the
arch is detached. This can result in ptosis of the suborbicularis oculi fat pad. This fat pad
must be re-suspended to the lateral orbit.

Reference:
Manson PN, Clark N, Roberston B, et al Subunet Principles in Midface Fractures:The
Importance of Sagittal Buttresses, Soft Tissue Reductions and Sequencing Treatment of
Segmental Fractures. Plast Reconstr Surg 103:1287-1306, 1999.

The American Board of Oral and Maxillofacial Surgery 19


17. Which of the following is the treatment of choice in management of bilateral mandibular body
fractures in a patient with an extremely atrophic mandible (less than 6 mm of bone height)?

A. Closed reduction with a gunning splint


B. Lag screw fixation of the mandibular segments
C. Fixation with mini plates
D. Fixation with reconstruction plates

Answer: D

Rationale:
In order to allow healing of a fracture of the atrophic mandible, the fixation technique must
neutralize the tension forces on the mandible. The only fixation technique that satisfies these
criteria is the use of a reconstruction plate.

Reference:
Sikes JW, Smith BR, Mukherjee DP. An In-Vitro Study of the Effect of Bony Buttressing on
Fixation Strength of a Fractured Atrophic Edentulous Mandible Model J Oral Maxillofacial
Surgery 58:56-61, 2000.

The American Board of Oral and Maxillofacial Surgery 20


18. A patient presents with a displaced inner and outer table frontal sinus fracture with associated
CSF leak. There is no involvement of the nasofrontal drainage system. Which of the following
procedures is indicated for the management of the frontal sinus?

A. Open reduction and internal fixation of the inner and outer table of the frontal sinus
B. Obliteration of the frontal sinus with preservation of the nasofrontal drainage system
C. Cranialization of the frontal sinus with obliteration of the nasofrontal draining system
D. Obliteration of the frontal sinus with obliteration of the nasofrontal drainage system

Answer: C

Rationale:
Fracture of the inner and outer table of the frontal sinus with communition and CSF leak
represent the most complex of frontal sinus injuries. The dual tears associated with the frontal
sinus leak must be repaired and they are usually approached via a coronal incision with the
removal of a portion of the frontal bone. With communition of the posterior table ORIF of
these fractures would be difficult. Because of the rapid growth of respiratory mucosa any and
all remnants of epithelium within the sinus must be removed and the bony walls curetted to
remove any epithelium remnants. Obliteration of the sinus is indicated when there is an intact
posterior sinus wall. This allows for maintenance of the material used for obliteration to
remain within the sinus cavity. In all cases of obliteration of the frontal sinus, obliteration of
the nasofrontal ducts is also indicated to prevent respiratory microflora from entering into this
cavity or entering into the sinus, thus preventing infection. In this case, due to the fracture of
the inner and outer table of the frontal sinus with associated CSF leak, all of the posterior
table should be removed to allow for expansion of the frontal lobes into the residual cavity.
The outer table is reconstructed in this procedure. In all cases of cranialization of the frontal
sinus, the nasofrontal drainage system is obliterated.

Reference:
Rohrich RJ, Hollier LH. Management of Frontal Sinus
Fractures; Changing Concepts. Clinics in Plastic Surgery 19:219-232:1992.

The American Board of Oral and Maxillofacial Surgery 21


19. Penetrating trauma located just above the clavicles would be an injury to what zone of the neck?

A. Zone 1
B. Zone 2
C. Zone 3
D. Zone 4

Answer: A

Rationale:
The zones of the neck were originally described by Dr. Monson in
1969. Zone 1 is defined as the area from the clavicles to the cricoid cartilage. The risk of
injury to the great vessels is common in this area. Zone 2 represents the area from the cricoid
cartilage to the angle of the mandible. It is the largest area of the neck and thus is most likely
to be injured with penetrating neck trauma. Zone 3represents the region from the angle of the
mandible to the base of the skull. The area presents the most difficult area for surgical access.

Reference:
Reference:Monson DO, Saletta JD, Freeark RJ. Carotid Vertebral Trauma. J Trauma 9:987-
99, 1969.

The American Board of Oral and Maxillofacial Surgery 22


20. At what distance behind the superior aspect of the medial orbital rim would the anterior
ethmoidal foramen and its associated artery be located?

A. 0 to 5 mm
B. 10 to 15 mm
C. 20 to 25 mm
D. 30 to 35 mm

Answer: C

Rationale:
The anterior and posterior ethmoidal arteries will require identification when performing
medial orbital wall or roof dissections. The vessels may also be identified for ligation in order
to control nasal bleeding. The anterior ethmoidal foramen transmits the anterior ethmoidal
artery and anterior ethmoidal branches from the nasociliary nerve. The posterior ethmoidal
foramen transmits the posterior ethmoidal artery and the spheno-ethmoidal nerve from the
nasociliary nerve. These foramina can be located posterior to the junction of the medial
orbital wall and orbital roof. The anterior ethmoidal foramen is located twenty to twenty-five
millimeters behind the medial orbital rim and the posterior ethmoidal foramen is located
twelve millimeters posterior to this point or approximately thirty-two to thirty-seven
millimeters.

Reference:
Ochs MW. Orbital and Ocular Trauma. Miloro M (Ed):Peterson's Principals of Oral and
Maxillofacial Surgery, Second Edition. BC Decker, Inc. Hamilton, Ontario 2004. p. 465

The American Board of Oral and Maxillofacial Surgery 23


22. A 9-year-old child is brought to the emergency room after being struck by a car. The patient is
unresponsive to command and breathing only infrequently. Cervical immobilization is in place.
Oral intubation attempts are unsuccessful because of brisk bleeding from facial fractures. The
most appropriate next step is:

A. obtain an angiogram
B. surgical cricothyroidotomy.
C. surgical tracheostomy.
D. percutaneous tracheostomy.

Answer: B & C

Rationale:
The child is too young for cricothyroidotomy and percutaneous tracheostomy is not indicated
for emergency airway. Cricothyroidotomy is not recommended in children less than 12 years
of age, since the cricoid cartilage is the only circumferential support to the upper trachea.
ATLS teaches that a child may be temporarily oxygenated with needle jet insufflation, but this
technique does not provide ventilation and is not a definitive airway. Although surgical
tracheostomy is unappealing in this age category, it is the procedure of last resort in this
scenario.

Reference:
Advanced Trauma Life Support for Doctors Course Student Manual, 7th Ed., First Impression
Press, 2004, page 48,49,250

The American Board of Oral and Maxillofacial Surgery 24


23. An adult patient has a self-inflicted shot gun wound to the mouth. He is brought to the
emergency room intubated with a large facial wound. Vital signs upon arrival are heart rate 130,
respiration 35, and blood pressure 90/60. The patient is fighting restraints. A urinary catheter
insertion produced only 10 cc of urine. Which of the following Class of hemorrhage and initial
resuscitation treatment is correct?

A. Class II. Treat with crystalloid solution


B. Class III. Treat with blood transfusion
C. Class III. Treat with crystalloid solution and blood transfusion
D. Class IV. Treat with crystalloid and blood transfusion.

Answer: C

Rationale:
Class III hemorrhagic shock is characterized by tachycardia, tachypnea, mental status changes
and measurable fall in systolic blood pressure, but without the significant narrowing of pulse
pressure seen in Class IV shock. Also the tachycardia and tachypnea are worse in Class IV
shock. Lactated Ringers or 0.9% normal saline is the fluid of choice for initial resuscitation.
Given excessive hemorrhage, >20%, red blood cells should be replaced to maintain oxygen
carrying capacity. Cross-matched is the ideal choice, but time constraints (30 min) may
require Type-specific blood (5-15 min) to be used.

Reference:
Advanced Trauma Life Support for Doctors Course Student Manual, 7th Ed., First Impression
Press, 2004, page 75
Miloro M, et al. Peterson's Principles of Oral and Maxillofacial Surgery, 2nd Edition. Pg
343. BC Decker Inc, 204.

The American Board of Oral and Maxillofacial Surgery 25


25. When opening a subcondylar fracture through a retromandibular approach:

A. the patient should be paralyzed .


B. the forehead should always be in view in the surgical field.
C. the closure of the parotid capsule/SMAS and platysma layer is critical to minimize the
occurrence of a salivary fistula.
D. the local anesthesia should be injected deep to the platysma muscle.

Answer: C

Rationale:
Salivary-cutaneous fistulae are potential sequelae to the retromandibular approach. To help
avoid this complication it is recommended that a water tight closure be performed.

Reference:
Ellis E. Zide M. Surgical Approaches to the Facial Skeleton Williams and Wilkins, Media,
PA, 1995

The American Board of Oral and Maxillofacial Surgery 26


27. Intravenous fluids administered to a patient with acute brain injury should be:

A. hypotonic.
B. hypertonic.
C. normotonic.
D. glucose-containing.

Answer: C

Rationale:
Intravenous fluids administered to acutely brain-injured patients should normotonic in order
to maintain normovolemia. Hypovolemia and hypervolemia is harmful to these patients.
Hyperglycemia from use of glucose-containing fluids is harmful to the injured brain. Normal
saline or Ringer's lactate is recommended for resuscitation. Serum sodium levels need to be
monitored to prevent hyponatremia which can lead to brain edema.

Reference:
Advanced Trauma Life Support for Doctors. Student Course Manual, 7th Edition, p.165.
American College of Surgeons 2004.

The American Board of Oral and Maxillofacial Surgery 27


28. Regarding blunt force trauma to the midface, which of the following anatomic structures can be
considered a pillar of support that can resist compression and deformation?

A. Anterior maxillary sinus wall


B. Posterior maxillary sinus wall
C. Zygomatico-maxillary junction
D. Zygomatico-sphenoid junction

Answer: C

Rationale:
There are three vertical buttresses of the midface:zygomatico-maxillary, pterygo-maxillary,
and naso-maxillary. These structures consist of thicker supporting regions of bone that
transmit and/or resist forces to the base of the skull. The horizontal buttresses of the midface
are the (1) pyriform aperture, (2) maxillary alveolus and palate, (3) orbital rims and (4) base
of the skull.

Reference:
Fonseca, RJ, Oral and Maxillofacial Trauma Vol 1, Advances in Maxillofacial Trauma
Surgery, Elsevier 2005.
Manson P, et al. Plastic and Reconstructive Surgery. Structural Pillars of the Facial
Skeleton:An Approach to the Management of Le Fort Fractures. July 1980, 66(1) pg. 57.

The American Board of Oral and Maxillofacial Surgery 28


29. Stabilization of the medial canthal tendon during a medial canthopexy should be directed at
which of the following directions?

A. Anterior and superior


B. Anterior and inferior
C. Posterior and superior
D. Posterior and inferior

Answer: C

Rationale:
Repair of detached medial canthal tendons during a severe naso-orbital ethmoidal fracture
repair involves reattachment of one or both tendons to the posterior lacrimal crest in a
slightly posterior and superior direction to overcome the forces of migration, relapse, and
telecanthus.

Reference:
Fonseca, RJ, Oral and Maxillofacial Trauma Vol 2, Diagnosis and Treatment of Midface
Fractures, Elsevier 2005

The American Board of Oral and Maxillofacial Surgery 29


31. An injury to the temporo-facial trunk of the facial nerve is suspected within a deep cheek
laceration. Which of the following will best confirm an intact and functioning facial nerve?

A. A normal Bells phenomenon


B. An intact corneal reflex
C. Absence of divergent strabismus
D. Absence of an oculo-cardiac reflex

Answer: B

Rationale:
Integrity of the temporo-facial trunk of the facial nerve can be assessed by performing a
corneal reflex (tearing to touching the ipsilateral cornea.) The afferent and efferent limbs of
the reflex involve the first division of the trigeminal nerve and facial nerve respectively.
Strabismus is associated with defects in cranial nerves III, IV, and VI.

Reference:
Bates, B, A Guide to Physical Examination and History Taking, The Nervous System, JB
Lippincott 2002

The American Board of Oral and Maxillofacial Surgery 30


32. Which of the following would be an indication for frontal sinus obliteration?

A. Non-displaced greenstick fracture of the anterior table.


B. Fracture of the posterior table.
C. Nasofrontal duct occlusion.
D. Nasolacrimal duct damage.

Answer: C

Rationale:
Displaced anterior table fractures should be opened with careful inspection of the sinus
mucosa. A non-displaced posterior table fracture without CSF leak can be managed without
obliteration of the frontal sinus. Occlusion of the nasofrontal ducts requires removal of all
sinus mucosa from the frontal sinus, inversion of the mucosa into the ducts, and elimination of
dead space. This eliminates the possibility of developing a hematoma or seroma. Fat has
been historically used with most success; no filler, hydroxyapatite, glass wool, bone, cartilage,
muscle, absorbable gelatin sponge and fabric, acrylic have also been used.

Reference:
Miloro M, et al. Peterson's Principles of Oral and Maxillofacial Surgery, 2nd Edition. Pg
496-501. BC Decker Inc, 204.

The American Board of Oral and Maxillofacial Surgery 31


33. Which of the following surgical approaches to the inferior orbital rim has the least potential for
producing post-operative ectropion and poor cosmesis?
A. Infraorbital
B. Subciliary
C. Transconjunctival
D. Inferior Lid

Answer: C

Rationale:
Infraorbital or rim incision results in the worst esthetics and offers no advantages. The
subciliary incision has an ectropion rate of approximately 6 %, whereas the transconjunctival
incision has an ectropion/entropion rate of approximately 1.2 %. Thus, the transconjunctival
incision as the approach of choice to avoid ectropion.

Reference:
Miloro M, et al. Peterson's Principles of Oral and Maxillofacial Surgery, 2nd Edition. BC
Decker Inc, 2004.

The American Board of Oral and Maxillofacial Surgery 32


34. Which of the following is the most common early complication related to frontal sinus trauma?
A. Mucocele formation
B. Brain abscess
C. Mucopyocele formation
D. Meningitis

Answer: D

Rationale:
All of the above cited answers are potential complications to frontal sinus fractures.
Meningitis is the most common EARLY complication. The remainders of the answers are
delayed or late potential complications. Frontal sinus complications can occur many years
after injury (often greater than 20 years after injury). The most serious complications: can
include displacement of the frontal bone in the brain and floor of frontal sinus into the orbit.
Infectious complications, such as meningitis or intracranial abscess occur via bacterial spread
through posterior table fractures or the diploe veins of Behet; or by osteomyelitis, or
mucocoele formation. Mucoceles are the most common chronic complication with diagnosis
made via CT scanning. Periodic review by CT should be done every 1, 2 and 5 years
following surgery. Symptoms include headache and can include loss of smell (anosmia.).

Reference:
Fonseca, RJ, Oral and Maxillofacial Trauma, Third Edition, 2005, Vol 2, pp 731-32. Miloro
M, et al. Peterson's Principles of Oral and Maxillofacial Surgery, 2nd Edition. Pg 503. BC
Decker Inc, 204.

The American Board of Oral and Maxillofacial Surgery 33


36. A comminuted angle fracture of the mandible would be treated most effectively with the
following fixation:

A. ORIF with reconstruction bone plate applied to the inferior border with 3 bone screws on
each side of the fracture.
B. antibiotics and soft diet for 4 weeks.
C. anatomic reduction with Champy bone plate at the superior border.
D. maxillo-mandibular fixation to restore occlusion for 4 weeks.

Answer: A

Rationale:
The reconstruction plate is designed to bear the entire load of the mandibular movement
across the fracture site. The remaining methods mentioned do not adequately stabilize the
fracture segments and increase the possibilities for non-union and infection.

Reference:
Miloro, M. et al, Peterson's Principles of Oral and Maxillofacial Surgery, 2nd Edition, page
375, BC Decker INC, 2004

The American Board of Oral and Maxillofacial Surgery 34


37. The best technique to manage a 40% traumatic avulsion of the upper eyelid is:

A. allow the wound to heal by secondary granulation.


B. undermine and close primarily.
C. split thickness skin graft from the ipsilateral postauricular region.
D. full thickness skin graft harvested via blepharoplasty approach from the contralateral
upper eyelid.

Answer: D

Rationale:
Full thickness skin graft from the opposite upper lid is preferred due to similar skin thickness,
color, texture and availability with minimal donor site morbidity. This avoids scar and
ectropion.

Reference:
Sandler NA, Evaluation and management of traumatic eyelid injury, OMS Knowledge Update
vol 3, AAOMS, 2001

The American Board of Oral and Maxillofacial Surgery 35


38. The most predictable means to treat a large posterior auricular skin/cartilage avulsion is:

A. reattachment as a composite graft with hyperbaric oxygen.


B. microvascular reimplantation.
C. full thickness skin graft to cover exposed cartilage.
D. 2 phase bipedicled postauricular flap.

Answer: D

Rationale:
Exposed cartilage will not accept a skin graft. Composite free auricular grafts and
microvascular reanastomosis have poor success rates due to poor venous drainage. The
bipedicled flap nicely and predictably reconstructs the posterior helical defect. If the avulsed
segment is 1 cm or less it can be reattached and allowed to revascularize. Larger avulsive
injuries, the pocket principle can be used where the detached ear is dermabraded and
reattached to the stump and then buried under a skin flap in the posterior auricular region to
provide vascular supply. 2-3 weeks later the revascularize ear is uncovered and allowed to
reepithelialize.

Reference:
Dierks EJ, Deeb GR, Evaluation and management of ear injuries, OMS Knowledge Update
vol 3, AAOMS, 2001

The American Board of Oral and Maxillofacial Surgery 36


39. Recurrent sialocoele formation after emergency room repair of a deep facial laceration is best
handled by:

A. multiple aspirations, pressure dressings, and antisialagogues.


B. superficial parotidectomy.
C. reexploration of the wound, cannulation of Stensens duct, and repair of the parotid
capsule.
D. low dose radiation.

Answer: A & B

Rationale:
Sialoceoles that repeatedly reform, even after aspiration, and pressure dressings, and
antisialogogues may indicate a parotid capsular or Stensen's duct injury that requires early
reexploration and repair. Sialocoele formation within 48 hours can indicate an injury to the
salivary ductal system and exploration of the injury is needed. The patient also needs to have
antibiotic coverage with penicillin or a cephalosporin.

Reference:
Fonseca RJ and Walker RV, Management of soft tissue injuries, Oral and Maxillofacial
Trauma, vol 1, pg. 642. WB Saunders, 1991

The American Board of Oral and Maxillofacial Surgery 37


40. A 5cm. full thickness scalp avulsion down to the cranium is best treated by a:

A. split thickness skin graft.


B. full thickness skin graft.
C. split thickness skin graft placed over bur holes made in the outer table.
D. rotation-advancement flap.

Answer: D

Rationale:
Exposed cortical cranial bone will not accept skin grafts. Rotational-advancement flaps with
multiple galeal releasing incisions parallel to the long axis of the flap will provide adequate
coverage in most instances.

Reference:
Fonseca RJ and Walker RV, Management of soft tissue injuries, Oral and Maxillofacial
Trauma, vol 1, WB Saunders, 2001

The American Board of Oral and Maxillofacial Surgery 38


41. Success after replantation of an avulsed tooth that was preserved in a clean and moist container
by the patient is most dependent on:

A. sterilization before replantation.


B. replantation within 30 minutes after avulsion.
C. curettage of the tooth root and socket.
D. rigidity of fixation splinting.

Answer: B

Rationale:
Optimal success of treatment is to replant and stabilize avulsed teeth within 2 hours. The
PDL cells become irreversibly necrotic after this time frame. Treatment with a nonrigid splint
will allow movement of the tooth, allowing the PDL to heal.

Reference:
Abubaker AO, Giglio JA, Mourino AP, Diagnosis and management of dentoalveolar injuries.
In:Fonseca RJ:Oral and Maxillofacial Surgery, Vol 1. W.B. Saunders Co, Philadelphia, 2000.
Miloro M, et al. Peterson's Principles of Oral and Maxillofacial Surgery, 2nd Edition. Pg
394-5. BC Decker Inc, 2004.

The American Board of Oral and Maxillofacial Surgery 39


42. Failure after replantation of an avulsed tooth occurs mostly due to:

A. internal resorption, external resorption, pulpal necrosis.


B. external resorption, ankylosis, periodontal disease.
C. internal resorption, tooth migration, ankylosis.
D. external resorption, pulpal necrosis, periodontal disease.

Answer: A

Rationale:
The most common tooth avulsed is the maxillary central incisor. Treatment is geared towards
early re-establishment of the PDL cell physiology, via solutions with physiologic pH and
osmolarity. Re-implant and stabilize the tooth within two hours of avulsion.

Reference:
Abubaker AO, Giglio JA, Mourino AP, Diagnosis and management of dentoalveolar injuries.
In:Fonseca RJ:Oral and Maxillofacial Surgery, Vol 1. W.B. Saunders Co, Philadelphia, 2000.
Miloro M, et al. Peterson's Principles of Oral and Maxillofacial Surgery, 2nd Edition. Pg
393-395. BC Decker Inc, 2004.

The American Board of Oral and Maxillofacial Surgery 40


43. Generally speaking, soft tissue injuries associated with dentoalveolar trauma are treated:

A. before management of all hard tissue injuries.


B. after management of all hard tissues injuries.
C. before management of bony injuries, but after management of dental injuries.
D. should never be closed primarily but should be allowed to heal by secondary intention.

Answer: B

Rationale:
Soft tissue wounds associated with dentoalveolar trauma are always treated after management
of the hard tissue. This prevents wasting time of suturing wounds that are likely to be
removed or compromised during the intraoral manipulation to treat the hard tissue or dental
injuries.

Reference:
Peterson LJ, Contemporary Oral and Maxillofacial Surgery, 2nd edition, Ed. Pg. 583. JB
Lippincott Co., Philadelphia 1992

The American Board of Oral and Maxillofacial Surgery 41


44. In order to avoid irreversible vision loss, an orbital fracture associated with optic nerve
compression must be addressed within which length of time?

A. 24 hours
B. 36 hours
C. 4 hours
D. 2 hours

Answer: D

Rationale:
The retina is an extension of the brain, and will have similar reaction in the presence of
ischemia. Signs and symptoms of visual impairment, severe ocular pain should be evaluated
for retrobulbar hematoma. Computed tomography evaluation can demonstrate signs of
retrobulbar hematoma formation, but immediate treatment with a lateral canthotomy based on
clinical symptoms and signs should be completed prior to a lengthy CT scan. Other signs of
optic nerve compromise include (1) afferent papillary defect (2) visual field loss and (3) color
vision defects.

Reference:
Girotto JA, Gamble WB, Robertson B, et al, Blindness After Reduction of Facial Fractures,
Plast Reconstr Surg 102:1821, 1998.
Miloro M, et al. Peterson's Principles of Oral and Maxillofacial Surgery, 2nd Edition. Pg
472. BC Decker Inc, 204.

The American Board of Oral and Maxillofacial Surgery 42


45. After surgical treatment of a zygomaticomaxillary complex fracture, the immediate treatment of
a retrobulbar hematoma accompanied by loss of visual acuity and increased intraocular pressure
is:

A. decompression of the frontal sinus.


B. intravenous corticosteroids.
C. lateral canthotomy.
D. medial canthotomy.

Answer: C

RATIONALE: The emergency procedure of choice for acute visual acuity loss associated with
acute orbital compartment syndrome is dissection of the lateral canthus and disinsertion of at
least the inferior crus of the lateral canthal tendon, which allows complete mobility of the lower
lid. Visual loss without clear signs consistent with increased IOP is not an indication for this
procedure. Other primary indications for lateral canthotomy and cantholysis include an IOP
greater than 40 mm Hg and proptosis, which may be used as a criterion for unconscious patients
whose visual acuity cannot be determined. Secondary criteria include afferent pupillary defect,
ophthalmoplegia, cherry-red macula, optic nerve head pallor, and severe pain, but these are all
considered less sensitive or very late signs. Contraindication for this procedure would be a
suspected ruptured globe.

REFERENCE: Fonseca, RJ, Walker, R, Oral and Maxillofacial Trauma Volume 1, WB


Saunders, 1991

The American Board of Oral and Maxillofacial Surgery 43


46. To restore proper projection of facial width and orbital volume in a patient with a high energy
grossly comminuted zygomaticomaxillary fracture one should:

A. use a Carroll- Girard screw placed transcutaneous to reduce the fracture in a closed
method.
B. align the comminuted segments of the infraorbital rim to assure proper placement of the
zygoma.
C. expose the sphenozygomatic suture to aid in anatomic reduction.
D. expose and plate the zygomatic-frontal suture area before exposing other areas of the
zygoma.

Answer: C

Rationale:
A grossly comminuted zygomatic complex fracture will not be stable with a closed technique.
Comminuted segments of the infraorbital rim are a poor guide to placement of the zygoma.
The zygomatic-frontal suture is a poor reference for placement of the zygoma. In high energy
grossly comminute fractures, the sphenozygomatic suture with its broad contact can aid in the
anatomic reduction of the fracture.

Reference:
Baley JS, Goldwasser MS, Management of Zygomatic complex fractures. In: Miloro M ed,
Peterson's Principles of Oral and Maxillofacial Surgery Second edition BC Deker, London
2004 Chapter 23.2 pp 452

The American Board of Oral and Maxillofacial Surgery 44


47. Irreversible ischemic damage to the retinal tissue secondary to pressure and hypoxia has been
shown:

A. within 60 minutes to 2 hours following a traumatic event.


B. avoidable by use of megadose steroids which should be initiated 48 hours after injury.
C. to respond better to treatment when the patient has no initial light perception.
D. to respond better when there is penetrating trauma versus blunt trauma.

Answer: A

Rationale:
The optic nerve and retinal tissue, unlike skeletal muscle, have been shown to be extremely
sensitive to pressure and hypoxia. Irreversible ischemic damage often occurs within 60
minutes and certainly within 2 hours. Megadose steroids have been recommended during the
first 8 hours following injury. The outcome in patients with traumatic optic neuropathy with
no light perception on presentation is a poor prognostic factor. Likewise patient presenting
with traumatic optic neuropathy after penetrating trauma have a worse prognosis than those
with blunt facial trauma.

Reference:
Wang BH, Robertson BC, Girotto JA, Liem A, Miller NR, Iliff N, Manson P, Traumatic optic
neuropathy: A review of 61 patients. Plastic Reconstructive Surg 2001; 107:1653-1664

The American Board of Oral and Maxillofacial Surgery 45


48. Rebleeding following a traumatic hyphema:

A. is associated with the use of topical cycloplegics.


B. can be prevented by increasing intraocular pressure.
C. may occur in 5 to 30% of patients.
D. is increased with the use of carbonic anhydrase inhibitors.

Answer: C

Rationale:
The management of a patient with a traumatic hyphema is to use topical cycloplegics and
carbonic anhydrase inhibitors to decrease intraocular pressure. Rebleeding can occur in 5-
30% of the patients with hyphema.

Reference:
Baley JS, Goldwasser MS, Management of Zygomatic complex fractures. In:Miloro M ed,
Peterson's Principles of Oral and Maxillofacial Surgery Second edition BC Deker, London
2004 Chapter 23.2 pp 459

The American Board of Oral and Maxillofacial Surgery 46


49. An afferent pupillary defect in the presence of a normal eye:

A. is diagnosed when the pupil reacts to direct stimulation but shows a noticeably more
brisk consensual response when the contralateral pupil is directly tested.
B. indicates a normal optic nerve in the involved eye.
C. has a different response than a Marcus Gunn pupil.
D. is associated with pupil asymmetry.

Answer: A

Rationale:
The diagnosis of an afferent pupillary defect in the presence of normal eye is when the pupil
reacts to direct stimulation but shows a noticeably more brisk consensual response when the
contralateral pupil is directly tested. It indicates an optic nerve injury. An afferent pupillary
defect is also call a Marcus Gunn pupil. Pupillary symmetry is not affected by the afferent
pupil defect. This implies an efferent lesion or an iris lesion or both.

Reference:
Gossman MD, Roberts DM, Barr CC, Ophthalmic aspects of orbital injury; A comprehensive
diagnostic and management approach. Clin Plastic Surg 1992; 19:17-85

The American Board of Oral and Maxillofacial Surgery 47


50. A CT scan reveals an orbital fracture with a trap door appearance with clinical and radiographic
eidence of inferior rectus muscle incarceration is:

A. more likely to occur in adults than in children.


B. warrants early intervention to free up the tissues.
C. can be managed by steroids and delay of surgery for 3 weeks
D. frequently requires the use of an orbital implant to bridge the floor defect.

Answer: B

Rationale:
A CT scan of an orbit that has a trapdoor appearance with clinical and radiographic evidence
of inferior rectus muscle incarceration is more like to occur in children than in adults and it
warrants early intervention to prevent ischemic necrosis to the rectus muscle. Steroids are
may be used in cases where there is no evidence of muscle entrapment and mild restricted
motility. By its nature, a trapdoor defect seldom requires an implant to bridge a defect.

Reference:
Ochs MW, Orbital and ocular trauma. In: Miloro M ed, Peterson's Principles of Oral and
Maxillofacial Surgery Second edition BC Deker, London 2004 Chapter 24 pp 479

The American Board of Oral and Maxillofacial Surgery 48


51. Calcium phosphate bone cement used for pediatric frontal sinus obliteration:

A. is an almost ideal alloplast for this application.


B. breaks up when placed directly over dura or sinus membranes.
C. is not replaced by bone and remains inert.
D. is readily available and inexpensive.

Answer: B

Rationale:
While calcium phosphate bone cement is a near ideal alloplast in that it has unlimited
availability, is biocompatible, has high mechanical strength, has a low risk for infection, is
radiopaque and is incorporated into the bone, it does have drawbacks. It is expensive, and
when placed directly over dura or sinus membranes, it breaks up and a chronic foreign body
reaction ensues. This is particularly true for the pediatric population.

Reference:
D'Addario M, Haug RH, Talwar R: Biomaterials for Use in Frontal Sinus Obliteration.
Journal of Long-Term Effects of Medical Implants. 14: 455-465, 2004.

The American Board of Oral and Maxillofacial Surgery 49


53. 2 transferrin is found only in:

A. cerebrospinal fluid, aqueous humor and perilymph.


B. cerebrospinal fluid, serum and aqueous humor.
C. cerebrospinal fluid, nasal secretions and perilymph.
D. serum, nasal secretions and aqueous humor.

Answer: A

Rationale:
2 transferrin is one of two major variants of iron binding transferrin glycoproteins forum in
several forms of various body fluids. 2 transferrin lacks a carbohydrate side chain which
increases its positive charge, permitting isolation during electrophoresis. 2 transferrin is only
found in cerebrospinal fluid, aqueous humor and perilymph. Thus, with the exception of
contamination by a ruptured globe, it is an ideal substance to distinguish between
cerebrospinal fluid, nasal secretions and serum.

Reference:
Brandt MT, Jenkins WS, Fattahi TT, Haug RH:Cerebrospinal Fluid:Implications in Oral and
Maxillofacial Surgery. J Oral Maxillofac Surg. 60:1049-1056, 2002.

The American Board of Oral and Maxillofacial Surgery 50


54. In the recumbent position, normal intracranial pressure is:

A. 1 to 5 mm Hg.
B. 8 to 12 mm Hg.
C. 15 to 19 mm Hg.
D. 22 to 26 mm Hg.

Answer: B

Rationale:
In the recumbent position, intracranial pressure, and thus cerebral spinal fluid pressure, is
about 8 to 12 mm Hg or 110 to 150 mm H2O. Autoregulation maintains this pressure.

Reference:
Brandt MT, Jenkins WS, Fattahi TT, Haug RH:Cerebrospinal Fluid:Implications in Oral and
Maxillofacial Surgery. J Oral Maxillofac Surg. 60:1049-1056, 2002.

The American Board of Oral and Maxillofacial Surgery 51


55. The cause of permanent visual loss subsequent to traumatic hyphema is:

A. elevated intraocular pressure.


B. retinal artery spasm.
C. traumatic mydriasis.
D. corneal staining.

Answer: D

Rationale:
While concomitant injury has been associated with hyphema and permanent visual loss, it is
not the primary cause in traumatic hyphema. Retinal artery spasm is not associated with
hyphema. Both increased intraocular pressure and traumatic mydriasis are concomitant
problems. Corneal staining from myoglobin is the primary cause of permanent loss of vision.

Reference:
Brandt MT, Haug RH:Traumatic Hyphema: A Comprehensive Review. J Oral Maxillofac
Surg. 59:1462-1470, 2001.

The American Board of Oral and Maxillofacial Surgery 52


57. The differences between the child and adult respiratory apparatus are that children possess:

A. greater diaphragmatic breathing


B. high lung compliance.
C. ribs and sternum that are rigid.
D. a low metabolic rate.

Answer: A

Rationale:
The respiratory apparatus and physiology in children differs from adults in many ways.
Children have a high metabolic rate, thus a high oxygen demand and hypoxemia develops
rapidly their upper airway has a smaller caliber, large soft tissues, larynx that is positioned
cephalad, with a shorter trachea, and epiglottis that is short and narrow. This results in
increased airway resistance, easy obstruction, difficult intubation, easy extubation. The child's
ribs and sternum are compliant, and when respiratory efforts are diminished, their reserve
decreases. The child relies more on diaphragatic breathing, therefore high intrathoracic
pressure or abdominal distention diminishes ventilation. The child possesses low lung
compliance and therefore ventilation is inefficient during respiratory distress.

Reference:
American College of Surgeons Committee on Trauma:ATLS Advanced Trauma Life Support
Program for Doctors. 7th Edition, American College of Surgeons, Chicago, IL, 2004, pgs
243-262.

The American Board of Oral and Maxillofacial Surgery 53


58. The circulatory system for children in trauma differs from adults in that children have a
proportionally:

A. higher cardiac output.


B. smaller stroke volume.
C. equal blood volume.
D. decreased physiologic reserve.

Answer: B

Rationale:
Children possess a higher cardiac output than adults. Under the physiologic stress of trauma,
the child's stroke volume cannot increase, so that under stress proportionate increases in
stroke volume cannot occur and increases in cardiac output are rate determined. Bradycardia
could result in hypoxia and hypercapnia, and is ominous. The child's proportional blood
volume is greater than an adult. However, the absolute blood volume and therefore tolerable
absolute blood loss is less, so that minor injuries may result in blood loss which is
physiologically significant. The child's blood pressure is maintained through physiologic
compensation (vasoconstriction, tachycardia, myocardial contractility). This compensation
may be misleading by masking volume reduction. The healthy child has an increased total
systemic physiologic reserve, thus with trauma, symptoms of hypoxemia or hypovolemia may
not occur until a potentially catastrophic hypovolemia has occurred.

Reference:
American College of Surgeons Committee on Trauma:ATLS Advanced Trauma Life Support
Program for Doctors. 7th Edition, American College of Surgeons, Chicago, IL, 2004, pgs
243-262.

The American Board of Oral and Maxillofacial Surgery 54


59. Cervical spine fractures concomitant to maxillofacial injury are most often:

A. burst fractures.
B. hangman fractures.
C. Jefferson fractures.
D. subluxations.

Answer: D

Rationale:
When considering maxillofacial injury and cervical spine injury, a cause and effect
relationship exists between the mandible and cervical spine. There is also a relationship
between neurocranial injury and mid- and upper-facial third facial injuries. Approximately
one out of every fifty patients with a mandibular fracture will suffer a cervical spine fracture.
The vast majority of these are subluxation injuries. When the mandible sustains a force
applied to it, the reaction of the body is to move away from the force. This twisting of the
neck, when accentuated, results in a subluxation injury of the cervical spine.

Reference:
Haug RH, Wible RT, Likavec MJ, et al:Cervical spine fractures and maxillofacial trauma. J
Oral Maxillofac Surg 49:725-729, 1991.

The American Board of Oral and Maxillofacial Surgery 55


60. The difference between infected human and animal bites is the presence of:

A. pasteurella multocida.
B. staphylococcus aureus.
C. eikenella corrodens.
D. fusobacterium nucleatum.

Answer: A

Rationale:
The human and animal population is very similar in their oral flora. Each has a predomination
of Streptococcus species and numerous gram negative anaerobic organisms. Each has
numerous species of viruses and some mycobacteria have been reported. It is however,
Pasteurella multocida that is only found in the non-human population

Reference:
Haug RH, Assael L, Infections in the Maxillofacial Trauma Patient, Chapter 17 in Topazian
RG, Goldberg MH, Hupp J (Eds.). Oral and Maxillofacial Infections. Philadelphia PA, W B
Saunders, 2002, pp 359-380.

The American Board of Oral and Maxillofacial Surgery 56


61. Treatment for inflammatory fibrous hyperplasia where areas of gross tissue redundancy are
present, is best treated by excision with:

A. electrosurgical technique.
B. laser technique.
C. primary closure.
D. secondary epithelialization.

Answer: D

Rationale:
When areas of gross tissue redundancy are found, excision frequently results in total
elimination of vestibule. In such cases excision of the epulides, with peripheral mucosal
repositioning and secondary epithelialization, is preferred.

Reference:
Ochs, MW, Tucker, MR; Preprosthetic Surgery in Peterson, Ellis, Hupp, Tucker.
Contemporary Oral & Maxillofacial Surgery 4th edition. Page 274, Mosby, 2003 USA.

The American Board of Oral and Maxillofacial Surgery 57


62. Which of the following is true regarding submucosal vestibuloplasty?

A. A postoperative stent is essential


B. A superiosteal dissection is performed
C. Split-thickness skin grafts are required
D. Commonly used in the mandible

Answer: A

Rationale:
Distortion/inversion of the upper lip when a mirror is placed to the depth of the maxillary
anterior vestibule indicates a lack of vestibular depth (which is why the lip distorts). The
Submucous vestibuloplasty is contraindicated in this instance. An open type vestibuloplasty
with secondary skin graft or laser vestibuloplasty is a better choice.

Reference:
Spagnoli, DB, Gollehon, SG, Misiek, DJ; Preprosthetic and Reconstructive Surgery in
Peterson's Principles of Oral and Maxillofacial Surgery, Miloro, M. et al., Editors, Second
Edition, BC Decker Inc, 2004, pp. 174-175

The American Board of Oral and Maxillofacial Surgery 58


63. For alveolar distraction to be successful to increase alveolar ridge height and width it is
imperative that the:

A. transport segment not be stabilized by screws to prevent resorption.


B. periosteum be maintained on the crestal alveolus of the transport segment.
C. transport segment be increased in size due to resorption during the process.
D. process be completed as quickly as possible.

Answer: B

Rationale:
Distraction of the alveolus is successful only if the periosteal blood supply is maintained to
the transport segment. The segment must be secured to the device to keep it stable during the
process by rigid fixation. Minimal resorption of the transport segment occurs due to the intact
periosteal blood supply. The process must be governed by the latency, activation, and
consolidation phases and not hurried

Reference:
Spagnoli, DB, Gollehon, SG, Misiek, DJ; Preprosthetic and Reconstructive Surgery in
Peterson's Principles of Oral and Maxillofacial Surgery, Miloro, M. et al., Editors, Second
Edition, BC Decker Inc, 2004, pp183-4

The American Board of Oral and Maxillofacial Surgery 59


64. One of the most critical complications involved in genial tubercle reduction is:

A. over-reduction of the tubercle.


B. under-reduction of the tubercle.
C. partial detachment of the genioglossus muscle.
D. hematoma of the floor of the mouth.

Answer: D

Rationale:
Reduction of the genial tubercle is performed to create an appropriate base for a denture when
significant mandibular resorption has occurred. Under reduction will not provide the desired
alveolar surface, over reduction may result in complete muscle detachment and difficulty
swallowing for several months until reattachment occurs. The genioglossus muscle is partially
detached or may require complete detachment to adequately reduce the tubercle, it will re
attach independently. Floor of the mouth hematoma may cause airway embarrassment and an
emergent situation.

Reference:
Spagnoli, DB, Gollehon, SG, Misiek, DJ; Preprosthetic and Reconstructive Surgery in
Peterson's Principles of Oral and Maxillofacial Surgery, Miloro, M. et al., Editors, Second
Edition, BC Decker Inc, 2004, pp169-170.
Davis et al. Soft Tissue Procedures in Reconstructive and Preprosthetic Surgery, Fonseca and
Davis Editors, Second Edition. WB Saunders Co., 1995. pp748, 759-760

The American Board of Oral and Maxillofacial Surgery 60


65. Which of the following statements is true regarding alveolar distraction?

A. The consolidation period is the time after the osteotomy and prior to distraction
B. Activation occurs 3mm/day in 3 divided, equal segments per day
C. The consolidation phase should be three times the length of the active distraction period
D. The latency period should be shortened for distraction of irradiated bone

Answer: C

Rationale:
After the osteotomy is performed and the distraction device placed, a latency period must be
observed. The latency period can be from 4-7 days depending on age of patient, blood supply,
irradiation to the area, scar tissue, etc. Secondary to impaired blood supply, the latency period
should be longer in irradiated bone. The latency period is followed by the active distraction
period which varies depending on the transport distance. The rate and rhythm of distraction is
how far and how often. A rate of 1mm/day at a rhythm of .25mm four times a day is ideal.
The consolidation period begins when active distraction stops and is generally three times the
active distraction period.

Reference:
Spagnoli, DB, Gollehon, SG, Misiek, DJ; Preprosthetic and Reconstructive Surgery in
Peterson's Principles of Oral and Maxillofacial Surgery, Miloro, M. et al., Editors, Second
Edition, BC Decker Inc, 2004, pp184-5

The American Board of Oral and Maxillofacial Surgery 61


66. When performing a maxillary labial frenectomy where the base of the frenum is extremely
wide, the most effective surgical technique is:

A. Z-plasty technique.
B. simple excision (diamond excision).
C. localized vestibuloplasty with secondary epithelialization.
D. kazanjian technique (lip switch).

Answer: C

Rationale:
Localized vestibuloplasty with secondary epithelialization has been described as the most
beneficial for broad frenum attachments. The Z-plasty technique is useful when the mucosal
and fibrous band is narrow. The diamond excision can result in scarring and relapse. The
lip-switch is used for mandibular vestibuloplasty.

Reference:
Fonseca, Davis. Reconstructive Preprosthetic Oral & Maxillofacial Surgery (1st Edition).
Page 65, W.B Saunders Company, 1986 Philadelphia
Ochs, MW, Tucker, MR; Preprosthetic Surgery in Peterson, Ellis, Hupp, Tucker.
Contemporary Oral & Maxillofacial Surgery 4th edition. pp 275-278, Mosby, 2003 USA.

The American Board of Oral and Maxillofacial Surgery 62


68. Which of the following is more characteristic of a thin split-thickness skin graft (STSG) when
compared to a thick STSG?

A. Increased probability of graft survival


B. Decreased secondary contracture of the graft
C. More likely to result in recipient site hair growth
D. Slower donor site re-epithelialization

Answer: A

Rationale:
The thin STSG is more likely to survive on its recipient site because it can survive well during
the phase of plasmatic absorption and therefore wait longer for vascularization. The thicker
the graft, the less its tendency to undergo secondary contraction; full-thickness skin grafts
show little or no evidence of contracture. The thicker the graft, the more likely a hair follicle
will be transplanted. The thinner a graft, the more accessory skin structures remain at the
donor site for epithelial growth.

Reference:
Fonseca, Davis. Reconstructive Preprosthetic Oral & Maxillofacial Surgery (1st Edition).
Page 35, W.B Saunders Company, 1986 Philadelphia

The American Board of Oral and Maxillofacial Surgery 63


69. The angiogenic effects of hyperbaric oxygen therapy occur at regions of:

A. tissue hypoxia.
B. a shallow oxygen gradient
C. a steep oxygen gradient
D. tissue hyperoxia

Answer: C

Rationale:
HBO creates an oxygen gradient in radiated tissue which does not normally exist. This steep
oxygen gradient stimulates the normal wound healing mechanism that is missing in radiated
tissue. The effects of HBO fall off when the tissue is sufficiently revascularized so that it no
longer generates a sufficient oxygen gradient to continue the angiogenic process.

Reference:
Fonseca, Davis. Reconstructive Preprosthetic Oral & Maxillofacial Surgery (2nd Edition).
Page 1107-08, W.B Saunders Company, 1995 Philadelphia.

The American Board of Oral and Maxillofacial Surgery 64


70. When is the most appropriate time to perform a labial frenectomy to facilitate closure of a
diastema between teeth numbers 8 and 9?

A. Prior to eruption of the maxillary lateral incisors


B. Prior to eruption of the maxillary canines
C. During eruption of the maxillary canines
D. After complete eruption of the maxillary canines

Answer: C

Rationale:
Maxillary labial frenectomy should be done following the eruption of the lateral incisors and
during active eruption of the maxillary canines. The combination of a correctly timed and
executed maxillary frenectomy and the anterior eruption force of the maxillary canines should
lead to the closure of the diastema.

Reference:
Kearns, G, Pediatric Dentoalveolar Surgery in Fonseca, RJ, Oral and Maxillofacial Surgery
Vol 1, Anesthesia/Dentoalveolar Surgery/ Office Management, WB Saunders 2000. pp 360-
361.

The American Board of Oral and Maxillofacial Surgery 65


71. Which of the following statements is true regarding the Bio-Col ridge preservation technique?

A. Primary closure is necessary


B. Requires use of some autogenous bone graft
C. Should not be used in the esthetic zone
D. Can be used with immediate or delayed implant placement

Answer: D

Rationale:
Site preservation usually begins at the time of tooth removal. In aesthetic areas, the referenced
author uses the Bio-Col alveolar ridge preservation technique to preserve hard and soft tissue
alveolar ridge anatomy in preparation for immediate or delayed implant placement. This
technique can be used to reduce or avoid osseous ridge resorption by minimizing trauma
during tooth removal. The prepared extraction sockets or the voids surrounding the
immediately placed implant are then grafted with Bio-Oss (Osteohealth, Shirley, NY), a
natural, porous bone-grafting material. Subsequently, the grafted socket is isolated with an
absorbable collagen material (Collaplug; Zimmer Dental, Carlsbad, CA) that has been coated
with an impervious tissue cement (Isodent; Ellmann International, Hewlett, NY); this allows
for guided bone regeneration without the need for flap elevation and primary closure, thus
preserving the surrounding soft-tissue volume. Finally, the scalloped soft tissue architecture is
preserved with the use of interim provisional restorations, anatomic healing abutments, or
custom tooth-form healing abutments designed to support the marginal tissues and interdental
papillae. This is of critical aesthetic importance for implant patients who present with thin
scalloped periodontal biotypes who are predisposed to loss of alveolar ridge volume
secondary to remodeling, resorption of bone following tooth removal, and soft tissue
recession following subsequent surgical or restorative interventions.

Reference:
Sclar AG, Strategies for management of single-tooth extraction sites in aesthetic implant
therapy. J Oral N:Maxillofac Surg. 2004 Sep;62(9 Suppl 2) 90-105.
Sclar AG, Preserving alveolar ridge anatomy following tooth removal in conjunction with
immediate implant placement. The Bio-Col technique. Atlas Oral Maxillofac Surg Clin
North Am. 1999 Sep;7(2): 39-59.

The American Board of Oral and Maxillofacial Surgery 66


72. A 48-year-old woman presents with a history of metastatic breast carcinoma, previous
mastectomy, and radiation to the chest. She continues periodic chemotherapy infusions for
control of her disease. Her general dentist extracted tooth #30 six weeks previous to referring
the patient to you for a non-healing socket and bone exposure in the area. The most likely cause
of the non-healing site is:

A. failure to attain primary closure at the time of extraction.


B. osteoradionecrosis secondary to the radiation therapy.
C. bisphosphonates included in the chemotherapy regimen.
D. traumatic extraction with failure of the patient to follow instructed home care.

Answer: C

Rationale:
Bisphosphonates are widely used in the management of metastatic disease to the bone and in
the treatment of osteoporosis. Recently, there is emerging evidence that these drugs are
etiologically related to osteonecrosis of the jaws. Answer A is incorrect because primary
closure is not possible or required. Although radiotherapy is known to be a cause of
osteonecrosis, the stem of the question explicitly indicates radiation to the chest; therefore,
option B is incorrect. Option D is also incorrect since even if traumatic extraction of a tooth
occurred, healing should have been complete after six weeks, even without appropriate home
care.

Reference:
Ruggiero, S.L., Mehrotra, B., Rosenberg, T.J., and Engroff, S.L.:Osteonecrosis of the Jaws
Associated with the use of Bisphosphonates:A Review of 63 cases. J. Oral Maxillofac Surg
62:527-534, 2004.

The American Board of Oral and Maxillofacial Surgery 67


73. A 70-year-old diabetic man who has a history of left sinus symptoms was referred to you for
extraction of erupted and loose tooth #15. Your examination revealed marked necrosis of
osseous and soft tissue of the left maxilla. You removed several teeth, the necrotic bone and soft
tissue, and submitted the specimen to pathology. The pathologist reported necrotic bone and soft
tissue with inflammatory infiltrate and the presence of nonseptate irregularly wide fungal hyphae
with frequent right- angle branching. Your diagnosis is:

A. Aspergillosis.
B. Candidiasis.
C. Actinomycosis.
D. Mucormycosis.

Answer: D

Rationale:
The correct answer to this question requires specific knowledge of the morphology of fungi.
Aspergillus forms fruiting bodies and septate filaments branching at acute angles. Candida
grows as yeast forms, tandem arrays of elongated forms without hyphae (pseudohyphae), and
true hyphae with septate. Actinomyces is a gram-positive filamentous or rod-shaped bacteria.
Mucormycosis form nonseptate, irregularly wide fungal hyphae with frequent right-angle
branching. In addition mucormycosis frequently occur in diabetics and may spread from
nasal sinuses to the orbit and brain-giving rise to rhinocerebral mucormycosis.

Reference:
Regezi:Oral Pathology:Clinical Pahtologic Correlations, 4th., 2003 Elsevier, pp 37-38.
Strauss, R.A.:Fungal Infections of the Head and Neck, In:Oral & Maxillofacial Surgery
Clinics of North America. Vol. 3, Number 2, W. B. Saunders 1991, pp 295-309.

The American Board of Oral and Maxillofacial Surgery 68


74. During extraction of teeth #14 and #15, the two teeth were delivered with a large attached piece
of the buccal alveolar plate. An 8 mm diameter oro-antral communication is noted. The most
appropriate treatment course is:

A. attempt suturing as best as possible with available local tissue.


B. develop a buccal mucoperiosteal flap and advance it to achieve primary closure without
tension.
C. develop a free palatal mucosa graft to passively cover the defect.
D. immediately proceed with packing the sinus with iodoform gauze saturated with triple
antibiotics and perform a nasal antrostomy.

Answer: B

Rationale:
Acute oroantral communication is a common occurrence during extraction of the posterior
maxillary teeth. In most instances these perforations are small and the blood clot that fills the
extraction site usually seals such small perforations. If there is an infection in the antrum, the
defect is large (>5 mm in diameter), the gingival tissue are not approximated, the wound is
dehisced, or the patient does not follow the postoperative instructions, an oro-antral fistula
occurs. Although option A seems reasonable for a small fistula, due to the size of the defect,
local suturing without flap development will most likely not result in primary closure, and
then result in an oro-antral communication. Option B offers the best chance of primary
closure of the large communication and prevents development of de novo sinusitis. A free
graft cannot survive over a hole without its own blood supply and thus option C is incorrect.
Option D is also incorrect because it treats chronic sinusitis. There is no indication in the
stem of the question that sinusitis occurred. Closure of the communication is not addressed.

Reference:
Laskin, D.M.:Management of Oro-antral fistula and other sinus-related complications.
In:Oral Maxillofac Surg Clinic North America Vol. Number 1, Feb. 1999, W.B. Saunders,
Philadelphia, pp 155-179.

The American Board of Oral and Maxillofacial Surgery 69


75. During extraction of erupted tooth #19, the mesial root tip fractures. During your attempt to
elevate, the fragment disappears through the socket. The most appropriate surgical approach to
retrieve the root tip is:

A. create a buccal flap and remove the buccal alveolar plate to gain access.
B. develop a lingual flap.
C. a Risdon approach.
D. widen the cortical defect through which the root disappeared.

Answer: B

Rationale:
The mylohyoid muscle forms the floor of the mouth attaching to the mylohyoid ridge. The
latter curves superiorly as it approaches the third molar. Roots of first molars are usually
superior to level of mylohyoid muscle attachment to the mandible. The root tips of third
molars are usually inferior to the muscle attachment. Also, the lingual plate is thin in this site.
Therefore, option B is correct. Options A and D are both incorrect since extensive procedures
that may not gain visualization of the displaced root, are required. Option C is incorrect
because anatomically the root is likely to be superior to mylohyoid. If a third molar root was
displaced, it may be inferior to the mylohyoid. In this latter situation the submandibular
approach may be necessary.

Reference:
Gregg, J.M.:Surgical Anatomy. In Oral and Maxillofacial Surgery, Vol. 1, edit:D.M. Laskin.
The C.V. Mosby Company, St. Louis, 1980, pp 3-49.
Pogrel, M.A.:Complications of Third Molar Surgery. In Oral and Maxillofacial Surgery
Clinics of North America Vol 2, Number 3, 1990, W.B. Saunders Company. Pp. 441-451.

The American Board of Oral and Maxillofacial Surgery 70


76. Which erupted maxillary tooth is usually associated with the highest rate of oral-antral
communication after routine extraction?

A. Maxillary second premolar


B. Maxillary first molar
C. Maxillary second molar
D. Maxillary third molar

Answer: B

Rationale:
The palatal root of the maxillary first molar is usually the most closely related to the floor of
the maxillary sinus.

Reference:
Alling CC (Ed). Dentoalveolar Surgery. Oral and Maxillofacial Surgery Clinics of North
America. Vol 5. WB Saunders. Philadelphia, 1993

The American Board of Oral and Maxillofacial Surgery 71


77. During routine extraction of a maxillary first molar with radiographic periapical pathology, a
5mm fragment of the palatal root is dislodged into the maxillary sinus. What is the most
appropriate next step?

A. Obtain a CT scan of the maxillary sinus


B. Completion of an antibiotic course and observation
C. Attempt careful visualization and retrieval through the extraction socket
D. Perform a Caldwell-Luc antrostomy

Answer: C

Rationale:
The most appropriate first step in this situation is to attempt visualization and retrieval of the
displaced root tip through the extraction socket. If that is unsuccessful subsequent measures
should include further radiographic documentation of the root tip position and discussing the
operative findings with the patient. Initial localization with periapical or panorex x-rays
would be appropriate. If conservative surgical approaches are unsuccessful, then a Caldwell-
Luc may be appropriate.
In general, root tips between 1mm and 3mm and are without infection or pathology can be
managed expectantly. A root that is 2mm or greater and exhibits infection or pathology
should be removed.

Reference:
Wells, DL, Capes, JO, Powers, MP; Complications of Dentoalveolar Surgery in Fonseca, RJ,
Oral and Maxillofacial Surgery Vol 1, Anesthesia/Dentoalveolar Surgery/ Office
Management, WB Saunders 2000. p432.

The American Board of Oral and Maxillofacial Surgery 72


78. A 23-year-old male is involved in sports related injury and sustains a displacement of his
maxillary right central incisor. Which direction of displacement is more likely to lead to pulpal
necrosis?

A. Intrusion
B. Extrusion
C. Lateral
D. Palatal

Answer: A

Rationale:
Intrusive forces are more likely to cause direct compressive injury to the pulpal vascular
supply than forces in any other direction. The risk of pulpal necrosis after intrusion of teeth
with a closed apex is 95% and 65% if the apex is open.

Reference:
Andreassen JO, Andreassen FM:Essentials of Traumatic Injuries to the Teeth 2nd Edition;
Munksgaard, Copenhagen, 2000, pp 77-112.

The American Board of Oral and Maxillofacial Surgery 73


79. Which of the following materials used for retrograde fill of an apicoectomy has the lowest
success rate?

A. IRM (intermediate restorative material)


B. SuperEBA (ethoxy benzoic acid)
C. Amalgam
D. MTA (Mineral Trioxide Aggregate)

Answer: C

Rationale:
MTA, IRM, and SuperEBA all have success rates greater than that of a retrograde amalgam.
Following retrograde preparation with an ultrasonic microtip or use of intraoperative
magnification, the success rates of these three materials range from 85-97%. The success rate
of traditional retrograde amalgam filling is reported to be much lower, around 68%.

Reference:
von Arx T. Failed root canals: the case for apicoectomy (periradicular surgery). Journal of
Oral & Maxillofacial Surgery. 63(6):832-7, 2005 Jun.

The American Board of Oral and Maxillofacial Surgery 74


80. Which of the following is a characteristic of cracked tooth syndrome?

A. Percussion sensitivity
B. Pain on release of bite force
C. More likely to occur in a tooth without restorations.
D. The most appropriate treatment is extraction.

Answer: B

Rationale:
Cracked tooth syndrome frequently occurs in teeth that have had 1 or more restorations in the
marginal ridge area. The tooth frequently exhibits pain on release of bite force, rather than on
percussion. Restoration with onlays, composite resin, or splinting with an orthodontic band is
frequently successful. Extraction is necessary only if the tooth root fractures vertically.

Reference:
Homewood CI. Cracked tooth syndrome--incidence, clinical findings and treatment.
Australian Dental Journal. 43(4):=217-22, 1998 Aug.

The American Board of Oral and Maxillofacial Surgery 75


81. The above axial CT scans are from a patient who developed instantaneous facial swelling during
surgical extraction of an erupted tooth utilizing a high-speed dental handpiece. The scans were
taken immediately following the procedure. What is the most appropriate management?

A. An antibiotic course and observation


B. Hyperbaric oxygen therapy
C. A cervical drainage procedure
D. A mediastinal drainage procedure

Answer: A

Rationale:
These are axial CT scans of air emphysema. The use of a high-speed dental handpiece for the
procedure and the instantaneous swelling are characteristic of this phenomenon. The air was
introduced by the handpiece into the cheek, and has progressed along fascial planes into the
carotid sheath and then the mediastinum. The treatment of tissue emphysema varies with the
severity of the condition. Most cases of subcutaneous emphysema will begin to resolve after
2 to 3 days of supportive treatment, and residual swelling is usually minimal after 7 to 10 days
of observation. Treatment is usually conservative, and consists of antibiotic coverage to
prevent infection. Oral bacteria may possibly be carried with the aerosol into the soft tissue
and represent a potential nidus of infection. Additionally, a course of systemic corticosteroids
may promote faster resolution. Surgical decompression of the extensive emphysema should
not be routinely used, because it is likely to be ineffective and may even worsen or spread the
emphysema. In most cases the patient can be discharged with an explanation of the nature of
the tissue emphysema and its course.

Reference:
Peterson LJ. Emphysema and dental drill (comment). J Am Dent Assoc 1990; 120:423
Pynn BR, Amato D, Walker DA. Subcutaneous emphysema following dental treatment:report
of two cases and review of the literature. J Can Dent Assoc 1192; 58:496-499
Schuman NJ, Edwards BC, Walker W. Subcutaneous emphysema during operative
dentistry:report of a case with a thirty-month follow up. J Oral Med 1983; 38:168-169
Woehrlen AE. Subcutaneous emphysema. Anesth Prog 1985; 32:161-163
Salib RJ, Valentine P. Surgical emphysema following dental treatment. The Journal of
Laryngoscopy and Otology 1999; 113:756-758

The American Board of Oral and Maxillofacial Surgery 76


82. Fracture of alveolar process during the extraction of a tooth is usually due to:

A. use of excessive amounts of uncontrolled force.


B. presence of adjacent crowded teeth.
C. periapical infection.
D. expansion of surrounding alveolar bone.

Answer: A

RATIONALE:
If the alveolus is fractured, dissect the alveolus off the root, replace the periosteum and
alveolus which will act as a vascularized graft. If soft tissue is inadvertently dissected from
the alveolus, the segment of bone will likely undergo necrosis. If the maxillary tuberosity is
fractured, and the tooth cannot be dissected from the alveolus, replace and splint for 6 8
weeks. Later surgically extract the tooth. If the tuberosity is completely avulsed without soft
tissue pedicle, then smooth the remaining bony edges and close, checking for oral antral
communication.

Reference:
Peterson, LJ; Prevention and Management of Complications in Peterson, Ellis, Hupp, Tucker.
Contemporary Oral & Maxillofacial Surgery 4th edition. pp 228-230., Mosby, 2003 USA.
Peterson LJ:Contemporary Oral and Maxillofacial Surgery, 3rd edition Ed. Pg. 261 262.
Mosby, Missouri

The American Board of Oral and Maxillofacial Surgery 77


83. If the lingual alveolar plate is fractured and mobile during removal of an erupted mandibular
third molar, the fractured segment should be:

A. stabilized to avoid damage to the lingual nerve.


B. left in place with minimal manipulation.
C. removed with careful subperiosteal dissection.
D. removed and the lingual nerve explored for evidence of injury.

Answer: B

Rational:
Minimal manipulation of the segment will provide the greatest chance of maintaining the
periosteal attachment, and thus the blood supply to the segment. If soft tissue is
inadvertently dissected from the alveolus, the segment of bone will likely undergo necrosis.
Removal of the segment is generally not advised, dissection in this region would increase the
risk of injury to the lingual nerve.

REFERENCE:
Berman SA. Basic principles of dentoalveolar surgery. In:=Petersen LJ, Indresano AT,
Marciani RD, Roser SM. Principles if Oral and Maxillofacial Surgery. Philadelphia, PA; JB
Liincott CO; 1992, pp. 95-96.
Peterson, LJ; Prevention and Management of Complications in Peterson, Ellis, Hupp,
Tucker. Contemporary Oral & Maxillofacial Surgery 4th edition. pp 228-230., Mosby, 2003
USA.
Peterson LJ:Contemporary Oral and Maxillofacial Surgery, 3rd edition Ed. Pg. 261 262.
Mosby, Missouri

The American Board of Oral and Maxillofacial Surgery 78


84. The biological width surrounding natural teeth and freestanding osseointegrated root form
implants demonstrate a relatively constant thickness of:

A. 1mm.
B. 3 mm.
C. 5 mm.
D. 7 mm.

Answer: B

Rationale:
An appreciation of biologic width is important in best determining implant positioning to
allow for an aesthetic result. Numerous studies document the dimensions of biological width
for both natural teeth and implants. Biologic width is comprised of the zone of supracrestal
connective tissue that measures approximately 1 mm and epithelial structures, including the
junctional and sulcular epithelium that measure about 2 mm in height.

Reference:
Burglund H, T. and Lindhe, J. Dimensions of the Peri-Implant Mucosa; Biological Width
Revisited. Journal of Clinical Periodontology, 1996. Vol. 23, pages 971-973.
Herman, J. S, Buser, D., Shenk, R.K., Higginbottom, F.L., and Cochran, D. L. Biological
Width Around Titanium Implants:A Physiologically Formed and Stable Dimension Over
Time. Clinical Implants Res., 2000. Vol. 11, pages 1-11.

The American Board of Oral and Maxillofacial Surgery 79


85. Which of the following can cause premature loss of temporary implants used for the purpose of
providing orthodontic anchorage?

A. Non-keratinized surrounding mucosa


B. Type of intended tooth movement
C. Mini-plate implant configuration
D. Mini-screw implant configuration

Answer: A

Rationale:
While success rates for mini-implants are high, studies like that of Chang and Tseng
demonstrate that implants placed through non-keratinized mucosa have a higher rate of
failure. Another factor that may have an adverse affect on success is placement in the
posterior mandible. Implant type (mini-screw of mini-plate) does not effect failure rates of
rigid orthodontic anchorage.

Reference:
Chang, S and Tseng, I, International Journal of Oral and Maxillofacial Implants, 2004, Vol.
19, pages 100-106,

The American Board of Oral and Maxillofacial Surgery 80


86. The most important factor associated with the success of osteotome-mediated sinus floor
elevation technique is:

A. the height of the residual alveolar bone.


B. implant design.
C. type of graft material used.
D. method of sinus in-fracture.

Answer: A

Rationale:
The ability to obtain primary implant stability is associated with the height of the residual
alveolar ridge, and is the primary factor related to success when implants are placed in
conjunction with osteotome mediated sinus floor elevation. Implant design graft material and
method of sinus in-fracture have minimal influence on survival; however, factors such as
osteoporosis, type of final prosthesis, and operator experience may also impact the success of
this technique.

Reference:
Toffler, M., Osteotome Mediated Sinus Floor Elevation: Clinical Report. International
Journal of Oral and Maxillofacial Implants, Vol. 19, No. 2, 2004, pp. 266273.

The American Board of Oral and Maxillofacial Surgery 81


87. Which of the following has a favorable aesthetic impact on an implant-supported restoration?

A. Thick biotype
B. Thin biotype
C. Restorative table >6 mm apical to the contact point of the adjacent tooth.
D. Positive IL-1 genotype

Answer: A

Rationale:
A thick biotype provides greater stability for the peri-implant soft tissues with more
predictable healing response following surgery and around restorations. Thin biotypes
predispose not only to unpredictable healing but also may allow titanium show through.
When implant restorative tables are placed more than 5 mm below the contact point of an
adjacent tooth, loss of the papilla is common, having an adverse effect on the final aesthetic
result. Finally, genetic testing utilizing a swab has identified that positive intraleuken-1 (IL-1)
genotype patients have an increased risk of developing periodontitis with enhanced risk of
peri-implant complications.

Reference:
Buser, Martin and Belser, Optimizing Aesthetics for Implant Restorations in the Anterior
Maxilla: Anatomic and Surgical Considerations, International Journal of Oral and
Maxillofacial Implants. Vol. 19, pages 43-61. Tonel, Magner, and Fletcher. The Effective
Distance from the Contact Point to the Crestal Bone on the Presence or Absence of
Interproximal Dental Papilla, Journal of Periodontology, 1992. Vol. 63, pages 995-96
.

The American Board of Oral and Maxillofacial Surgery 82


88. Which of the following is the best method to determine the completion of facial growth?

A. Use of hand/wrist films.


B. Completion of changes in skeletal height
C. Use of serial lateral cephalograms
D. Attainment of maturity by the Tanner classification

Answer: C

Rationale:
Completion of skeletal height does not correspond to completion of facial growth. A hand-
wrist radiograph is inappropriate for assessing facial growth, because it is not specific enough
for each patient. The best method of evaluating the completion of facial growth is by
superimposing sequential cephalometric radiographs. Most boys do not complete their facial
growth until the late teenage years. A 14- or 15-year-old boy may not have gone through his
adolescent growth spurt. It is advisable to wait until an adolescent male has completed growth
in height. At that point, a cephalometric radiograph should be taken. Another radiograph
should be taken at least 6 months to a year later. If these radiographs are superimposed, and
there are no changes in vertical facial height (nasion to menton), this indicates that most of the
facial growth has been completed. Achievement of Tanner stage V indicates maturity in
development of secondary sexual characteristics, but not skeletal maturity.

Reference:
Kokich V:Orthodontic-restorative management of the adolescent patient. In Orthodontics
and Dentofacial Orthopedics, eds McNamara JA Jr. Ann Arbor, MI:=Needham Press; 2001.
425-452.
Kokich V:Maxillary lateral incisor implants:=planning with the aid of orthodontics. J Oral
Maxillofac Surg. 2004 Sep;62(9 Suppl 2):=48-56. Marshall, W, Tanner, J. Variations in the
pattern of pubertal changes in girls. Arch Dis Child 1969; 44:=291 Marshall, W, Tanner, JM.
Variations in the pattern of pubertal changes in boys. Arch Dis Child 1970; 45:=13. Law, SJ;
Use of Orthopedic appliances in Growth Modification in Fonseca, RJ, Oral and Maxillofacial
Surgery Vol 6, Cleft/ Craniofacial/ Cosmetic Surgery, WB Saunders 2000. pp 15-17.
Bishara, Facial and dental changes in adolescents and their clinical implications. The Angle
orthodontist, 200 Vol 70:=6,p 471

The American Board of Oral and Maxillofacial Surgery 83


89. Which of the following is an absolute contraindication for dental implant placement?

A. Oral lichen planus


B. Uncontrolled periodontal disease
C. Insulin dependent diabetes mellitus
D. Cigarette smoking

Answer: B

Rationale:
Contraindications for the use of an implant restoration are in developing patients (particularly
in the maxilla, where vertical growth continues after permanent teeth are fully erupted),
uncontrolled periodontal disease, aesthetic areas with thin, highly scalloped gingiva, adjacent
periapical pathology, and non motivated patients. Relative contraindications where adjacent
root flaring precludes placement (correction needed with orthodontics), smokers (increased
failure rate especially in type IV bone), connective tissue diseases, and diabetes and
autoimmune diseases.

Reference:
Salinas TJ, Block MS, Sadan A. Fixed partial denture or single-tooth implant restoration?
Statistical considerations for sequencing and treatment. J Oral Maxillofac Surg. 2004
Sep;62(9 Suppl 2):2-16. Lindquist LW, Carlsson GE, Jemt T. Association between marginal
bone loss around osseointegrated mandibular implants and smoking habits:A 10 year follow-
up study J Dent Res 1997;76:1667. De Bruyn H, Collaert B. The effect of smoking on early
implant failure Clin Oral Implants Res 1994;5:=260 Isidor F, Brondum K, Hansen HJ.
Outcome of treatment with implant-retained dental prostheses in patients with Sjgren
syndrome Int J 1999;14:736. Balshi TJ, Wolfinger GJ. Dental implants in the diabetic
patient:A retrospective study Implant Dent 1999;8:355 Rajnay ZW, Hochstetter RL.
Immediate placement of an endosseous root-form implant in an HIV-positive patient:Report
of a case J Periodontol 1998;69:=1167

The American Board of Oral and Maxillofacial Surgery 84


91. In 75% of patients, in order for the dental papilla to fill the embrasure space of a single tooth
dental implant supported restoration, the maximum distance between the crestal bone and the
contact point is:

A. 6mm.
B. 5mm.
C. 4mm.
D. 3mm.

Answer: A

Rationale:
The presence or absence of a peri-implant papilla mainly depends on the distance between the
alveolar crest and the contact point. In single-tooth gaps, the bone height at adjacent teeth
determines the status of the papilla. A clinical study by Dennis Tarnow , et al, demonstrated
that a distance of 6mm or more from the alveolar crest to the contact point reduces the
probability of intact papillae.

Reference:
Tarnow DP, Magner AW, Fletcher P. The effect of the distance from the contact point to the
crest of bone on the presence or absence of the interproximal dental papilla. J Periodontology
1992;63:995-996

The American Board of Oral and Maxillofacial Surgery 85


92. The osteotome technique for implant placement:

A. is primarily used in the mandible.


B. is used in Type I bone.
C. compromises vascularity.
D. compresses bone laterally.

Answer: D

Rationale:
In soft maxillary bone, preparation of the implant osteotomy site can be completed using a
series of osteotomes rather than burs. Because the maxillary bone is soft, an osteotome can be
used to split or widen a narrow ridge to receive an implant. In addition, the osteotome
condenses and laterally compresses the soft bone at the osteotomy site, placing a denser,
compressed bone immediately adjacent to the implant..

Reference:
Fonseca, RJ, Oral and Maxillofacial Surgery Vol 7, Reconstructive and Implant Surgery, WB
Saunders, 2000, pp 111-114

The American Board of Oral and Maxillofacial Surgery 86


93. In order to maintain the crestal bone between adjacent dental implants, the implants should be
placed at least how far apart?

A. 2.0 mm
B. 2.5 mm
C. 3.0 mm
D. 3.5 mm

Answer: C

Rationale:
In sites with adjacent implants, bone resorption of 1 to 2 mm at the proximal aspects of the
implant leads to a flattening of the inter-implant bone and consequently a short inter-implant
papilla. A distance of at least 3 mm has been recommended between 2 adjacent implants to
minimize this bone resorption.

Reference:
Tarnow DP, et al. The effect of inter-implant distance on the height of inter-implant bone
crest. J Periodontology 2000; 71:=546-549

The American Board of Oral and Maxillofacial Surgery 87


94. When performing immediate placement of an implant into extraction site of a maxillary central
incisor, which of the following statements is true?

A. The tooth socket must be completely obliterated by the dental implant.


B. Immediate placement is not recommended if buccal plate integrity is lost.
C. The thick flat gingival biotype has a higher risk of buccal plate resorption.
D. The implant should be placed adjacent to the buccal wall of the extraction site.

Answer: B

Reference:
Hmmerle CHF, et al, Consensus Statements and Recommended Clinical Procedures
Regarding the Placement of Implants in Extraction Sockets, Int. J of Oral & Maxillofacial
Implants, Vol 19 Supp, pp 26-28

The American Board of Oral and Maxillofacial Surgery 88


95. Soft tissue peri-implantitis occurs most frequently with which of the following implant
restorations?

A. Single tooth replacement


B. Implant and tooth supported fixed prosthesis
C. Implanted supported fixed prosthesis
D. Implant supported overdenture

Answer: D

Rationale:
Soft tissue peri-implantitis occurs most commonly in association with implant supported
overdentures, with a reported frequency of 11% to 32%. Rates of soft tissue peri-implantitis
associated with implant supported fixed prostheses range from 7% to 20%.

Reference:
Ardekian, L; Dodson, TB, Complications associated with the placement of dental
implants:Oral and Maxillofacial Surg Clin NAm15 (2003) 243-249

The American Board of Oral and Maxillofacial Surgery 89


96. When considering immediate loading of a dental implant, the minimum insertional torque with
which the implant must be placed is:

A. 10 - 15 N/cm2.
B. 20 - 25 N/cm2.
C. 30 - 35 N/cm2.
D. 40 - 45 N/cm2.

Answer: C

Rationale:
Studies that use insertion torque values are in general agreement that the values should be at
least 30 to 35 Ncm."

Reference:
Ganeles, J, Wismeijer, D:Early and Immediately Restored and Loaded Dental Implants for
Single-Tooth and Partial-Arch Applications. The International Journal of Oral and
Maxillofacial Implants, Volume 19 Supplement, 2004, page 99.

The American Board of Oral and Maxillofacial Surgery 90


97. The minimal bone height required for placement of a transmandibular implant is how many
millimeters?

A. 3
B. 6
C. 9
D. 12

Answer: A

Rationale:
The cortical screws for the TMI system are available in lengths of 5,8,11, & 14 mm.
Protocol is for the cortical screws to extend beyond the bone for a length of 2mm, but not
pierce the periosteum. This allows for tenting of the periosteum and thought to lead to
subsequent bone growth. A 3mm mandible is the minimum mandibular height to allow 2mm
of tenting with the 5mm cortical screw.

REFERENCE:
Powers MP, Bosker H, Pelt AJW, Dunbar N, The Transmandibular implant reconstruction
system: From progressive bone loss to controlled bone height. JOMS 1994; 52:904-910.
Powers MP, Bosker, H The Transmandibular Reconstruction System in Reconstructive and
Preprosthetic Surgery, Fonseca and Davis Editors, Second Edition. WB Saunders Co., 1995.
pp 565-668.

The American Board of Oral and Maxillofacial Surgery 91


98. When comparing the apically repositioned flap technique and the closed eruption technique for
exposure of an unerupted labially positioned maxillary canine lying high in the alveolus, the
closed eruption technique results in:

A. less periodontal attachment around the canine.


B. a greater risk of intrusive relapse.
C. improved gingival esthetics.
D. excessive canine clinical crown length.

Answer: C

Rationale:
Both techniques allow for maintenance of the attached gingiva. In the closed eruption
technique, a crestal incision is made, and a full-thickness flap is reflected. Bone is carefully
removed around the crown and a bracket attached. The flap is closed and the tooth is allowed
to erupt down through the crestal incision site as traction is placed on the ligature. The
referenced study, comparing the two techniques, shows that although the periodontal
attachment level was the same in the two techniques, the labially impacted maxillary anterior
teeth uncovered w/ apical flap techniques had more unaesthetic scarring, increased clinical
crown length, and greater risk of intrusive relapse. The results of treatment with the closed
technique were aesthetically superior and there was less relapse.

Reference:
Beck, RJ, Powers, MP; Soft Tissue Considerations in Fonseca, RJ, Oral and Maxillofacial
Surgery Vol 7, Reconstruction and Implant Surgery, WB Saunders 2000. p353.
Vermette ME: Uncovering labially impacted teeth: Apically positioned flap and closed
reduction techniques. Angle Orthodontics 1995; 65: 23-32

The American Board of Oral and Maxillofacial Surgery 92


99. In regard to measured periodontal attachment at the distal of the second mandibular molar
following extraction and complete healing of the third mandibular molar, the following is true:

A. the best results are obtained when guided tissue regeneration techniques are used at the
third molar extraction site.
B. the best results are obtained when demineralized bone powder is placed within the third
molar extraction site.
C. third molar extraction sites that undergo no special treatment demonstrate periodontal
attachment levels similar to those treated with guided tissue regeneration techniques or
demineralized bone powder.
D. use of demineralized bone powder within the third molar extraction site more commonly
results in formation of a chronic giant cell reaction and periodontal attachment loss.

Answer: C

Rationale:
At the time of third molar extraction, different techniques can be implemented in an attempt
to improve the periodontal attachment levels distal to the second molar. A study divided
patients undergoing third molar extraction into three different perioperative treatment groups
(Guided tissue regeneration (GTR), Demineralized bone powder (DMP), and no special
treatment). There were 12 patients in each group. There were no statistical differences
between any of the treatment groups in probing depth or attachment level around the second
molar. Neither GTR nor DMP techniques showed any advantage over simple healing
following third molar extraction

Reference:
Dodson TB, Management of mandibular third molar extraction sites to prevent periodontal
defects. JOMS. 2004 Oct; 62(10): 1213-24.

The American Board of Oral and Maxillofacial Surgery 93


100. A 20-year-old presents with a radiolucency associated with an impacted tooth. What is the most
common pathologically significant lesion detected on histopathologic examination of the
pericoronal tissue?

A. Odontogenic keratocyst
B. Ameloblastoma
C. Odontoma
D. Dentigerous cyst

Answer: D

Rationale:
The cited study reported the histopathologic diagnoses of a large series of pericoronal lesions
in adults submitted to an oral and maxillofacial pathology biopsy service. Of the 2646 lesions
submitted, 33% showed demonstrable pathology. The most common pathology was
dentigerous cyst (28.4%), followed by OKC (3%), odontoma (0.7%), and ameloblastoma
(0.5%).

Reference:
Curran A, Pathologically significant pericoronal lesions in adults: Histopathologic evaluation.
JOMS. 2002 Jun; 60(6): 613-7

The American Board of Oral and Maxillofacial Surgery 94


101. What stage of root development is optimal for the surgical uprighting of an impacted mandibular
second molar?

A. Less than 1/3 root


B. 1/3
C. 2/3
D. Complete

Answer: C

Rationale:
The procedure is best performed after 2/3 of root development is completed. At this stage the
risk of root fracture is minimal. Performing this procedure when less than 2/3 of root
development has been completed could result in the second molar floating in its new position.
Although the procedure has been performed when root development is complete, the
incidence of subsequent pulpal necrosis or calcification is increased.

Reference:
Dessner, S; Surgical uprighting of second molars: rationale and technique: Oral and
Maxillofacial Surg Clin N Am 14 (2002) 201-212.
Costello, BJ, Malik, AKB, Powers, MP, Fonseca, RJ; Complicated Exodontia in Fonseca, RJ,
Oral and Maxillofacial Surgery Vol 1, Anesthesia/Dentoalveolar Surgery/ Office
Management, WB Saunders 2000. p240.

The American Board of Oral and Maxillofacial Surgery 95


102. Which statement is correct regarding pericoronitis of a mandibular 3rd molar?

A. Initial treatment can be limited to lavage of material alba and debris


B. Pericoronitis occurs only in immunosuppressed patients
C. Antibiotics are mandatory
D. Extraction should be delayed until completion of a course of antibiotic therapy

Answer: A

Rationale:
Antibiotics are a key aspect in localizing an infection and limiting its spread to adjacent tissue
organs, areas, and spaces. Pericoronal infections that are localized to the immediate
enveloping tissues and give no evidence of spread to adjacent tissue planes require local
debridement and definitive treatment consisting of removal of the erupting tooth and/or
pericoronal tissues. Antibiotics are not mandatory, and tooth extraction need not to be
delayed to complete a course of antibiotics.

Reference:
Ness, GM, Peterson, LJ; Impacted Teeth in Peterson's Principles of Oral and Maxillofacial
Surgery, Miloro, M. et al., Editors, Second Edition, BC Decker Inc, 2004, pp140-142.

The American Board of Oral and Maxillofacial Surgery 96


103. Which of the following statements, regarding alveolar osteitis, is true?

A. Generally develops 7-10 days after surgery


B. Represents a localized bone infection
C. Is theorized to be caused by lysis of a fully formed blood clot prior to its replacement by
granulation tissue
D. Requires vigorous bone scraping under local anesthesia to stimulate new blood clot
formation

Answer: C

Rationale:
Alveolar osteitis is essentially an inflammation of the bony socket from a recently extracted
tooth. Treatment consists of debridement of the socket and placement of a suitable obtundent
until the area becomes asymptomatic. Usually no local anesthesia is required.

Reference:
Ness, GM, Peterson, LJ; Impacted Teeth in Peterson's Principles of Oral and Maxillofacial
Surgery, Miloro, M. et al., Editors, Second Edition, BC Decker Inc, 2004, p. 151.

The American Board of Oral and Maxillofacial Surgery 97


104. Indication for removal of impacted 3rd molars include:

A. prevention of mandibular anterior tooth crowding.


B. to allow for postoperative regeneration of bone distal to a 2nd molar with chronic
periodontitis.
C. prior to anticipated sagittal split osteotomy.
D. deeply impacted maxillary third molars, prior to anticipated LeFort I osteotomy.

Answer: C

Rationale:
While somewhat controversial, crowding of mandibular incisor teeth appears to be associated
with deficient arch length rather than the mere presence of impacted teeth. After 3rd molar
removal the bone height distal to the 2nd molar usually remains at the pre-operative level.
Removal of 3rd molars at mandibular advancement osteotomy reduces the thickness and
quality of lingual bone at the proximal aspect of the distal segment where fixation screws are
usually applied. During a LeFort osteotomy, removal of deeply impacted maxillary 3rd
molars from the maxillary sinus side may be done safely without compromising the soft tissue
vascular pedicle of the maxilla.

Reference:
Ness, GM, Peterson, LJ; Impacted Teeth in Peterson's Principles of Oral and Maxillofacial
Surgery, Miloro, M. et al., Editors, Second Edition, BC Decker Inc, 2004, pp. 140-153.

The American Board of Oral and Maxillofacial Surgery 98


105. Which of the following statements is correct regarding the mandibular third molar?

A. The tooth germ is usually visible on a radiograph by the age of 6.


B. By age 11, the tooth is located within the anterior border of the ramus with the occlusal
surface facing superiorly.
C. The orientation of the crown within the alveolus may be affected by underdevelopment
or overdevelopment of its mesial or distal roots.
D. The tooths position from ramus to alveolus is determined strictly by growth of the
mandibular body.

Answer: C

Rationale:
The mandibular 3rd molar, visible radiographically as a tooth germ usually by age 9 within
the anterior border of the ramus, progresses with mandibular growth to the body due to
resorption of the anterior border of the ramus. The tooth's anterior facing occlusal surface
changes orientation to vertical, usually by age 20, during its root development. The failure to
progress to final vertical orientation may be attributed to one or more factors including:
Over or under development of the mesial and/or distal roots
Insufficient arch length to accommodate the normal position of the tooth within the arch

Reference:
Ness, GM, Peterson, LJ; Impacted Teeth in Peterson's Principles of Oral and Maxillofacial
Surgery, Miloro, M. et al., Editors, Second Edition, BC Decker Inc, 2004, pp. 139-140

The American Board of Oral and Maxillofacial Surgery 99


106. Which of the following statements regarding adjunctive medications administered to an 18-year-
old healthy male during removal of 3rd molars, is true?

A. Prophylactic antibiotics are recommended to prevent post-operative infection.


B. Local measures are as effective as systemic antibiotics in the reduction of postoperative
dry sockets.
C. Perioperative steroids administered to reduce post-operative swelling have no
appreciable affect on dry socket incidence
D. Maximum control of swelling requires no additional steroid doses other than the initial
perioperative dose

Answer: B

Rationale:
The usage of prophylactic antibiotics involves the issue of risk versus benefits. Without
further extenuating conditions, the increase of antibiotic-related complications, e.g. allergy,
resistant bacteria, gastrointestinal side effects, and secondary infections, is not outweighed by
the benefits in an otherwise uncomplicated 3rd molar surgery. Perioperative steroids do have
a major clinical impact on swelling in the early postoperative period but, for maximum
control, additional steroids must be given for 1 or 2 days following surgery. The
administration of perioperative steroids may increase the incidence of alveolar osteitis after
third molar surgery, the data are lacking as to the precise degree of increase.

Reference:
Ness, GM, Peterson, LJ; Impacted Teeth in Peterson's Principles of Oral and Maxillofacial
Surgery, Miloro, M. et al., Editors, Second Edition, BC Decker Inc, 2004, pp 147-149.

The American Board of Oral and Maxillofacial Surgery 100


107. 7 days after third molar extraction, a patient presents with swelling and trismus. The large
contrast-enhancing fluid collection on the above scan is best described as residing in which of
the following spaces?

A. Submental
B. Masseteric
C. Sublingual
D. Pterygomandibular

Answer: D

Rationale:
This is an axial scan of a pterygomandibular space abscess.
The scan reveals a radiolucent, contrast-enhancing collection between the ascending ramus of
the mandible and distended medial pterygoid muscle. The lateral pharyngeal space and airway
are distorted by mass effect and edema. The borders of the pterygomandibular space are the
ascending ramus of the mandible laterally, the medial pterygoid muscle medially, the
pterygomasseteric sling inferiorly, and the lateral pterygoid muscle superiorly. The parotid
gland and capsule form the posterior border. The pterygomandibular raphe is the anterior
border.

REFERENCE: Flynn , TR In Topazian , RG, Goldberg, MH, Hupp, JR, editors: Oral and
Maxillofacial Infections, ed 4, Philadelphia, 2002, W B Saunders Co., pp 188-213.

The American Board of Oral and Maxillofacial Surgery 101


108. What stage of root development is optimal for the transplantation of a developing mandibular
third molar to a first molar extraction site?

A. Less than 1/2


B. Between 1/2 and 2/3
C. Between 2/3 and complete
D. Complete

Answer: B

Rationale:
The donor third molar roots should be no less than half and preferably about two thirds
developed.

Reference:
Silva, RG, Pogrel, MA; Tooth Reimplantation and Transplantation in Fonseca, RJ, Oral and
Maxillofacial Surgery Vol 1, Anesthesia/Dentoalveolar Surgery/ Office Management, WB
Saunders 2000. pp 299-301.
Kaban, LB, Dentoalveolar Surgery in Kaban Pediatric Oral and Maxillofacial Surgery, WB
Saunders, 1990, p109.

The American Board of Oral and Maxillofacial Surgery 102


109. On the above scan, what is the location of the third molar inadvertently displaced during an
attempted extraction?

A. Superficial temporal space


B. Maxillary sinus
C. Pterygomaxillary fissure
D. Infratemporal space

Answer: D

Rationale:
The tooth clearly is not in the maxillary sinus. Additionally is not in the pterygomaxillary
fissure between the pterygoid plate and posterior maxilla. The level of the scan is below the
superficial temporal space, the inferior boundary of which is the zygomatic arch. The tooth
resides in the infratemporal space. The infratemporal space is the portion of the deep temporal
space that lies inferior to the infratemporal crest of the sphenoid bone. The inferior border is
the superior surface of the lateral pterygoid muscle. The anterior border of the deep temporal
space is composed of the posterior wall of the maxillary sinus, the pterygomaxillary fissure,
and the posterior surface of the orbit, including the inferior orbital fissure.

Reference:
Flynn, TR In Topazian, RG, Goldberg, MH, Hupp, JR, editors: Oral and Maxillofacial
Infections, ed 4, Philadelphia, 2002, W B Saunders Co., pp 200-203.

The American Board of Oral and Maxillofacial Surgery 103


110. The diagnosis of mitral valve prolapse is associated with which of the following?

A. Early systolic murmur


B. Von Willebrands syndrome
C. Murmur accentuated by respiration
D. Patent ductus arteriosus (PDA)

Answer: B

Rationale:
Mitral valve prolapse is a common condition affecting 2%-5% of the population. It likely is
inherited as an autosomal dominant trait, and is more common in women. The murmur
associated with mitral valve prolapse occurs late in systole and is not affected by respiration.
MVP is associated with ventricular premature beats, paroxysmal atrial tachycardia, atrial
fibrillation, Von Willebrand's syndrome, polycystic kidney disease, atrial secundum defects
and Marfan's syndrome.

Reference:
LeBlond R, DeGowin R, Brown D:DeGowin's Diagnostic Examination 8th ed., McGraw-Hill,
New York, 2004, pp.428-429. Sdrales L, Miller R:Anesthesia Review., Churchill Livingstone,
Philadelphia, 2001, pp.268-269.

The American Board of Oral and Maxillofacial Surgery 104


111. Intravenous conscious sedation is being administered to a 62-year-old male with a history of
exertional angina. Local anesthetic without a vasoconstrictor has been infiltrated and extraction
of tooth #12 has been accomplished. Abruptly, the patients blood pressure rises 60% above
baseline. Conservative measures fail to reduce the blood pressure and pharmacologic
intervention is now required. Which antihypertensive agent is contraindicated in this setting?

A. Esmolol
B. Clonidine
C. Hydralazine
D. Labetalol

Answer: C

Rationale:
Sustained elevated blood pressure can lead to left ventricular failure, pulmonary edema,
myocardial ischemia, and cardiac dysrhythmias. Antihypertensive agents must be chosen
carefully when a patient with a history of ischemic heart disease is being treated. Labetalol is
a selective Alpha, and nonselective Beta-blocker. Reflex tachycardia is not seen after
administration and cardiac output is not changed. Pulmonary artery and wedge pressures
decrease. Esmolol is a rapid acting cardioselective beta blocker which does not increase
cardiac output or cardiac oxygen consumption. Oral clonidine is a centrally acting alpha
agonist with a wide margin of safety. Hydralazine is a parenteral antihypertensive that acts as
a direct vasodilator., and may result in unopposed beta stimulation with significant reflex
increases in cardiac output. Hydralazine is contraindicated when a patient has a history of
ischemic heart disease.

Reference:
Crawford M:Current Diagnosis & Treatment in Cardiology., Lange Medical Books/McGraw-
Hill, New York, 2003, pp.176-178. Ogle O, ed:Oral and Maxillofacial Surgery Clinics of
North America., W.B. Saunders, Philadelphia, 1998, Vol 10, No.3, pp.349-361.

The American Board of Oral and Maxillofacial Surgery 105


112. A thin appearing 52-year-old male with a long history of alcohol abuse is having multiple dental
extractions under local anesthesia when he suddenly becomes disoriented and confused. An ECG
reveals a polymorphic ventricular tachycardia. His vital signs are as follows: BP 85/55, HR 165,
RR 22. Which is the most appropriate drug to administer?

A. Epinephrine
B. Amiodarone
C. Lidocaine
D. Magnesium

Answer: D

Rationale:
Chronic alcoholism and malnutrition is frequently associated with hypomagnesemia which
can provoke a polymorphic ventricular tachycardia, torsades de pointe, with a prolonged QT
interval. The immediate treatment is to administer magnesium. Epinephrine may increase
myocardial irritability and worsen the tachycarida. Amiodarone prolongs the QT interval
which can cause a worsening of the torsades. Lidocaine may be given but is not as efficacious
as magnesium for this type of arrhythmia. If available, overdrive pacing is an acceptable
alternative to magnesium.

Reference:
Cummins:Case 3. ACLS Provider Manual:75-95, 2001

The American Board of Oral and Maxillofacial Surgery 106


113. A 20-year-old female presents for routine third molar surgery. She appears emaciated, has
indicated that she has an eating disorder and you confirm that she has anorexia nervosa. Which
one of the following pre-operative tests is most indicated prior to surgery?

A. AST, GGT, ALT, and alkaline phosphotase


B. 24 hour urine for metanephrines
C. serum electrolytes
D. Coagulation tests (PT, PTT, fibrin degradation products)

Answer: C

Rationale:
Dysfunction is found in most organ systems in the severely malnourished patient, but the
most dangerous are fluid/electrolyte disorders. Amongst the answers given, a determination of
electrolyte status would be the most appropriate in this situation. Other pertinent pre-op tests
would include an electrocardiogram and complete blood count. Metabolic alkalosis,
hypocalemia, and hypokalemia may manifest with prolongation of the QT interval. Sudden
death in these individuals is often precipitated by ventricular arrhythmias. AST, GGT, ALT
and alkaline phosphatase are blood screening tests for hepatic function. Although anorexia
may affect the hepatobiliary system, cardiac function associated with anorexia would have the
greatest potential impact in safely managing this patient. A 24 hour urine measuring the
levels of metanephrines would be an appropriate study in the differential diagnostic work up
for an adrenal neoplasm (such as a pheochromocytoma). This study will not yield any data
that would assist in the diagnosis or management of anorexia nervosa. Although Prothrombin
time (PT) and Partial Thromboplastin Time may be prolonged secondary to liver disease,
these are not usually associated with anorexia. Fibrin degradation products are associated with
disseminated intravascular coagulopathy (DIC), not with anorexia.

Reference:
The Merck Manual, 17th Ed., Merck and Co., 1999, pp.1595-6.
2002 Ferri's Clinical Advisor, Fred Ferri, Mosby 2002 Edition, pp. 66-7.

The American Board of Oral and Maxillofacial Surgery 107


114. During cardiac exercise stress testing, ECG monitoring during or after exercise would look for
signs of ischemia based on:

A. excessively tall QRS complexes.


B. prolonged P-R intervals.
C. biphasic P-waves.
D. ST depression.

Answer: D

Rationale:
Patients with stress-induced symptoms of ischemia usually provide the best assessment of risk
for adverse events from CAD. Ischemic tendencies would be evidenced by the presence of ST
depression or inverted T waves. The addition of cardiac imaging increases the sensitivity of
the stress test, especially when resting ECG abnormalities exist or conditions are present that
may cause abnormal stress-induced ECG changes not secondary to ischemia. Excessively tall
QRS complexes may indicate ventricular hypertrophy. Prolonged P-R intervals indicate
conduction delay at a location between the atria and the ventricles (such as at the A-V node.)
Biphasic P waves are indicative of atrial hypertrophy.

Reference:
OMS Knowledge Update; Vol. I, Part II, 1995, p PEV21. Peterson LJ, Indresano AT,
Marciani RD, Roser SM, Principles of Oral and Maxillofacial Surgery; 1992, Chapter 2, p 20.

The American Board of Oral and Maxillofacial Surgery 108


115. An obese 60-year-old male presents for extraction of tooth #32. His past medical history is
significant for angina, syncopal episodes and congestive heart failure. A crescendo-decrescendo
systolic murmur is noted upon auscultation of the precordium. The most likely diagnosis is:

A. aortic regurgitation.
B. mitral stenosis.
C. aortic stenosis.
D. mitral regurgitation.

Answer: C

Rationale:
Aortic stenosis is the most common congenital cardiac abnormality. Patients typically present
with angina, syncope and congestive heart failure. The classic murmur of aortic stenosis is a
harsh systolic ejection murmur. The only effective treatment is valve replacement. Aortic
regurgitation is associated with a high pitched diastolic decrescendo murmur. Mitral stenosis
auscultation yields a diastolic rumble, loud S1, and an opening snap. Mitral regurgitation
creates a holosystolic murmur with radiation to the axilla.

Reference:
Crawford M:Current Diagnosis & Treatment in Cardiology., Lange Medical Books/McGraw-
Hill, New York, 2003, pp.108-117. Andreoli T:Cecil Essentials of Medicine., W.B. Saunders,
Philadelphia, 2004, pp.78-80.

The American Board of Oral and Maxillofacial Surgery 109


116. Tricuspid valve insufficiency initially produces:

A. atrial fibrillation.
B. left Ventricular dilitation.
C. volume overload of right atrium/ventricle.
D. ventricular Tachycardia.

Answer: C

Rationale:
Insufficiency of the tricuspid valve initially produces a volume overload to the right atrium
and ventricle, which dilate as a compensatory mechanism. This gives rise to elevated central
venous pressures which cause right sided congestive failure. If the tricuspid regurgitation is
massive, under-filling of the left heart can result.(this last sentence need a little work)

Reference:
Luce J, Pierson D (Eds.):Critical Care Medicine, 1988,WB Saunders, p. 47.

The American Board of Oral and Maxillofacial Surgery 110


117. Idiopathic Hypertrophic Subaortic Stenosis (IHSS):

A. obstructs right ventricular outflow.


B. diminishes diastolic valvular function.
C. affects left ventricular outflow.
D. hypertrophies the tricuspid valve.

Answer: C

Rationale:
IHSS is usually an inherited autosomal dominant characteristic, and can be associated with
long term hypertension. Depending on the area of hypertrophy, the systolic left ventricular
outflow can be significantly obstructed during systole. Fatal dysrhythmias and sudden death
may result even in healthy teenagers with undiagnosed hypertrophy.

Reference:
Peterson's Principles of Oral and Maxillofacial Surgery, Second Edition, Vol I pg 69

The American Board of Oral and Maxillofacial Surgery 111


118. Which of the following statements concerning intravascular replacement is true?

A. 0.9% saline is isosmolar to lactated Ringers solution


B. Renal resorption of potassium makes intravenous supplementation unnecessary
C. Excessive lactated Ringers solution administration can cause hypermagnesemia
D. One liter of 5% dextrose in water yields 200 kilocalories

Answer: D

Rationale:
One liter of 5% dextrose in water yields 50 g of dextrose; with available calories:4 kcal/gm, or
200 kcal. O.9% (normal) saline has an osmolality of 308; while lactated Ringer's solution is
273. Obligate renal potassium losses are about 40 mEq/day, and requires replacement (either
from oral intake or by a parenteral supplement.) Ringer's lactate has no magnesium hence no
possibility of hypermagnesemia.

Reference:
Washington University Manual of Medical Therapeutics (30th ed.), pp43-45

The American Board of Oral and Maxillofacial Surgery 112


119. Woff-Parkinson-White (WPW) Syndrome:

A. is treated with verapamil IV.


B. requires extra-nodal conduction.
C. is characterized by severe bradycardia.
D. is unresponsive to radiofrequency ablation.

Answer: B

Rationale:
During normal embryonic cardiogenesis, strands of electroconductive tissue form
connections between the atria and ventricles outside of the normal conduction pathway. These
strands normally become non-functional shortly after birth. In WPW, persisant connections
via extranodal pathways bypass part of all of the normal conduction system and function in a
re-entry capacity. In WPW the accessory pathway is called the Kent Bundle and connects the
atria directly to the ventricles and is a cause of atrial fibrillation. This can result in a
ventricular rate of 250-300 beats/min. Treatment of WPW, especially if atrial fibrillation is
present, with IV verapamil may decrease the accessory pathway refractory period, increasing
the ventricular rate and potentiating ventricular fibrillation. Radiofrequency ablation has
become one of the procedures of choice in ablating the accessory conductive pathways in
WPW syndrome.

Reference:
ACLS Quick Review Study Guide, Second Ed. Barbara Aehlert, RN pg 180 Mosby.

The American Board of Oral and Maxillofacial Surgery 113


120. Which of the following concerning acute chest pain is true?

A. ST depression and chest pain with positive cardiac troponins are an indication for
thrombolytic therapy
B. Q-wave MIs contraindicate percutaneous coronary angioplasty
C. Unstable angina is an indication for antiplatelet medication
D. Heparin may not be used with fibrinolytic therapy

Answer: C

Rationale:
Unstable angina is a pre-infarctive condition, in which the developing thrombus is platelet
rich and where antiplatelet drugs are indicated. ST depression with lack of Q waves and
positive cardiac troponin indicate an early or intermittent infarction (termed a non-Q-wave
myocardial infarction.) The thrombus at this stage is a mixture of a fibrin clot and platelets;
and treatment at this time with a thrombolytic may liberate clot-bound thrombin, enlarging or
embolizing the clot and may paradoxically worsen coronary occlusion. Q wave MIs indicate
total thrombotic occlusion and are an indication for prompt coronary angioplasty. Heparin is
commonly used with both fibrin specific thrombolytics and with angioplasty procedures.

Reference:
ACLS Provider Manual (2001), pp123-127

The American Board of Oral and Maxillofacial Surgery 114


121. Which of the following is most likely to be associated with untreated mitral stenosis?

A. Increased pulse pressure


B. Right atrial hypertrophy
C. Pulmonary hypertension
D. Left ventricular hypertrophy

Answer: C

Rationale:
Mitral stenosis is associated with diminished pulse pressure as there may be impaired left
ventricular filling. A triad of symptoms associated with MS includes left atrial hypertrophy,
pulmonary hypertension and right ventricular hypertrophy due to stenosis of the mitral valve.
This condition requires the cardiac structures upstream from the mitral valve to overcome
increased resistance across a stenotic mitral valve resulting in hypertrophic changes in those
areas. Note: pulmonary hypertension is the result of smooth muscle hypertrophy involving the
pulmonary arteriolar bed.

Reference:
McCabe, Roser: Evaluation and management of the cardiac patient for surgery. Oral Max
Surg Clin of N Amer 10:3, 1998. Ramadan, et al: Preoperative evaluation of cardiac risk. Adv
Surg 28: 1995

The American Board of Oral and Maxillofacial Surgery 115


122. A 40-year-old male comes to your office as a new patient upon referral from his family physician
for recurrent pericoronitis of a lower third molar. He is currently asymptomatic, and desires a
deep sedation for third molar removal. His physical examination reveals an early systolic click
and a 2/6 murmur in the aortic area. In the referral letter, his physician notes the following test
results: ECG is normal; echocardiogram shows a bicuspid aortic valve without significant flow
obstruction. His ventricle size and function are normal. What intervention would be warranted
prior to proceeding with an office anesthetic and surgery for this patient?

A. Antibiotic endocarditis prophylaxis


B. Aortic valve replacement
C. Balloon valvuloplasty
D. No intervention

Answer: A

Rationale:
All patients with bicuspid aortic valves --- even those with no significant stenosis or
regurgitation --- should be given antibiotic prophylaxis for bacterial endocarditis for surgeries
involving the aerodigestive tract. As much as 2% of the population has congenitally bicuspid
aortic valves. A bicuspid aortic valve may present as an incidental finding on physical
examination or echocardiography done for other reasons. On physical examination, the
cardinal sign of a bicuspid aortic valve is an early systolic ejection click. If no significant
hemodynamic abnormality is present, either no murmur or a soft ejection murmur may be
heard; a very mild murmur of aortic regurgitation (AR) is not uncommon, even with
hemodynamically insignificant bicuspid aortic valves. Aortic stenosis (AS) or AR from any
other cause will produce similar findings. Both the presence of a bicuspid aortic valve and its
hemodynamic significance can be determined by echocardiography. Serial studies are useful
in following the progression of the lesion.. Patients with AR from a bicuspid valve who are
asymptomatic and have normal systolic function are followed with echocardiograms and
physical examinations at regular intervals. If they begin to show decreasing systolic function,
symptoms of heart failure, or progressive dilation of the left ventricle, surgical replacement of
the aortic valve is indicated.

Reference:
Harrison's Principles of Internal Medicine 14th ed, p. 1320

The American Board of Oral and Maxillofacial Surgery 116


123. Which of the following may be observed in pulmonary sarcoidosis?

A. Distortion or obstruction of the airway


B. Obstructive lung disease
C. Pneumothorax in early stages
D. Sparing of parenchyma

Answer: A

Rationale:
Sarcoidosis is a multisystem granulomatous disease. Respiratory manifestations include
laryngeal granulomas or nodules as well as intrathoracic nodes that may compress or distort
the airway. Involvement of the recurrent laryngeal nerve can cause unilateral vocal cord
paralysis. Obstruction of the lower airways can result in atelectasis, but local obstruction is
not the same as obstructive lung diseases. Sarcoidosis is a restrictive lung disease, with the
majority of pulmonary damage occurring in the parenchyma. Patients may be prescribed
steroids and perioperative care must ensure appropriate steroid coverage. Pulmonary fibrosis
seen in advanced stages of the disease and may result in a pneumothorax. Patients generally
take rapid shallow breaths to compensate for the physiologic changes. Hypoxemia is common
at rest in severe disease.

Reference:
Kinney MAO, Harrison BA, Ruyter MLD: Sarcoidosis in Complications in Anesthesia Eds
Altee JL. WB Saunders 1999

The American Board of Oral and Maxillofacial Surgery 117


124. Which laboratory study is the most informative in the diagnosis of asthma?

A. Plane chest radiography


B. Spirometry
C. Arterial blood gas determination
D. Stethoscopy

Answer: B

Rationale:
Asthma diagnosis rests upon the measurement of peak expiratory flow rates (spirometry)
before and after administration of an inhaled beta adrenergic agonist; the reversibility of
lowered expiratory flows being diagnostic for this disorder. Alterations in plane chest
radiographs, arterial blood gases, and in auscultation of the chest by stethoscope may occur
with asthma, but changes in these are not specific for the diagnosis of asthma.

Reference:
Little JW, Falace DA et al: Dental Management of the Medically Compromised Patient 6th
ed. . St. Louis, Mosby, 2002 p. 132. Harrison's Principles of Internal Medicine 14th ed. New
York, McGraw-Hill, 1997 p. 1423

The American Board of Oral and Maxillofacial Surgery 118


125. A 28-year-old patient suffers a prolonged laryngospasm in your office. Upon breaking the
spasm through the use of positive pressure oxygen and succinylcholine, the patient is still
somewhat somnolent and difficult to arouse. His respiratory rate is 16/min and there are crackles
heard throughout his lung fields, especially near the bases. At this point the patient will likely
require:

A. bronchial alkalinization.
B. positive end expiratory ventilation.
C. pleurocentesis.
D. antibiotics.

Answer: B

Rationale:
Patients who have had a laryngospasm may be susceptible to secondary pulmonary edema.
This can be the result of negative alveolar pressure of an expanding diaphragm against a
closed glottis, or due to barotrauma from positive pressure ventilation. Pulmonary trauma
arising from laryngospasm may become clinically evident hours after the spasm. In this case,
signs arising immediately after the treated laryngospasm may indicate imminent pulmonary
failure which may require endotracheal intubation and airway support using positive end
expiratory pressure, and possibly diuretics. While this patient may have had an aspiration, the
management of this complication would not be alkalinization of the airways. It is unlikely
that the acute onset of dyspnea in this case was due to pleural effusion or pneumonia

Reference:
Brandom BW. Pulmonary Edema After Airway Obstruction. International Anesthesiology
Clinics 35(3):75-84, 1997. Halow KD, Ford EG. Pulmonary Edema following Post-operative
Laryngospasm:A Case Report and Review of the Literature. American Surgeon 59(7):=443-
447, 1993.

The American Board of Oral and Maxillofacial Surgery 119


126. Which is a characteristic feature of pulmonary emphysema?

A. Increased hematocrit
B. Cor pulmonale
C. Decreased functional residual capacity
D. PaCO2 is normal to slightly decreased

Answer: D

Rationale:
Chronic obstructive lung disease encompasses chronic obstructive bronchitis and emphysema.
With both disorders, FEV/ FVC is decreased; but residual volume, functional residual
capacity and tidal volume are generally increased. Chronic obstructive bronchitis is referred to
as blue bloater. The disease is characterized by obstruction of small airways secondary to
mucus and inflammation. Patients with chronic bronchitis have significant decreases in PaO2
(< 65 mmHg is typical.) Compensatory consequences include erythrocytosis. PaCO2 is also
chronically elevated. This results in pulmonary hypertension and cor pulmonale. Physiologic
changes found with emphysema include destruction of lung parenchyma, enlargement of
airway spaces, loss of lung elasticity and closure of small airways. Rapid, shallow respirations
generally result in a PaCO2 which is normal to slightly decreased and a PaO2 > 65 mmHg.
This disease is categorized as pink puffer and is generally not associated with cor
pulmonale.

Reference:
Stoelting RK & Dierdorff SF: Handbook for Anesthesia and Co-Existing Disease. 2nd
edition Churchill Livingston 2002

The American Board of Oral and Maxillofacial Surgery 120


127. Which of the following statements concerning pulmonary thromboembolism is true?

A. ELISA test for d-dimer yields the quickest diagnostic results


B. Helical CT is highly specific indicator for subsegmental emboli
C. Fluid replacement is indicated for decreased left ventricular output
D. Serial deep venous ultrasonography is indicated if other tests yield equivocal results

Answer: D

Rationale:
The diagnosis of pulmonary thromboembolism can be difficult, and the results of clinical,
laboratory, and radiologic exams can be equivocal. Testing is generally based upon risk
stratification (according to clinical presentation, medical history, surgical procedures, and
other risk factors both for developing PE and for risks of this disorder.) The most common
suspected site of origin for pulmonary thrombi are from deep leg veins; so in the case of
suspected but unconfirmed PE, repeat serial leg vein ultrasound is indicated. ELISA tests
vary in sensitivity the d-dimer breakdown products from thrombi; with the results generally
not immediately available. Venous contrast helical CT is being advocated as a screening tool
for PE, but this modality has not proven to be highly sensitive or specific for (small)
subsegmental PE's, which are notoriously difficult to diagnose. Decreased in left ventricular
output are an ominous sign in PE, caused by decreased left atrial filling. Fluid bolus therapy
in these patients, who already are in right heart failure, is contraindicated.

Reference:
Kearon C: Diagnosis of pulmonary embolism. J. Can. Med Assoc, 168:2, Jan 2003
Goldhaber S: Pulmonary thromboembolism. In: Fauci et al (eds;) Harrison's Principles of
Internal Medicine 14th ed. p. 1468

The American Board of Oral and Maxillofacial Surgery 121


128. Montelukast has its primary mechanism in reducing inflammation associated with asthma
through:

A. inhibition of prostaglandins.
B. antagonism of leukotrienes.
C. direct beta-2 stimulation.
D. blocking production of arachidonic acid.

Answer: B

Rationale:
Leukotrienes are potent biochemical mediators that are released from mast cells, eosinophils,
and basophils. Leukotrienes work to contract airway smooth muscle, increase vascular
permeability and mucus secretions, and attract and activate inflammatory cells in the airways
of patients with asthma. They are produced by the oxidation of arachidonic acid through the
enzyme lipoxygenase. The action of leukotrienes can be blocked through either of two
specific mechanisms: 1) inhibition of leukotriene production and, 2) antagonism of
leukotriene binding to cellular receptors. The FDA has approved three products thus far.
Zafirlukast and montelukast are both selective and competitive leukotriene receptor
antagonists of leukotriene D and E. These are components of slow-reacting substance of
anaphylaxis. Zileuton, a specific inhibitor of 5-lipooxygenase, inhibits leukotriene formation,
especially LTB1, LTC1, LTD1, LTE1. As a leukotriene antagonist, SingulairR (montelukast)
has no effect on lipoxygenase or arachidonic acid function. Arachidonic acid is produced by
membrane phospholipids through the enzyme phospholipase A2 which is blocked by
corticosteroids. Cyclooxygenase inhibitors (i.e. NSAIDs, COX inhibitors) block the
production of prostaglandins. Beta-2 stimulation results in bronchodilation and is the main
mechanism of medications such as albuterol and not the leukotriene inhibitors.

Reference:
Kastrup EK, et al. (eds) Respiratory drugs. In Drug Facts and Comparisons, 1998. St. Louis,
Facts and Comparisons. Pp 1133-1140. Aharony D. Pharmacology of leukotriene receptor
antagonists. Am J Respir Crit Care Med 1998;157:S214-9.

The American Board of Oral and Maxillofacial Surgery 122


129. Which of the following decreases pulmonary diffusion capacity?

A. Acute respiratory distress syndrome


B. Diaphragmatic paralysis
C. Chylothorax
D. Pleural effusion

Answer: A

Rationale:
The diffusion capacity of the lungs for any gas is dependent upon the diffusion coefficient (for
the gas in question), the surface area and thickness of the alveolar-capillary membrane, and
the partial pressure gradient across the barrier. Changes in any of these variables will affect
the diffusing capacity. The alveolar capillary membrane is made up of various layers. These
layers traveling from the alveolus toward the capillary are: surfactant layer, alveolar
epithelium, interstitial space, pulmonary capillary endothelium, plasma, and red blood cell.
On average, this barrier is about 0.5 m in thickness. Alveolar edema or exudates such as in
acute respiratory distress syndrome increases the thickness of the blood-gas barrier in the
lung, which increases the time necessary for equilibration of CO2 and O2 across the blood-
gas barrier. Diaphragmatic paralysis will inhibit air exchange but not the ability to equilibrate
gaseous exchange across the alveolar/capillary surface. A pleural effusion and chylothorax
are is present in the pleural space and therefore will not directly affect the diffusion of gases
in the alveoli of the lung.

Reference:
Vander AJ, Sherman JH, Luciano DS: Respiration In Human Physiology: The Mechanisms of
Body Function, , 4th edition, 1985. Pocock, G. & Richards, C.D. Human Physiology: the
Basis of Medicine, Oxford Core Texts, 1st Edition, 1999

The American Board of Oral and Maxillofacial Surgery 123


130. Which of the following can be used to classify an airway in the Fujita system?

A. Epworth Sleep Scale


B. Polysomnography
C. Bed partner interview
D. Nasopharyngoscopy

Answer: D

Rationale:
Nasopharyngoscopy , especially supine and using Mueller's Maneuver (holding the nose and
mouth closed against forced inspiration) is the most useful to diagnose the level of airway
obstruction in the sleep apnea patient. A Fujita Type II airway obstruction occurs at both the
retropalatal (oropharyngeal) and retrolingual (hypopharyngeal) areas; while a Type I is
retropalatal and Type III is retrolingual. Polysomnography establishes the diagnosis of
obstructive sleep apnea but not the location of obstruction. The Epworth Sleep Scale is a
subjective evaluation of daytime hypersomnolence and is useful in the referral of patients for
further sleep disorder work up. The interview of a bed partner, especially as to sonorous
breathing, loud snoring, or observed obstructive apneas is highly sensitive for the presence of
obstructive sleep apnea and should prompt the practitioner to further sleep disorder work up
including polysomnography.

Reference:
Bond T: Evaluation and diagnosis of sleep-disordered breathing. OMS Clinics N
America14:3, 293-296 Aug 2002

The American Board of Oral and Maxillofacial Surgery 124


131. Which of the drugs listed below is appropriate for use in the epileptic patient with asthma?

A. Piroxicam
B. Methohexital
C. Morphine
D. Ketamine

Answer: D

Rationale:
The sources of asthmatic airway processes are rapid release of chemical mediators,
abnormalities of airway neuroregulation, or an intrinsic defect of airway smooth muscle. Ten
percent of patients with asthma can experience bronchial constriction and rhinorrhea in
response to aspirin or NSAIDs such as prioxicam. The mechanism is via blockade of
prostaglandin synthesis, causing leukotriene build up. Methohexital may be epileptogenic
(unlike other barbiturates); and histamine release by methohexital or morphine may also
precipitate an asthma attack. Ketamine acts as a bronchidilator and does not possess
epileptogenic properties.

Reference:
Little J, Falace D: Dental Management of the Medically Compromised Patient 6th ed.,
Mosby, St. Louis 2002, P 136. Malamed S: Sedation: A Guide to Patient Management 4th ed,
Mosby, St. Louis, 2003, pp 560-561. Ruggiero S: Evaluation, Treatment, and Management of
the Asthmatic Patient, Oral and Maxillofacial Surgery Clinics of North America, WB
Saunders, Philadelphia, 10:3 Aug 1998, pp339-343

The American Board of Oral and Maxillofacial Surgery 125


132. An early diagnostic test used to test infants suspected of having cystic fibrosis is:

A. urine sodium.
B. sweat chloride.
C. fecal aluminum.
D. sputum alcohol level.

Answer: B

Rationale:
Cystic fibrosis is an autosomal recessive inherited disease of the exocrine glands, primarily
affecting the GI and respiratory systems, and usually characterized by COPD, exocrine
pancreatic insufficiency, and abnormally high sweat electrolytes (particularly chloride > 60
mEq/L). In addition to thickened sputum and pancreatic enzyme dysfunction, males often
have azospermia secondary to obstruction of the vas deferens by thickened secretions.

Reference:
Facts About Cystic Fibrosis, Cystic Fibrosis Foundation, Bethesda, MD, November 16, 2001.
Farrell, P.M., et al. Early diagnosis of cystic fibrosis through neonatal screening prevents
severe malnutrition and improves long-term growth. Pediatrics, volume 107, number 1,
January 2001, pages 1-13.

The American Board of Oral and Maxillofacial Surgery 126


133. Which antibiotic has the greatest potential for adverse interaction with theophylline?

A. Penicillin
B. Clindamycin
C. Cephalexin
D. Ciprofloxacin

Answer: D

Rationale:
Theophylline, a thioxanthine, acts centrally as an indirect adrenergic, causing dilation of
bronchial smooth muscle and is used in the management of refractory asthma. Toxicity is
manifested as hyperadrenergia including tachycardia, hypertension, and hyperreflexia.
Erythromycin reversibly inhibits hepatic degradation of theophylline and can lead to toxic
serum levels of the bronchodilator. The same is true with concomitant use of theophylline
with cimetidine, ciprofloxacin, and allopurinol. Clindamycin, cephalexin, and penicillin do
not diminish theophylline metabolism.

Reference:
Abubaker AO, Benson KJ: Oral and Maxillofacial Surgery Secrets, Philadelphia, Hanley &
Belfus, 2001 P. 122

The American Board of Oral and Maxillofacial Surgery 127


134. In a healthy individual, the major factor in regulation of alveolar ventilation is arterial:

A. PaO2
B. PaCO2
C. pH
D. HCO3

Answer: B

Rationale:
A number of factors play a role in controlling alveolar ventilation. The central areas of
inspiratory and expiratory control lie in the medulla, and primarily respond to increases in
hydrogen ion concentration in the cerebrospinal fluid. Although hydrogen ion concentration
is the most important stimulus to the chemosensitive centers in the medulla, these ions cross
from blood through the blood-brain barrier to cerebrospinal fluid with difficulty. In contrast,
CO2 readily crosses the blood-brain barrier to react with water to form carbonic acid in the
CSF, which dissociates to provide the required hydrogen ion necessary for stimulation.
Peripheral chemoreceptors transmit signals via cranial nerves ten (to the aortic bodies) and
nine (to the carotid bodies) to the inspiratory area of the medulla and pons. These signals help
control breathing frequency and lung inflation. The peripheral receptors respond to drops in
arterial oxygen pressure.

Generally speaking, carbon dioxide pressure plays a much greater influence on ventilation
than oxygen pressure. To exemplify the role that the PaCO2 plays in controlling alveolar
ventilation, a 50% increase in arterial PaCO2 produces a tenfold increase in alveolar
ventilation, while a 40mmHg decrease in arterial PaO yields only a 1.5 fold increase in
alveolar ventilation. The high blood flow through the richly vascularized peripheral
chemoreceptors allows the needs of the receptors to be almost entirely met by dissolved
oxygen, and excess oxygen bound as oxyhemoglobin is not used by the receptors. As a
result, the PaO2 determines the level of peripheral receptor stimulation rather than the arterial
oxygen saturation (SaO2).

Reference:
Stoelting R: Pharmacology and Physiology in Anesthetic Practice 3rd ed., Lippincott,
Williams, and Wilkins, New York, 2001, pp 726-728. Morgan G, Maged S, Clinical
Anesthesiology 2nd ed., Lange Medical Books, New York, 1996, pp436-437.

The American Board of Oral and Maxillofacial Surgery 128


135. A 24-year-old male motorcyclist is involved in a head on collision with a stationary vehicle. His
GCS on arrival is 8. As you are completing the secondary survey his blood pressure acutely rises
to 220/140. A ventriculostomy is placed and indicates his intracranial pressure is 40mmhg. What
is his cerebral perfusion pressure?

A. 20mmHg
B. 70mmHg
C. 130mmHg
D. 170mmHg

Answer: C

Rationale:
Cerebral perfusion pressure is calculated by the following formulae:

Mean arterial pressure: diastolic blood BP + 1/3 (systolic BP diastolic BP)


140 + 1/3 (220-140)
167

Cerebral perfusion pressure (CPP): Mean arterial pressure-Intracranial pressure (ICP)


167 40
127 (about 130)

Cerebral blood flow is constant when CPP is maintained between 50-150mmHg. To maintain
cerebral blood flow, the MAP should be between 60-160mmHg and the ICP between 5-
15mmHg. As the ICP begins to rise, cerebral autoregulation occurs increasing cerebral
vascular resistance causing cerebral hypertension in an attempt to maintain cerebral blood
flow.

Reference:
Cushing H. The blood pressure reaction of acute cerebral compression, illustrated by cases of
intracranial hemorrhage. Am J Med Sci:125, 1017-1044, 1903

The American Board of Oral and Maxillofacial Surgery 129


136. A 70 kg, 35-year-old female is undergoing removal of a submandibular gland for sialolithiais
under intubation general anesthesia. She was induced with 100mg propofol, and was given
vecuronium as the paralytic agent. Isoflurane was used for maintenance of general anaesthesia.
The case took approximately 60 minutes. At the end of the procedure she continued to have
significant neuromuscular blockade. She was awake, able to open her eyes able and follow
commands with difficulty. Her neurologic exam was nonfocal. She exhibits the following:
respiratory rate 16/ min, tidal volume 100cc. end tidal CO2 55mmHg. What is the first
diagnostic test that would be appropriate?

A. Edrophonium (Tensilon) Test


B. Head CT
C. Electromyography
D. Pyridostigmine Test

Answer: A

Rationale:
Myasthenia gravis is an acquired autoimmune disorder affecting transmission at
neuromuscular junction. It is an autosomal dominant condition. 80-90% of patients will have
auto antibodies to the acetylcholine receptors of the post synaptic membrane. Patients exhibit
prolonged neuromuscular blockade to non-depolarizing muscle relaxants; but usually show
some resistance to the muscle paralysis of succinylcholine.
When MG is suspected the first test that may be performed is the Tensilon test using
edrophonium 2mg IV wait 30 seconds for effect, assess for improvement in muscle strength
that lasts for 5minutes. Repeated doses of 8mg may be given. If this test is positive supportive
care is required until the neuromuscular blockade improves. In a myesthenic crisis
plasmapheresis may be indicated for temporary relief, but not as first line measure.
Head CT would not be the first line diagnostic test in an awake patient who has a nonfocal
neurologic examination. Electromyography and muscle biopsy can be used as definite test for
MG. Pyridostigmine is used for treatment of MG, and not usually for diagnosis due to its
slower onset and longer duration of action.

Reference:
Sanders DB. Myasthenia Gravis and Myasthenic syndrome. Philadelphia, 1994. Drachman
DB. Myasthenia gravis; medical progress. N Eng J Medicine 330, 1797, 1994

The American Board of Oral and Maxillofacial Surgery 130


137. Which of the following is characteristic of absence seizures (petit mal)?

A. Sudden cessation of ongoing conscious activity and loss of postural control without
convulsive activity
B. Pathognomonic electroencephalogram showing 3 Hz spike and wave discharges
C. Difficulty of control with anti-epleptogenic medication
D. Initial presentation generally in middle age ENDTEXT

Answer: B

Rationale:
Absence seizures have cessation of conscious activity without any convulsions or loss of
postural control. These tend to be very brief, lasting seconds to sometimes a few minutes, and
are usually diagnosed in young children. The EEG is pathognomonic and often shows that
the child is having many more seizures than was evident. Antiepileptic drugs usually are
effective for treatment.

Reference:
Wilson, JD ed. Harrison's Principles of Internal Medicine. McGraw-Hill, Inc. 1991. p. 1969.

The American Board of Oral and Maxillofacial Surgery 131


138. Which of the following concerning Bells Palsy is true?

A. The transitory facial muscle paresis is usually due to a CNS vasospastic phenomenon
B. This disorder has a peak incidence in men in the sixth decade of life
C. Treatment for Bell's palsy include an antiviral combined with a glucocorticoid
D. Diagnosis of Bell's palsy is made based on increased creatine phosphokinase (CPK)
greater than 145mU/ml. secondary to low level rhabdomyolysis

Answer: C

Rationale:
Because the facial nerve has so many functions and is so complex, damage to the nerve or a
disruption in its function can lead to many problems. Symptoms of Bell's palsy, which vary
from person to person and range in severity from mild weakness to total paralysis, may
include twitching, weakness, or paralysis on one or both sides of the face, drooping of the
eyelid and corner of the mouth, drooling, dryness of the eye or mouth, impairment of taste,
and excessive tearing in one eye. These symptoms usually begin suddenly and reach their
peak within 48 hours. Other symptoms may include pain or discomfort around the jaw and
behind the ear, ringing in one or both ears, headache, loss of taste, hypersensitivity to sound
on the affected side, impaired speech, dizziness, and difficulty eating or drinking. Palsy
occurs when the nerve that controls the facial muscles is swollen, inflamed, or compressed,
resulting in facial weakness or paralysis. Exactly what causes this damage, however, is
unknown. Most scientists believe that a viral infection such as viral meningitis or the common
cold sore virus - herpes simplex - causes the disorder.
Bell's palsy afflicts approximately 40,000 Americans each year. It affects men and women
equally and can occur at any age, but it is less common before age 15 or after age 60. It
disproportionately attacks pregnant women and people who have diabetes or upper respiratory
ailments such as the flu or a cold. There is no cure or standard course of treatment for Bell's
palsy. The most important factor in treatment is to eliminate the source of the nerve damage.
Bell's palsy affects each individual differently. Some cases are mild and do not require
treatment as the symptoms usually subside on their own within 2 weeks. For others, treatment
may include medications and other therapeutic options. Recent studies have shown that
steroids are an effective treatment for Bell's palsy and that an antiviral drug such as acyclovir-
used to fight viral infections-combined with an anti-inflammatory drug such as the steroid
prednisone-used to reduce inflammation and swelling-may be effective in improving facial
function by limiting or reducing damage to the nerve. Analgesics such as aspirin,
acetaminophen, or ibuprofen may relieve pain. Another important factor in treatment is eye
protection. Bell's palsy can interrupt the eyelid's natural blinking ability, leaving the eye
exposed to irritation and drying. Therefore, keeping the eye moist and protecting the eye from
debris and injury, especially at night, is important. Lubricating eye drops, such as artificial
tears or eye ointments or gels, and eye patches are also effective. Facial massage and
exercises may help prevent permanent contractures (shrinkage or shortening of muscles) of
the paralyzed muscles before recovery takes place. Moist heat applied to the affected side of
the face may help reduce pain. Other therapies that may be useful for some individuals
include relaxation techniques, acupuncture, electrical stimulation, biofeedback training, and

The American Board of Oral and Maxillofacial Surgery 132


vitamin therapy (including vitamin B12, B6, and zinc), which may help nerve growth.
A diagnosis of Bell's palsy is made based on clinical presentation-including a distorted facial
appearance and the inability to move muscles on the affected side of the face-and by ruling
out other possible causes of facial paralysis. There is no specific laboratory test to confirm
diagnosis of the disorder. Generally, a physician will examine the individual for upper and
lower facial weakness. In most cases this weakness is limited to one side of the face or
occasionally to the forehead, eyelid, or mouth. A test called electromyography (EMG) can
confirm the presence of nerve damage and determine the severity and the extent of nerve
involvement. An x-ray of the skull can help rule out infection or tumor. A magnetic resonance
imaging (MRI) or computed tomography (CT) scan can eliminate other causes of pressure on
the facial nerve.

Reference:
th
Fauci JD et al (Eds.): Harrison's Principles of Internal Medicine 14 Ed,.McGraw-Hill, New
York, 1998 pp2378-9

The American Board of Oral and Maxillofacial Surgery 133


139. A patient who sustained a sharp blow to the chin was referred for evaluation of left TMJ pain.
The patient complained of attenuated hearing on the left side. Otoscopic examination revealed
blood in the middle ear. A positive Rinne test would suggest:

A. damage to cranial Nerve VIII.


B. damage to Cranial Nerve VII.
C. air conduction greater than bone conduction.
D. bone conduction greater than air conduction.

Answer: D

Rationale:
A Rinne test involves evaluating a subject's ability to hear a vibrating tuning fork when it is
held next to the ear and when it is placed on the mastoid process. Normally, air conduction is
greater than bone conduction. A positive Rinne test exhibits diminished hearing acuity
through air and somewhat heightened hearing acuity through bone, and is symptomatic of
conduction deafness. Testing for damage to cranial nerve VIII vestibular function (such as
cold water irrigation of the external auditory canal) often reveals nystagmus and for cochlear
function reveals hearing loss. Therefore, a patient with VIII nerve disruption would have
neither bone nor air conductive hearing. Injury to the facial nerve would be manifest by loss
of facial animation and loss of taste on the involved side.

Reference:
Handler, BH. History and Physical examination in PH Kwon, Daniel Laskin. Clinicians
Manual of Oral and Maxillofacial Surgery, 2nd edition. Quintessence Publishing, Chicago.
Chapter 1, Page 7

The American Board of Oral and Maxillofacial Surgery 134


140. Which one of the following regarding muscular dystrophy is true?

A. This group of disorders is characterized by progressive abnormalities at the acetylcholine


receptor site in the neuromuscular junction
B. Patients with Duchennes muscular dystrophy may be safely managed with a
nitrous/opioid anesthetic technique
C. Succinylcholine causes prolonged muscle paralysis in this disorder
D. Muscular dystrophy is a progressive, irreversible disease that usually leads to death in the
early 50s due to cardiac failure

Answer: B

Rationale:
Muscular dystrophy (MD) has historically been diagnosed and categorized based on the age
of onset, affected muscle groups, and genetic inheritance. Nearly one-third of new cases are
spontaneous mutations, but two-thirds are inherited. Duchenne's is by far the most common
form of MD and is due to abnormal and deficient dystrophin; other forms of MD may involve
other myocyte proteins. Administration of a depolarizing muscle relaxant to a patient with
MD would be likely to cause a massive potassium release that could precipitate cardiac arrest
due to hyperkalemia. MD patients, especially those with Duschenne's type, are susceptible to
malignant hyperthermia and therefore the use of triggering agents such as depolarizing muscle
relaxants (succinylcholine) and halogenated potent volatile anesthetics should be avoided. A
nitrous oxide/opioid (nitrous/narcotic) anesthetic technique would be appropriate for such a
patient. Prolonged muscle paralysis with succinylcholine is typical of a cholinesterase
deficiency, not muscular dystrophy. These patients also have delayed gastric emptying time,
with the associated risk of aspiration if the airway is not adequately protected. Demise from
MD usually occurs in the 20's from respiratory failure.

Reference:
Goldman:Cecil Textbook of Medicine, 22nd ed., Copyright 2004 W. B. Saunders Company,
pp. 2387-2391

The American Board of Oral and Maxillofacial Surgery 135


141. Which of the following lab tests should be ordered when considering carbamazepine therapy for
trigeminal neuralgia?

A. Complete blood count


B. Serum electrolytes
C. prothrombin time
D. Serum calcium and phosphate

Answer: A

Rationale:
Carbamazepine is the treatment of choice for trigeminal neuralgia. This anticonvulsant
provides relief to greater than 75% of trigeminal neuralgia patients. However, potential life-
threatening side effects include agranulocytosis, aplastic anemia, or thrombocytopenia.
Changes in levels of serum electrolyte levels, clotting factors/profactorslevels, or
calcium/phosphate are not generally thought to be within the side effect or complication
profile of carbamazepine.

Reference:
2002 Ferri's Clinical Advisor, Fred Ferri, Mosby 2002 Edition, p. 717. Oral and Maxillofacial
Pathology, Neville et. al., W.B. Saunders, 1995, pp. 628-9.

The American Board of Oral and Maxillofacial Surgery 136


142. A 35-year-old female patient with myasthenia gravis is scheduled for a bilateral saggital ramus
osteotomy. When the patient is induced for general anesthesia, what reaction would you expect?

A. Reacts the same with all types of muscle relaxants


B. Increased sensitivity to nondepolarizing relaxants
C. Decreased sensitivity to nondepolarizing relaxants
D. Increased sensitivity to depolarizing relaxants

Answer: B

Rationale:
Myasthenia gravis is an autoimmune disorder that occurs 1/20,000 persons and is
characterized by injury of the acetylcholine receptors at the postsynaptic neuromuscular
junction. These patients exhibit marked sensitivity to nondepolarizing muscle relaxants and
slight resistance to succinylcholine

Reference:
Clinical Anesthesia Procedures of the Massachusetts General Hospital, 4th Ed., Davidson,
et.al., Little Brown, 1993, pp. 166-8.

The American Board of Oral and Maxillofacial Surgery 137


143. The signs of causalgia (reflex sympathetic dystrophy) include:

A. hypoesthesia.
B. pain of short duration.
C. exacerbation of symptoms with a regional sympathetic block.
D. skin flushing or pallor.

Answer: D

Rationale:
This complex regional pain syndrome typically occurs after a relatively minor injury resulting
in a persistent, burning sensation with hyperesthesia and hyperpathia. This is thought to occur
from a heightened sympathetic outflow. Peripheral vasodilation (skin flushing and warmth) or
vasoconstriction (skin pallor and coolness) may occur. Diagnosis and treatment depend on
pain relief following sympathetic blockade.

Reference:
Clinical Anesthesia Procedures of the Massachusetts General Hospital, 4th Ed., Davidson,
et.al., Little Brown, 1993, pp. 595-6. Handbook of Neurosurgery, 2nd Ed., Greenberg, M.,
Greenberg Graphics, Inc., 1991, pp. 328-9. The Merck Manual, 17th Ed., Merck and Co.,
1999, pp. 1372-3.

The American Board of Oral and Maxillofacial Surgery 138


144. Which of the following statements regarding temporal arteritis is true?

A. Predilection for adolescent females


B. Rarely causes severe headaches
C. Masticatory muscle pain during chewing
D. Unresponsive to corticosteroid therapy

Answer: C

Rationale:
Temporal arteritis is a chronic inflammatory disease involving large arteries of the carotid
system, particularly in those with a prominent elastica, occurring primarily in the elderly. This
disorder commonly causes pain on mastication in the masseter, temporals, and tongue
muscles. Estimated prevalence is about 1/1000 in patients > 50 yo with a slight predilection in
females. Symptoms typically include a severe headache, scalp tenderness, and visual
disturbances. It can cause blindness secondary to ischemic optic neuropathy in less than 20%
of patients; but this occurrence is very rare after high-dose steroid treatment.

REFEENCES
The Merck Manual, 17th Ed., Merck and Co., 1999, pp. 438-9.

2002 Ferris Clinical Advisor, Fred Ferri, Mosby 2002 Edition, p. 682.

The American Board of Oral and Maxillofacial Surgery 139


145. An 89-year-old male presents with the following: akinesia, a resting pill rolling tremor, rigidity
and a shuffling gait. What is the pathophysiology of this condition?

A. Focal areas of demylination followed by reactive gliosis


B. Lewy bodies present in the temproparietal lobe, cerebellum and basal ganglia
C. Dopamine depletion in the substansia nigra of the basal ganglia
D. Antibody-mediated neuromuscular receptor depletion

Answer: C

Rationale:
Parkinson's disease is the leading cause of neurologic disease in people older than 65-years-
old and affects approximately 1 million Americans. Typically it is characterized as an
idiopathic movement disorder with akinesia, 'resting pill' rolling tremor and a shuffling gait.
The primary pathogenesis is dopamine depletion in the substansia nigra of the basal ganglia.
Lewy bodies (cytoplasmic inclusions) can be seen in Parkinson's disease however are
localized to the cerebellum, basal ganglia, sympathetic ganglion and spinal cord and not
usually the temporparietal lobe. Focal areas of demylination followed by reactive gliosis is
seen in multiple sclerosis. Myesthenia gravis is an autoimmune disease causing muscular
weakness from antibody mediated destruction of the acetylcholine receptor in the
neuromuscular junction. Weakness is temporarily ameliorated by the administration of
edrophonium, and is diagnostic for the disease.

Reference:
Wichmann T et al. Parkinson's disease and the basal ganglia. Lessons from the laboratory and
from neurosurgery. Neuroscientist. 1:236, 1995

The American Board of Oral and Maxillofacial Surgery 140


147. A 22-year-old male presents with a tumor compressing his optic chiasm. What visual field defect
would you expect to see on physical examination? (Shaded areas indicate visual field deficit.)

Answer: A

Rationale:
The optic chiasm, contains axons from the nasal half of each retina, which look at the
temporal half of the visual field and cross to the side contralateral to their eye of origin. The
axons from the temporal retina remain uncrossed. Thus, if crossing fibers are destroyed in the
optic chiasm, for example, by pressure from an expanding tumor (such as from the
hypothalamus), the result is bitemporal hemianopsia., with the visual field deficit illustrated in
A. Answer B is homonymous hemianopia and is associated with severe contralateral optic
tract defects or hemispheric lesion. Answer C is binasal hemifield defect. These defects are
associated with lesions lateral to chiasm such as aneurysms; and other causes may include
glaucoma or optic disk drusens. Answer D. is altitudinal hemianopia and is associated with
occipital cortex ischemia and defects of the posterior occipital circulation.

Reference:
Hirai T. et al. Loss of stereopsis with optic chiasmal lesions and stereoscopic tests as a
differential test. Ophthalmology. 109(9):1692-702, 2002

The American Board of Oral and Maxillofacial Surgery 141


148. Which of the following headaches is described by unilateral, orbital or temporal distribution;
intense, steady pain; onset usually within 2 or 3 hours of falling asleep; lacrimation, nasal
congestion and then rhinorrhea?

A. Common migraine
B. Classic migraine
C. Cluster headache
D. Tension headache

Answer: C

Rationale:
Cluster headaches have a much higher incidence in men than women. The pain is often retro-
orbital and is described as intense and boring in character. In addition to the above symptoms,
there may be miosis, ptosis, flush and cheek edema which may last for two hours. It recurs
nightly for weeks to months followed by months to years of relief from symptoms. Cluster
headaches can sometimes be precipitated by vasodilating agents such as alcohol,
nitroglycerin, or tyramine containing foods. Cluster headaches may be prophylaxed with ergot
preparations at bedtime, amitriptyline or lithium carbonate.

Classic migraine headache involves painless prodromes such as visual changes (scotoma)
followed by (usually unilateral) throbbing headache lasting for 6 to 8 hours and often
accompanied by fatigue and photophobia. Common migraines may not be as painful or
intense as classic, and do not have a prodrome. Controversy revolves around tension
headaches, as some think they are musculoskeletal in origin although some may have a
partial vascular component.

Reference:
Wilson, JD ed. Harrison's Principles of Internal Medicine. McGraw-Hill, Inc. 1991. p. 110-
111.

The American Board of Oral and Maxillofacial Surgery 142


149. Which of the following correctly couples the muscle with the nerve that innervates it?

A. tensor tympani - VIII


B. posterior digastric -VII
C. geniohyoid - IX
D. superior oblique muscle -VI

Answer: B

Rationale:
The anterior digastric is innervated by the mylohyoid branch of the mandibular nerve (V),
while the posterior belly of the digastric is innervated by the facial nerve (VII). They are
actually 2 embryologically distinct muscles originating from branchial arches I and II,
respectively. The tensor tympani receives its motor innervation from a branch of the
mandibular nerve (V). The intrinsic (genioglossus and geniohyoid) and extrinsic tongue
musculature receives motor input from the hypoglossal nerve (XII). The glossopharyngeal
nerve (IX) provides special sensory innervation (taste) to the posterior tongue, innervates the
carotid sinus, and carries autonomic input to the parotid gland and oropharynx. The abducent
(VI) nerve innervates the lateral rectus muscle, while the trochlear nerve (IV) innervates the
superior oblique muscle.

Reference:
Goetz:Textbook of Clinical Neurology, 2nd Ed. Elsevier, 2003. Textbook of Head and Neck
Anatomy, 2nd Ed. Hiatt, J and Gartner, L. Williams and Wilkins, 1987.

The American Board of Oral and Maxillofacial Surgery 143


150. During an elective genioplasty under deep sedation in your office, you note that the right mental
nerve is severed. What type of injury is this according to the Seddon classification system, and
how should it be repaired?

A. Neurotmesis; repair with perineurial neurorrhaphy


B. Neurapraxia; repair with epineurial neurorrhaphy
C. Axonotmesis; repair with perineurial neurorrhaphy
D. Neurotmesis; repair with epineurial neurorrhaphy

Answer: D

Rationale:
The outer layer of the trigeminal nerve is the epineurium; this encloses the perineurium,
which is the connective tissue surrounding groups of nerve fascicles. The endoneurium is the
connective tissue layer surrounding individual fascicles within the perineurium. Seddon
classified nerve injuries into neurapraxia (brief conduction loss due to manipulation with
rapid recovery and no axonal degeneration), axonotmesis (more severe injury with axonal
damage and demyelination) and neurotmesis (complete nerve transaction). Repair of this
injury should be with 2 or 3 fine non-resorbing sutures placed in the epineurial layer.
Neurorrhaphy is the term for directly suturing 2 nerve ends together.

Reference:
Peterson's Principles of Oral and Maxillofacial Surgery, 2nd Ed. Miloro, M. (Ed.) BC Decker
Inc, 2004, pp 819-837.

The American Board of Oral and Maxillofacial Surgery 144


151. A 43-year-old alcoholic male has acute onset confusion and amnesia and is brought to the ER for
further evaluation. His toxicology screen is negative. His head CT shows some focal
demyelination and gliosis of the thalamus, cerebellum and periaqueductal grey matter, without
evidence of intracranial mass or bleeding. Further careful physical examination reveals bilateral
lateral rectus palsy. What is his most likely diagnosis?

A. Korsakoff syndrome
B. Wernickes encephalopathy
C. Parkinsons disease
D. Diffuse axonal injury

Answer: B

Rationale:
Wernicke's encephalopathy is a finding often associated with chronic alcoholism and is
thought to be due to thiamine (Vitamin B1) deficiency. It is associated with an acute
confusional state, amnesia, ataxia and opthalmoplegia. The pathogenesis is due to
demyleination, neuronal loss and gliosis of the thalamus, periaqueductal gray matter,
cerebellum and cranial nerve III, VI and VII. Kosakoffs syndrome is also found in chronic
alcoholism is the inability to form new memories and usually seen later than Wernickes
encephalopathy. It too is associated with thiamine deficiency; however A is not the answer
since the episode was of acute onset. Parkinson's disease is the most common neurologic
disorder in patients over 65 years old and is a progressive movement disorder. Parkinson's
disease is caused by the progressive loss of dopamine-producing neurons in the substantia
nigra. Diffuse axonal injury is usually seen in patients in a prolonged coma, where neurologic
recovery is slow.

Reference:
Lieber CS. Medical disorders of alcohalism. N Eng J Med 333:1058, 1995. Victor M.
Alcoholic dementia. Can J Neurol Sci 21:88, 1994

The American Board of Oral and Maxillofacial Surgery 145


152. From which cranial nerve does the parasympathetic fibers to the globe arise?

A. II
B. III
C. IV
D. VI

Answer: B

Rationale:
In the parasympathetic autonomic nervous system, pre- and post-ganglionic fibers synapse
close to the target organ, not close to the spinal cord, as occurs in the sympathetic nervous
system. Parasympathetics to the eye synapse behind the globe in the ciliary ganglion and arise
from CN III. Parasympathetics to the parotid synapse in the otic ganglion and arise from CN
IX. Parasympathetics to the palatal minor salivary glands and nose synapse in the
pterygopalatine ganglion and arise from CN VII. The neurotransmitter at this site is
acetylcholine. The optic nerve (II) has no sympathetic component.

Reference:
Vander, Sherman, and Luciano: Human Physiology: The Mechanisms of Body Function.
McGraw-Hill, 1998, p.213-216. Romanes, GJ: Cunningham's Manual of Practical
Anatomy. Oxford Medical Publications, 1986.

The American Board of Oral and Maxillofacial Surgery 146


153. In treatment of diabetic ketoacidosis, maximal insulin effect is desired. This can be achieved
through:

A. plasma insulin levels of 300-350 u/ml.


B. fluid restriction.
C. IV insulin at a rate of 2-10 units/hour.
D. sodium polystyrene resin.

Answer: C

Rationale:
Maximal insulin effect is reached with plasma levels of 20-200 u/ml. This can be
accomplished (ideally in an intensive care setting) by administering IV insulin with an initial
bolus of 10-15 units, followed by a continuous infusion at a rate of 2-10 units/hour. The
ketoacidotic patient generally presents with decreased intravascular volume, requiring volume
expansion and not fluid restriction. Although serum potassium is commonly elevated by
ketoacidotic shift from intracellular fluids, total body potassium is often decreased and insulin
can rapidly drop serum potassium to dangerously low levels. Often, potassium is judiciously
given IV with insulin to keep serum potassium at a safe level.

Reference:
Rose L, Kaye D:Internal Medicine for Dentistry 2nd ed., C.V. Mosby, Saint Louis, 1990, pp
1124-1125. Donoff R: Massachusetts General Hospital Manual of Oral and Maxillofacial
Surgery C.V. Mosby, Saint Louis, 1987, pp 105-108. Stobo, et al: The Principles and Practice
of Medicine 23rd ed., Appleton & Lange, Stamford, 1996, pp 322-328

The American Board of Oral and Maxillofacial Surgery 147


154. A patient presents with the following oral/perioral conditions: metallic bad taste, halitosis,
excessive salivation, stomatitis, and parotid swelling. These symptoms are most consistent with:

A. alcoholic cirrhosis.
B. gastric ulcer.
C. hepatocellular carcinoma.
D. chronic renal failure.

Answer: D

Rationale:
Elevation of blood urea nitrogen in renal failure results in salivary ammonia excretion.
Uremic stomatitis can occur in severe renal failure. This presents with symptoms of foul taste,
halitosis, excessive salivation, as well as signs and symptoms of soft tissue stomatitis. Parotid
as well as submandibular swelling may be seen in chronic renal failure with or without
accompanying signs of stomatitis. Additionally, parotid swelling may be part of a
malnutrition syndrome such as in alcoholism.

Reference:
Rose L, Kaye D: Internal Medicine for Dentistry 2nd ed., C.V. Mosby, Saint Louis, 1990, pp
587. Donoff R: Massachusetts General Hospital Manual of Oral and Maxillofacial Surgery
C.V. Mosby, Saint Louis, 1987, pp 85-96. Stobo, et al: The Principles and Practice of
Medicine 23rd ed., Appleton & Lange, Stamford, 1996, pp 382-388

The American Board of Oral and Maxillofacial Surgery 148


155. After an orthognathic surgical procedure with prolonged induced hypotension and transfusion of
two units of autodonated and one unit of banked red blood cells, you suspect olioguric acute
renal failure. Which of the following is more predictive for your diagnosis?

A. Urine specific gravity elevated


B. Urine sodium decreased
C. Urine osomality elevated
D. Serum creatinine increased

Answer: D

Rationale:
Patients in renal failure typically exhibit specific gravity which is isothenic (1.010) or less.
Also, urine osmolality is low (<300) and sodium is high (>20mmol/L). Serum creatinine will
be markedly elevated quickly in the initial (oliguric) phase of acute renal failure. This case of
failure is probably due to ischemic hypoperfusion from prolonged hypotension. However,
deposition of myoglobin from surgical insult or hemoglobin from transfusion may also be
contributing to renal tubular dysfunction.

Reference:
Rose L, Kaye D: Internal Medicine for Dentistry 2nd ed., C.V. Mosby, Saint Louis, 1990, pp
754. Donoff R: Massachusetts General Hospital Manual of Oral and Maxillofacial Surgery
C.V. Mosby, Saint Louis, 1987, pp 144. Stobo, et al: The Principles and Practice of Medicine
23rd ed., Appleton & Lange, Stamford, 1996, pp 382-388

The American Board of Oral and Maxillofacial Surgery 149


156. Which of the following are predictive factors for the development of hepatorenal syndrome in
patients with liver disease and ascites?

A. Low serum sodium


B. Proteinuria
C. Low plasma rennin
D. Presence of hepatomegaly

Answer: B

Rationale:
Hepatorenal syndrome is a relatively frequent complication in cirrhotic patients with ascites
that is associated with an extremely short survival. It is a complication of advanced cirrhosis
characterized by renal failure due to severe renal vasoconstriction, increases in systemic blood
pressure, and increased activity of endogenous vasoactive systems. Absence of
hepatomegaly, high plasma renin activity, and low serum sodium concentration are predictors
of hepatorenal syndrome occurrence in these patients. The fundamental problem in
hepatorenal syndrome is renal ischemia secondary to hypotension and profound renal cortical
vasoconstriction, leading to renal tubular dysfunction and proteinuria. Portal hypertension and
its associated splanchnic arterial vasodilatation initiate a cascade of events leading to
activation of systemic and local vasoconstrictors and depletion of local renal vasodilators.
Therapy with vasopressin V(1) receptor and alpha-adrenergic agonists, and plasma expanders,
reverses type I and type II hepatorenal syndrome and improves survival. The first detailed
description of HRS reported 9 patients with cirrhosis or acute hepatitis who developed renal
failure without associated proteinuria and with very low urinary sodium excretion. On
autopsy, these kidneys showed normal histology. It was later shown that kidneys from
patients with HRS regain their function when transplanted into patients without cirrhosis and
that HRS can be reversible following liver transplantation if done prior to irreversible renal
damage.

Reference:
Barada K, Hepatorenal syndrome: pathogenesis and novel pharmacological targets. Curr
Opin Pharmacol 2004 Apr;4(2):189-97. Cardenas A, Arroyo V; Hepatorenal syndrome. Ann
Hepatol 2003 Jan-Mar;2(1):23-9. Gines A, Escorsell A, Gines P, Salo J, Jimenez W, Inglada
L, Navasa M, Claria J, Rimola A, Arroyo V, et al.; Incidence, predictive factors, and
prognosis of the hepatorenal syndrome in cirrhosis with ascites. Gastroenterology 1993
Jul;105(1):229-36.
Hecker R, Sherlock S; Electrolyte and circulatory changes in terminal liver failure. Lancet.
1956;2:1221-1225. Iwatsuki S, Popovtzer, M Corman J, et al.; Recovery from hepatorenal
syndrome after orthotopic liver transplantation. N Engl J Med. 1973;289:1155-1159. Koppe
M, Coburn J, Mims M, et al; Transplantation of cadaveric kidneys from patients with
hepatorenal syndrome. Evidence for the functional nature of renal failure in advanced liver
disease. N Engl J Med. 1969;280:1367-1371.

The American Board of Oral and Maxillofacial Surgery 150


157. You are called to the Emergency Department to evaluate a patient presenting with a right
mandibular angle fracture. You note a history of severe alcoholism and anorexia in this cachetic
individual. She exhibits peripheral edema, and is ataxic and confused. Which is the most likely
diagnosis?

A. Korsakoffs psychosis
B. nutritional polyneuropathy
C. Wernickes encephalopathy
D. Wet beriberi

Answer: D

Rationale:
All of the listed disorders can be caused by the thiamine deficiency common in malnutrition,
such as found in anorexic or alcoholic individuals. Chronically malnourished individuals can
manifest with numerous metabolic and electrolyte disorders, and careful nutritional diagnosis
and support is vital in this patient population. Wet beriberi is a thiamine-depletion
cardiovascular disorder with three major derangements: 1) peripheral vasodilatation with
high-output cardiac failure 2) sodium retention and peripheral edema 3) biventricular heart
failure. Wernicke's encephalopathy (cerebral beriberi) is an acutely evolving disease. Its most
common manifestations include vomiting, nystagmus, ophthalmoplegia, and ataxia.
Korsakoffs psycosis is primarily a disturbance of the ability to form new memories.
Nutritional polyneuropathy, or dry beriberi, is a polyneuropathy involving the distal
peripheral nerves. It primarily affects sensory innervation.

Reference:
Rose L, Kaye D: Internal Medicine for Dentistry 2nd ed., C.V. Mosby, Saint Louis, 1990, pp
744-745. Stobo, et al: The Principles and Practice of Medicine 23rd ed., Appleton & Lange,
Stamford, 1996, pp 931

The American Board of Oral and Maxillofacial Surgery 151


158. You are evaluating a patient who has a left submandibular infection of three days duration. His
medical history is significant for hypertension and Type II diabetes mellitus controlled with oral
hypoglycemic agents. Which of the following is true regarding this patients diabetes?

A. Treatment with oral medications should continue without change


B. Concomitant infection will cause a decrease in serum glucose, oral medications should
be discontinued
C. Concomitant infection will cause an increase in serum glucose, insulin administration
should be considered
D. Serum potassium will rise in this patient and should be treated immediately

Answer: C

Rationale:
Infections cause gluconeogenesis and glycogenolysis. Even in Type II diabetics, exogenous
insulin administration is often necessary in order to control glucose levels. Ketoacidosis
leading to academia and hyperkalemia is not commonly seen even in severe hyperglycemia
with Type II diabetics. However, in Type I individuals severely elevated glucose levels are
indicative of insulinopenia. In severe insulin deficiency, glucose is not delivered through the
cell membrane and intracellular metabolilsm uses proteins and fats, causing release of
ketoacids. As ketoacidosis causes potassium shifts from inside the cells to the extracellular
fluid, serum potassium values initially rise to critical levels, but total body potassium
depletion occurs by renal potassium excretion.

Reference:
Rose L, Kaye D: Internal Medicine for Dentistry 2nd ed., C.V. Mosby, Saint Louis, 1990, pp
1124-1125. Donoff R: Massachusetts General Hospital Manual of Oral and Maxillofacial
Surgery C.V. Mosby, Saint Louis, 1987, pp 105-108. Stobo, et al: The Principles and Practice
of Medicine 23rd ed., Appleton & Lange, Stamford, 1996, pp 321-331

The American Board of Oral and Maxillofacial Surgery 152


159. Botulinum toxin type A (Botox) works by:

A. inhibition of acetylcholine release.


B. postsynaptic binding of acetylcholine.
C. blockade of calcium channels.
D. blockage of epinephrine release.

Answer: A

Rationale:
Botulinum toxin type A blocks neuromuscular conduction by binding to receptor sites on
motor nerve terminals, inhibiting the release of acetylcholine. This effect is temporary,
usually lasting 3 to 6 months.

Reference:
Coffield JA, et al. The site and mechanism of action of botulinum neurotoxin. In: Jankovic J,
ed. Therapy With Botulinum Toxin. New York, NY Marcel Dekker; 1994:3-15.

The American Board of Oral and Maxillofacial Surgery 153


160. Resistance to Botulinum toxin type A (Botox) is increased by which of the following:

A. low doses of Botox.


B. decreased frequency of treatments.
C. decreased time interval between treatments.
D. speed of injection.

Answer: C

Rationale:
Botox resistance is caused by high doses (>150 units), increased frequency of treatments, and
decrease time interval between treatments, which become significant if given less than two
months apart. This leads to antibody formation to the toxin.

Reference:
Garcia A, Fulton JE. Cosmetic denervation of the muscles of facial expression with
botulinum toxin A - a dose responses study. Dermatologic Surg. 1996; 22:39-43.

The American Board of Oral and Maxillofacial Surgery 154


161. The best approach to avoiding upper lid ptosis when treating forehead lines with Botox is:

A. injecting at the hairline and letting the toxin diffuse inferiorly.


B. injecting only midline at the procerus muscle.
C. injecting only the frontalis muscle.
D. injecting at least 1 cm above the supraorbital rim.

Answer: D

Rationale:
Treatment of glabellar lines is easy to accomplish with minimal side effects. The most
common side effect is lid ptosis, which is caused by diffusion of Botox to the superior lid
muscles (levator palpebrae superioris). Lid ptosis is best done by staying 1cm above the
supraorbital rim when injecting the corrugator muscles.

Reference:
Carruthers A, Kiene K, Carruthers J. Botulinum A exotoxin use in clinical dermatology. J
AM Acad Dermatol. 1996;34:788.

The American Board of Oral and Maxillofacial Surgery 155


162. The recommended dosage and number of injections for treating glabellar frown lines is:

A. 20 units given with 5 injections.


B. 40 units given with 10 injections.
C. 40 units given with 5 injections.
D. 20 units given with 10 injections.

Answer: A

Rationale:
The dosage for proper treatment of glabellar frown lines is 20 units given with 5 injections (4
units per injection): two injections in each corrugator muscle, and 0ne midline injection in the
procerus muscle.

Reference:
Carruthers JDA, Carruthers A. Treatment of glabellar frown lines with C. botulinum A
exotoxin. Dermatol Surg. 1996;22:39-43.

The American Board of Oral and Maxillofacial Surgery 156


163. The muscle groups to be injected for treating glabellar frown lines with Botox include:

A. frontalis, corrugator, and procerus muscles.


B. frontalis and procerus muscles.
C. corrugator and procerus muscles.
D. frontalis and corrugator muscles.

Answer: C

Rationale:
The treatment of glabellar frown lines with Botox requires injection of the procerus and
corrugator muscle groups.

Reference:
Carruthers JDA, Carruthers A. Treatment of glabellar frown lines with C. botulinum A
exotoxin. Dermatol Surg. 1996; 22:39-43

The American Board of Oral and Maxillofacial Surgery 157


164. The minimum pore size for porous facial implants for resistance to bacterial infection is:

A. <1 micron.
B. 10 to 25 microns.
C. 26 to 50 microns.
D. >50 microns.

Answer: D

Rationale:
Porous implants have the potential for ingrowth of bacteria that are introduced at the time of
surgery or post-operatively from tissue breakdown. This occurs when the pore size is
>1micron. Macrophages require a pore size of >50 microns to enter and engulf bacteria that
have infected the implant. Therefore, the ideal porous implant would have pores smaller than
1 micron to avoid bacterial inoculation or >50 microns to allow macrophages to engulf the
bacteria.

Reference:
Cohen MS, Constantino PO, Friedman CD. Biology of implants used in head and neck
surgery. Fac Plast Surg Clin North Am. 1999;9:17.

The American Board of Oral and Maxillofacial Surgery 158


165. Of the following, the best indication for collagen replacement therapy in lip augmentation is:

A. the scarred cleft lip.


B. the aged lip with lack of definitive white roll.
C. perioral rhytides.
D. very small lips.

Answer: B

Rationale:
Collagen replacement therapy for lip augmentation is used to achieve fullness in the white roll
area of the lip. Collagen has limited or no use in the scarred cleft lip or small lips. Perioral
rhytides are best managed by other means (laser resurfacing, chemical peel, Restylane).

Reference:
Niamtu, J. New lip and wrinkle fillers. Oral Maxillofacial Surg Clin N Am. 2005;17:17-28.

The American Board of Oral and Maxillofacial Surgery 159


166. When treating deep rhytides in the perioral region in a 68 year old female with sun damaged
skin, the treatment of choice would be:

A. Botox.
B. Myobloc.
C. Bovine collagen.
D. Restylane.

Answer: D

Rationale:
Deep perioral rhytides can be caused by many factors: habits (such as smoking), sun damage,
loss of elastic collagen in the dermal layers, and overhanging, ptotic skin. Botox and
Myobloc would have minimal effect in this scenario and are not recommended for patients
over age 65. Bovine collagen can exhibit a severe inflammatory response. It is also short
lived, lasting as little as two months. Restylane is longer acting with fewer side effects and
would be the treatment of choice.

Reference:
Kane, M. Botox injections for lower facial rejuvenation. Oral Maxillofacial Surg Clin N Am.
2005;17:41-49.

The American Board of Oral and Maxillofacial Surgery 160


168. The use of dermal or alloplastic material into the lips primarily achieves:

A. increased lip volume and vermillion exposure.


B. enhanced white roll definition.
C. reduced perioral rhytides.
D. improved definition of cupids bow.

Answer: A

Rationale:
The use of autogenous or alloplastic materials into the lips primarily achieves an increase in
lip volume and vermillion exposure. Improvement of white roll definition, perioral rhytides,
or cupid's bow is inconsistent with implants.

Reference:
Sykes, JM, Emery BE. Upper and lower lip augmentation with dermal autografts. Oper Tech
Otolaryngol Head Neck Surg. 1995;6:307 Niamtu J. Advanta facial implants. Oral
Maxillofacial Surg Clin N Am. 2005;17:29

The American Board of Oral and Maxillofacial Surgery 161


169. Patients undergoing chemical peel that are taking hormone replacement therapy are at increased
risk of which of the following:

A. hypertrophic scar formation.


B. delayed healing.
C. persistent erythema.
D. pigmentary changes.

Answer: D

Rationale:
Hormone replacement therapy may contribute to melanocyte activity and pigmentary changes.
It is recommended that therapy be stopped one month prior to surgery to decrease incidence.

Reference:
James R. Hupp, esthetic surgery of the aging face. Atlas of the Oral and Maxillofacial Surgery
September 1998. Page 2

The American Board of Oral and Maxillofacial Surgery 162


170. A medium depth chemical peel agent will penetrate the skin to the level of the:

A. papillary dermis.
B. upper reticular dermis.
C. mid reticular dermis.
D. lower reticular dermis.

Answer: B

Rationale:
Medium depth peel will penetrate to the level of the upper reticular dermis. Superficial peel
will penetrate to the level of the papillary dermis. A deep depth peel will penetrate to the level
of the mid reticular dermis.

Reference:
James R. Hupp, esthetic surgery of the aging face. Atlas of the Oral and Maxillofacial Surgery
September 1998. Page 3

The American Board of Oral and Maxillofacial Surgery 163


171. Which of the following medical conditions is a relative contraindication for the use of phenol
peel:

A. pulmonary disease.
B. gastric Disease.
C. hepatic disease.
D. endocrine disease.

Answer: C

Rationale:
Phenol is metabolized in the liver and excreted in the kidney, also has been associated with
cardiac dysrhythmias. These diseases are relative contraindications to its use.

Reference:
James R. Hupp, esthetic surgery of the aging face. Atlas of the Oral and Maxillofacial Surgery
September 1998. Page 2

The American Board of Oral and Maxillofacial Surgery 164


172. Erythema that persists more than 3 month after chemical peels can be treated with:

A. antibiotic ointment.
B. hydroquinone cream.
C. topical steroid cream.
D. CO2 laser.

Answer: C

Rationale:
Erythema up to three month is treated by observation. Erythema that persists after this period
is treated with topical steroid creams. Hydroquinone is used to treat pigmentary changes. CO2
laser is of no benefit. Prolonged uses of antibiotic ointments have been implicated with
persistent erythema.

Reference:
James R. Hupp, esthetic surgery of the aging face. Atlas of the Oral and Maxillofacial Surgery
September 1998. Page 2-22

The American Board of Oral and Maxillofacial Surgery 165


173. A 62-year-old female comes to your office and requests treatment of facial wrinkles. She has
white skin. She states that she usually burns in the sun and tans minimally. Based on the above
information, she can be classified as a Fitzpatrick type:

A. type I.
B. type II.
C. type III.
D. type IV

Answer: B

Rationale:
The Class II skin type patient has a white skin color that usually burns and tans minimally.
The Class I skin type always burns and never tans. The Class III skin type tans moderately
and burns moderately. The Class IV skin type burns minimally and tans easily.

Reference:
Obagi S: Skin resurfacing OMS knowledge update 2001

The American Board of Oral and Maxillofacial Surgery 166


174. Which of the following is a contraindication to CO2 laser resurfacing:

A. history of retinoid use 5 years ago.


B. Fitzpatrick type II skin classification.
C. history of hypertrophic scar formation.
D. history of herpetic lesion.

Answer: C

Rationale:
A history of herpetic lesions is not a contraindication to CO2 laser skin resurfacing. Recent
retinoid use can predispose a patient to hypertrophic scar formation. Fitzpatrick Class V
would predispose a patient to pigmentary changes.

Reference:
Robert A. Strauss, Steven A Guttenberg. Lasers in Oral and Maxillofacial Surgery. Oral and
Maxillofacial surgery clinic may 2004 page 178-183

The American Board of Oral and Maxillofacial Surgery 167


175. A patient that undergoes laser skin resurfacing and is on Valacyclovir 1000mg once daily,
develops a herpetic outbreak in the postoperative period. Which of the following is the
recommended treatment?

A. Change to Acylovir 400mg three time/day


B. Continue the current regimen for another two weeks
C. Change to Valacyclovir 1000mg three times/day
D. Treat the outbreak with Zithromax for 10 days

Answer: C

Rationale:
The dose should be increased to a herpes zoster dose. Herpetic outbreaks are very rare when
the patient is taking antiherpetic medication but they do occur. Answer A is wrong since this
is a prophylactic dose for acyclovir.

Reference:
Robert A. Strauss, Steven A Guttenberg. Lasers in Oral and Maxillofacial Surgery. Oral and
Maxillofacial Surgery Clinics. May 2004 page 178-183

The American Board of Oral and Maxillofacial Surgery 168


176. The chromophore for the Q-switched Nd-YAG laser is:

A. hemoglobin.
B. water.
C. melanin.
D. protein.

Answer: C

Rationale:
A chromophore is a compound or chemical capable of selective light absorption. Lasers with
different wavelengths of energy will be selectively absorbed by different chromophores,
therefore allowing the laser to target specific structures. The chromophore of the CO2 laser is
water. For the argon laser it is hemoglobin. For the Q-switched Nd-YAG laser it is melanin.
Proteins are heterogenous and do not have a specific wavelength absorption range.

Reference:
Guy A. Catone, Charles C. Alling. Laser Applications in Oral and Maxillofacial Surgery, WB
Saunders, 1997.

The American Board of Oral and Maxillofacial Surgery 169


177. The dolls hair look is usually seen with the use of which of the following hair transplant
grafts:

A. punch graft.
B. minigraft.
C. micrograft.
D. single hair transfer.

Answer: A

Rationale:
The doll's hair look is seen with the cylindrical plugs. The new techniques of minigrafts and
micrografts eliminate this look.

Reference:
James R. Hupp, esthetic surgery of the aging face. Atlas of the Oral and Maxillofacial Surgery
September 1998. Page 39-51.

The American Board of Oral and Maxillofacial Surgery 170


178. Which on the following has no effect on the survival of adipocytes when harvesting fat:

A. harvesting site.
B. harvesting technique.
C. preparation technique.
D. handling techniques.

Answer: A

Rationale:
The yield of adipocytes does not vary with the harvesting site. It is well established that the
harvesting technique, preparation of fat for transfer, and handling techniques such as
prolonged exposure to air can affect the yield of viable adipocytes.

Reference:
Obagi S:Autologous Fat Augmentation for Addressing Facial Volume Loss: Oral
Maxillofacial Surg Clin N Am 17:101-102, 2005.

The American Board of Oral and Maxillofacial Surgery 171


179. The modiolus:

A. influences the configuration of the nasolabial fold.


B. is a confluence of three muscles of facial expression.
C. is a confluence of nerves influencing expression.
D. influences the configuration of the brow area.

Answer: A

Rationale:
The modiolus is an area of confluence of five muscles of facial expression, located lateral to
the corner of the mouth. The five muscles are the levator anguli oris, zygomaticus major,
risorius, platysma, and depressor anguli oris. This group of muscles is bound to the
buccinator by connective tissue, and along with the cheekbone, represent the configuration of
the nasolabial fold. The modiolus is not comprised of nerves and has no influence on the
brow area.

Reference:
Vigliante CE:Anatomy and functions of the muscles of facial expression: Oral Maxillofacial
Surg Clin N Am 17:14-15, 2005.

The American Board of Oral and Maxillofacial Surgery 172


180. Your preoperative esthetic evaluation of a patient reveals brow ptosis, and an upper eyelid crease
which is more than 12 mm above the upper eyelid margin
with lid ptosis. This patient is best treated by:

A. transcutaneous upper eyelid blepharoplasty and brow lift.


B. brow lift only, but only via a coronal approach.
C. levator aponeurosis or Mullers muscle surgery and brow lift.
D. transconjunctival upper eyelid blepharoplasty and brow lift.

Answer: C

Rationale:
Transcutaneous blepharoplasty does not correct eyelid retraction, and occasionally will
worsen the condition due to skin removal. Brow lift procedures will correct brow ptosis, but
not lid ptosis. Surgery of the muscles of the upper eyelid is often performed to correct lid
ptosis. Lid ptosis must be evaluated and corrected prior to a blepharoplasty procedure. A
transconjunctival blepharoplasty will not correct eyelid retraction.

Reference:
Karesh JW: Blepharoplasty. In: James Hupp, ed. Esthetic surgery of the aging face, Atlas of
the Oral and Maxillofacial Surgery Clinics of North America. September 1998 Loeb R:
Esthetic surgery of the eyelids. Springer-Verlag, New York 1989

The American Board of Oral and Maxillofacial Surgery 173


181. Where is Erb's point located?

A. 3 cm inferior to the lobule of the ear and along the anterior border of the
sternocleidomastoid.
B. 3 cm inferior to the lobule of the ear and along the posterior border of the
sternocleidomastoid.
C. 6 cm inferior to the lobule of the ear and along the anterior border of the
sternocleidomastoid.
D. 6 cm inferior to the lobule of the ear and along the posterior border of the
sternocleidomastoid

Answer: D

Rationale:
Erb's point is approximately 6 cm inferior to the lobule of the ear and along the posterior
border of the sternocleidomastoid muscle, where the platysma crosses it obliquely. Remaining
superficial to the fascia over the sternocleidomastoid muscle in this region ensures that injury
to the greater auricular and accessory nerves is avoided.

Reference:
Waite PD, Cuzalina LA: Rhytidectomy (Face Lift). In Fonseca R, Baker S, Wolford LM
(eds). Oral and Maxillofacial Surgery, Vol 6, WB Saunders, Philadelphia, 2000

The American Board of Oral and Maxillofacial Surgery 174


182. At the corner of the mouth, the marginal mandibular branch of the facial nerve runs:

A. deep to the platysma until about 2 cm lateral to the corner of the mouth.
B. superficial to the platysma until about 2 cm lateral to the corner of the
mouth.
C. deep to the platysma until about 4 cm lateral to the corner of the mouth.
D. superficial to the platysma until about 4 cm lateral to the corner of the
mouth.

Answer: A

Rationale:
The marginal mandibular branch of the facial nerve lies deep to the platysma in this area until
about 2 cm lateral to the commissural of the mouth, at which stage, it penetrates the
undersurface of the facial mimetic muscles, and is found more superficially.

Reference:
Waite PD, Cuzalina LA: Rhytidectomy (Face Lift). In Fonseca R, Baker S, Wolford LM
(eds). Oral and Maxillofacial Surgery, Vol 6, WB Saunders, Philadelphia, 2000

The American Board of Oral and Maxillofacial Surgery 175


183. When planning a facelift procedure, the incidence of skin flap necrosis is found to be ______
times higher in smokers when compared to non-smokers.

A. 5
B. 8
C. 12
D. 15

Answer: C

Rationale:
Some surgeons consider smoking as an absolute contraindication; however, most recommend
that smoking be stopped at least 1-2 weeks before surgery and for 2 weeks postoperatively.
The overall complication rate for patients who continue to smoke is more than twice those
who stop smoking.

Reference:
Waite PD, Cuzalina LA: Rhytidectomy (Face Lift). In Fonseca R, Baker S, Wolford LM
(eds). Oral and Maxillofacial Surgery, Vol 6, WB Saunders, Philadelphia, 2000

The American Board of Oral and Maxillofacial Surgery 176


184. Where is McGregor's patch located?

A. an area near the posterior border of the sternocleidomastoid inferior to the ear lobule,
where important nerves are found.
B. an area near the zygomatic prominence where a plexus of vessels is found.
C. an area near the antegonial notch and inferior mandible where the facial artery is found.
D. the preauricular area where the facial nerve crosses the zygomatic arch.

Answer: B

Rationale:
MacGregor's patch is also known as the bloody gulch. It is named after the strong
zygomaticodermal fibrous attachments that often present as skin dimpling or retraction. It is
also pertinent because a plexus of vessels supplied by the facial artery and transverse facial
artery becomes superficial in this area. Also, the buccal nerve lies just deep to this danger
zone, and the zygomatic branch of the facial nerve becomes more superficial in this area.

Reference:
Waite PD, Cuzalina LA: Rhytidectomy (Face Lift). In Fonseca R, Baker S, Wolford LM
(eds). Oral and Maxillofacial Surgery, Vol 6, WB Saunders Philadelphia, 2000.

The American Board of Oral and Maxillofacial Surgery 177


185. The nerve that is most commonly injured in face lift procedures is the:

A. marginal mandibular branch of facial nerve.


B. spinal accessory nerve.
C. greater auricular nerve.
D. temporal branch of the facial nerve.

Answer: C

Rationale:
Injury to the facial nerve causing paralysis is rare, and reported only to occur in 0.53 to 2.6 %
of patients. 85 % of motor nerve injuries resolve spontaneously, and results of surgical repair
are unpredictable and not very encouraging. Injury to the greater auricular nerve is most
common, and occurs in up to 7 % of patients. Temporary neuropraxia usually resolves in 2-4
months, and causes numbness/paresthesia around the inferior portion of the ear and
surrounding skin. Transection of the nerve is best treated with immediate microanastomosis.

Reference:
Waite PD, Cuzalina LA: Rhytidectomy (Face Lift). In Fonseca R, Baker S,Wolford LM
(eds). Oral and Maxillofacial Surgery, Vol 6, WB Saunders, Philadelphia, 2000

The American Board of Oral and Maxillofacial Surgery 178


186. Which of the following best describes an ideal auriculocephalic angle:

A. 10 to 20 degrees.
B. 25 to 35 degrees.
C. 40 to 50 degrees.
D. 85 to 95 degrees.

Answer: B

Rationale:
In the normal ear, the posterior wall of the conchal bowl is set at about 90 degrees to the
mastoid. A second 90-degree angle is formed at the antihelical fold, and is called the scapha-
conchal angle. These two angles, in combination with the curvature of the helix, set the
auricle adjacent to the scalp at approximately 25-35 degrees.

Reference:
Tanzer RC: Congenital deformities. In, Converse J (ed): Reconstructive Plastic Surgery, Vol
4, 2nd edition, Philadelphia, WB Saunders, 1977.

The American Board of Oral and Maxillofacial Surgery 179


187. The main sensory innervation of the auricle is via the:

A. auriculotemporal nerve.
B. cervical plexus branches.
C. lesser occipital nerves and vagus branches.
D. greater auricular nerve.

Answer: D

Rationale:
The sensory nerve supply is primarily from the anterior and posterior branches of the greater
auricular nerve. This nerve travels 8 cm posterior to the postauricular crease. The
auriculotemporal and lesser occipital nerves supply the conchal cavity and external auditory
meatus, and auricular branches of the Vagus may supply the posterior wall of the external
auditory meatus.

Reference:
Owsley TG, Tejera TJ:Otoplastic surgery for the protruding ear. In, Fonseca R, Baker S,
Wolford LM (eds). Oral and Maxillofacial Surgery, Vol 6, WB Saunders, Philadelphia, 2000

The American Board of Oral and Maxillofacial Surgery 180


188. Which of the following correctly describes the Mustarde technique for otoplasty:

A. Cartilage excision technique in a stepwise fashion to reduce a hypertrophic conchal wall.


B. Postauricular approach to remove vestigial posterior auricular muscle and its ligament
down to the mastoid fascia.
C. Creation of a new antihelical fold by cartilage weakening and mattress sutures.
D. Creation of a new antihelical fold at the expense of sharp cartilaginous ridges seen
through the thin anterior auricular skin.

Answer: C

Rationale:
The Mustarde technique involves cartilage weakening and placement of a series of horizontal
mattress sutures to create an antihelical fold. The Davis technique corrects conchal
hypertrophy by stepwise cartilage excision. The Converse-Wood Smith technique involves
several full-thickness cuts through the cartilage and suturing to form an antihelical fold; these
sharp ridges can often be felt/seen through the thin anterior skin. The Furnas technique has a
high incidence of relapse and is a postauricular approach, where the muscle is removed and
sutures placed from the mastoid fascia to the ear cartilage.

Reference:
Owsley TG, Tejera TJ:Otoplastic surgery for the protruding ear. In, Fonseca R, Baker S,
Wolford LM (eds). Oral and Maxillofacial Surgery, Vol 6, WB Saunders, Philadelphia, 2000

The American Board of Oral and Maxillofacial Surgery 181


189. Perichondritis after otoplasty is most commonly caused by which of the following organisms:

A. Streptococcus pyogenes, Escherichia coli, Hemophyllus influenzae.


B. Streptococcus pyogenes, Neisseria gonorrhea, Bacteroides species.
C. Staphylococcus aureus, Hemophyllus influenzae, Bacteroides species.
D. Staphylococcus aureus, Escherichia coli, Pseudomonas aeruginosa.

Answer: D

Rationale:
Perichondritis occurs in the early postoperative period, and is usually related to an undetected
or inadequately treated hematoma. Symptoms include pain, erythema, fever, and discharge.
Treatment included high doses of antibiotics following wound cultures, and establishing
adequate drainage. Massive cartilage destruction and severe ear deformities can result, despite
aggressive treatment.

Reference:
Owsley TG, Tejera TJ: Otoplastic surgery for the protruding ear. In, Fonseca R,
Baker S, Wolford LM (eds). Oral and Maxillofacial Surgery, Vol 6, WB Saunders,
Philadelphia, 2000.

The American Board of Oral and Maxillofacial Surgery 182


190. Most surgeons agree that reduction otoplasties can be safely performed in children after they
have attained an age of:

A. 2 years.
B. 4 years.
C. 6 years.
D. 8 years.

Answer: B

Reference:
Owsley TG, Tejera TJ: Otoplastic surgery for the protruding ear. In, Fonseca R, Baker S,
Wolford LM (eds). Oral and Maxillofacial Surgery, Vol 6, WB Saunders, Philadelphia, 2000

The American Board of Oral and Maxillofacial Surgery 183


191. When comparing the CO2 laser and Erbium: Yag laser for skin resurfacing for facial rhytids:

A. the hemostasis and collagen shrinkage achieved with a CO2 laser is the same
as that of the Erbium:Yag laser.
B. the hemostasis and collagen shrinkage achieved with the CO2 laser is greater
than with the Erbium;Yag laser.
C. the hemostasis and collagen shrinkage achieved with the CO2 laser is less
than the Erbium:Yag laser.
D. hemostasis and collagen shrinkage is not associated with either laser.

Answer: B

Rationale:
The CO2 and Erbium:Yag laser have different tissue effects. The thermal effects of the CO2
laser leaves a relatively wide zone of coagulation necrosis, which results in highly effective
hemostasis, and produces immediate visible collagen shrinkage of up to 25 percent during
irradiation. In contrast, the erbium laser has less extensive coagulation necrosis, which
permits for more bleeding and only 1-2 percent of collagen shrinkage.

Reference:
Tina, A. Cutaneous Resurfacing with CO2 and Erbium: Yag lasers: Preoperative,
Intraoperative, and Postoperative Considerations. Plastic and Reconstructive Surgery. 103(2),
619-632. 1999

The American Board of Oral and Maxillofacial Surgery 184


192. Which of the following chemical skin resurfacing agents is associated with dysrythmias?

A. Glycolic Acid
B. Trichloroacetic acid
C. Phenol
D. Salicyclic acid

Answer: C

Rationale:
Phenol is rapidly absorbed through the skin and metabolized through the
liver, and excreted by the kidney. It can lead to renal failure and hepatotoxicity, and can
directly irritate the myocardium, inducing dysrythmias.

Reference:
Demas, P, Braun, T. Chemical Skin Resurfacing. Atlas of Oral and Maxillofacial Surgery.
6(2) 1-22. 1998.

The American Board of Oral and Maxillofacial Surgery 185


193. When injecting Botox for reduction of Crows feet, the surgeon should:

A. inject the agent at least 2cm away from the lateral canthus to prevent paralysis of the
orbicularis oculi muscle.
B. inject the agent at the lateral orbital rim to prevent paralysis of the inferior rectus muscle.
C. inject the agent just medial to the lateral orbital rim to prevent paralysis of the lateral
rectus muscle.
D. inject the agent at least 1cm from the lateral orbital rim to prevent paralysis of the lateral
rectus muscle.

Answer: D

Rationale:
Crow's feet are addressed by injecting the orbicularis oculi muscle in a semi-lunar pattern.
Care is taken to maintain a 1cm distance from the lateral orbital rim to avoid diploplia from
inadvertent injection into the lateral rectus muscle.

Reference:
Obagi S. Correction of surface deformities; Botox, soft-tissue fillers, lasers and intense pulsed
light, and radiofrequency. Atlas of Oral and Maxillofacial Surgery Clinics of North America.
12. 271-297. 2004

The American Board of Oral and Maxillofacial Surgery 186


194. In treatment planning for alloplastic cheek augmentation, the ideal position of the cheek
eminence is located:

A. 15mm lateral and 10mm inferior to the lateral canthus.


B. 20mm inferior and 10mm lateral to the most lateral point of the supraorbital rim.
C. 10mm lateral and 15-20mm inferior to the lateral canthus.
D. 5mm superior and 10-15mm superior to the most lateral point at the infraorbital rim.

Answer: C

Rationale:
The most ideal position of the cheek eminence is located 10mm lateral and 15-20mm inferior
to the lateral canthus. This mark aids in the proper superior-inferior and lateral positioning of
the cheek implant.

Reference:
Epker B. Maxillofacial contour esthetic deformities. Atlas of Oral and Maxillofacial Surgery
clinics of North America. 12. 75-89.2004

The American Board of Oral and Maxillofacial Surgery 187


195. How many fat pads are excised or reduced during an upper eyelid blepharoplasty with fat
debulking?

A. 1
B. 2
C. 3
D. 4

Answer: B

Rationale:
The upper eyelid contains 2 fat pads: the medial and central fat pads which may be removed
during upper eyelid blepharoplasty. The lower eyelid contains 3 fat pads: the medial, central,
and lateral fat pads. In the upper eyelid, the lateral compartment is occupied by the lacrimal
gland and should not be disturbed during upper eyelid blepharoplasty.

Reference:
Jarecki HL, Lucarelli MJ, Lemke BN: Blepharoplasty. Peterson's Principls of Oral and
Maxillofacial Surgery 2nd Edition. 2004: pp 1317-1344

The American Board of Oral and Maxillofacial Surgery 188


196. Injury to the levator aponeurosis during upper eyelid blepharoplasty may result in which
postoperative complication:

A. dry eye.
B. diplopia.
C. ptosis.
D. lagophthalmos.

Answer: C

Rationale:
Injury to the upper eyelid levator aponeuroris may result in ptosis. Dry eye may result from
resection of the lacrimal gland. Diplopia is a complication of lower lid blepharoplasty. And
Lagophthalmos results from excessive skin/muscle excision.

Reference:
Jarecki HL, Lucarelli MJ, Lemke BN: Blepharoplasty. Peterson's Principls of Oral and
Maxillofacial Surgery 2nd Edition. 2004: pp 1317-1344

The American Board of Oral and Maxillofacial Surgery 189


197. The upper eyelid crease (supratarsal fold) is produced by which anatomic structures:

A. the insertion of the levator palpebrae aponeurosis into the skin.


B. the insertion of the superior rectus to the tarsal plate.
C. the superior oblique muscle relationship to the trochlea.
D. the orbital septum attachment to the orbital rim.

Answer: A

Rationale:
The supratarsal fold is produced by the insertion of the levator aponeurosis into the skin of the
upper eyelid.

Reference:
Delgado JA, Jacobs JL, Baylis HI, Goldberg RA: Blepharoplasty and Periorbital Surgery.
Facial Plastic Surgery Clinics of North AmericA.1998: pp 41-58

The American Board of Oral and Maxillofacial Surgery 190


198. Postoperative bleeding after blepharoplasty may result in blindness by which mechanism:

A. compression of the optic nerve.


B. compression of the central retinal artery.
C. traumatic optic neuropathy.
D. cortical blindness.

Answer: B

Rationale:
Orbital hematoma may develop after orbital fat excision. This may result in increased
intraorbital pressure with compression of the central retinal artery causing blindness.

Reference:
Hatt M: Ophthalmic Plastic and Reconstructive Surgery. New York. Thieme Inc. pp 153-
159.

The American Board of Oral and Maxillofacial Surgery 191


199. An advantage of transconjunctival lower lid blepharoplasty over transcutaneous lid
blepharoplasty would be:

A. ease of performing skin excision.


B. decreased incidence of lower lid retraction.
C. ease of performing muscle excision.
D. decreased incidence of entropion.

Answer: B

Rationale:
Transconjunctival blepharoplasty has been shown to have a significantly reduced risk of
lower lid retraction.

Reference:
Jacono AA, Moskowitz B:Facial Plastic Surgery. Vol 17(1). 2001:pp 21-27.

The American Board of Oral and Maxillofacial Surgery 192


200. During alloplastic mandibular angle augmentation, the implant is placed in which position:

A. supraperiosteal, deep to the masseter.


B. superficial to the masseter muscle.
C. superficial to the buccinator muscle.
D. subperiosteal, deep to the masseter.

Answer: D

Rationale:
Mandibular angle augmentation may be performed by an intraoral approach by placing the
implant in a subperiosteal position, deep to the masseter muscle.

Reference:
Epker BN. Alloplastic Esthetic Facial Augmentation. Peterson's Principals of Oral and
Maxillofacial Surgery 2nd Edition. 2004:1435 1447.

The American Board of Oral and Maxillofacial Surgery 193


201. When planning an advancement genioplasty, the osteotomy should be designed:

A. 1 mm inferior to the mental foramen.


B. 3 mm inferior to the mental foramen.
C. 5 mm inferior to the dentition.
D. 7 mm inferior to the dentition.

Answer: C

Rationale:
The osteotomy should be designed to avoid the inferior course of the mental foramen and
must be placed 5 mm below the mental foramen.

Reference:
Hwang KL, Song YB, Chung IH. Vulnerability of the Inferior Alveolar Nerve and Mental
Nerve during Genioplasty:an anatomic study. J Craniofac Surg 200516(1):10-4, 2005.

The American Board of Oral and Maxillofacial Surgery 194


202. Which of the following agents may be used for skin preparation prior to cosmetic
blepharoplasty?

A. Povidone-iodine solution
B. Chlorhexidine solution
C. Hydrogen peroxide
D. Castille solution

Answer: A

Rationale:
Povidone-iodine solution is not harmful to the eye. Chlorhexidine has the potential to lyse
corneal epithelium. Hydryogen peroxide is a distractor. Castille solution has no therapeutic
benefit as a presurgical antimicrobial. Solutions containing alcohol can be harmful to the eye.

Reference:
Fonseca, R.J., et al: Oral and Maxillofacial Surgery, Volume 6: Cleft/Craniofacial/Cosmetic
Surgery. pp 355

The American Board of Oral and Maxillofacial Surgery 195


203. In laser resurfacing blepharoplasty, the snap and distraction tests are used to help evaluate:

A. extraocular muscle motion.


B. eyelid laxity.
C. epithelial keratinization.
D. conjunctivitis.

Answer: B

Rationale:
The snap and distraction tests are used prior to laser blepharoplasty resurfacing to identify
patients who may be at risk for lower lid ectropion.

Reference:
Guttenberg SA, Emery RW. Laser Cosmetic Skin Resurfacing: Superpulse Perspective. Oral
and Maxillofacial Surgery Clinics of North America. May 2004. Vol 16(2): 197-213.

The American Board of Oral and Maxillofacial Surgery 196


204. With the closed approach to rhinoplasty, the intercartilagenous incision is used most often when:

A. dorsal access is needed with little or no tip alteration.


B. for access to perform lateral nasal osteotomies.
C. switching to the open approach is anticipated.
D. complete tip exposure is necessary.

Answer: A

Rationale:
The intercartilagenous incision is used to access the nasal dorsum when little or no tip
alteration is necessary in the closed rhinoplasty approach. The intracartilagenous approach
will facilitate a cephalic trimming of the lower lateral cartilage. A marginal incision
combined with a transcolumellar incision is used for the open rhinoplasty. Complete tip
exposure, particularly to view the lower lateral cartilages in their correct anatomic position is
best done with the open approach.

Reference:
Rhinoplasty:Calhoun KH, in Atlas of Head and Neck Surgery-Otolaryngology:Baily BJ,
Calhoun KH, Coffey AR, Neely JG, Lippencott-Raven, Philadelphia, pp. 450-451, 1996.

The American Board of Oral and Maxillofacial Surgery 197


205. When evaluating the nose from a lateral view, the ideal range of columellar show is:

A. between 0 to 1mm.
B. between 2 to 4mm.
C. between 5 to 7mm.
D. between 8 to 10mm.

Answer: B

Rationale:
Columellar show is the amount of columella which is seen below the alar rim. This is
normally in the range of 2 to 4mm. Excessive or insufficient columellar show should be
addressed in esthetic rhinoplasty.

Reference:
Nasal-Facial Analysis, McGraw-Wall BL, in Atlas of Head and Neck Surgery-
Otolaryngology: Baily BJ, Calhoun KH, Coffey AR, Neely JG, Lippencott-Raven,
Philadelphia, pp. 442-443, 1996.

The American Board of Oral and Maxillofacial Surgery 198


206. The spreader graft in rhinoplasty surgery is placed:

A. between the lower lateral and upper lateral cartilages


B. between the upper lateral cartilage and nasal septum
C. between the lower lateral cartilage and nasal septum
D. between the two medial crus

Answer: B

Rationale:
The internal nasal valve is formed by the junction of the upper lateral cartilage and the nasal
septum. Spreader grafts are placed to increase the size of this junction, hence improving nasal
airway breathing, and in some cases, nasal esthetics.

Reference:
Rohrich R, et al.: Use of Spreader Grafts in External Approach to Rhinoplasty. In, Clinics in
Plastic Surgery, 255-262, 1996.

The American Board of Oral and Maxillofacial Surgery 199


207. The hemi-transfixion incision differs from the complete transfixion incision because:

A. the hemi-transfixation incision gives better access to the nasal dorsum,


B. the hemi-transfixation incision provides better access to the anterior nasal spine.
C. the hemi-transfixion incision preserves more nasal tip support.
D. the hemi-transfixion incision is more often combined with the transcolumellar incision.

Answer: C

Rationale:
The transfixion and hemitransfixion incision provide access to the caudal septum in endonasal
rhinoplasty. The complete transfixion incision separates the medial crura from the septum.
The hemitransfixion incision is only made on one side, and usually stops short of the anterior
nasal spine, preserving tip support. The nasal dorsum is not approached with the transfixion
incision. The transcolumellar incision is used in an open approach and is not normally
combined with either the transfixion or hemi-transfixion incision.

Reference:
Sublabial and Septal Incisions: Renner G, in Atlas of Head and Neck Surgery-
Otolaryngology: Baily BJ, Calhoun KH, Coffey AR, Neely JG, Lippencott-Raven,
Philadelphia, pp. 853-853, 1996.

The American Board of Oral and Maxillofacial Surgery 200


208. In Caucasians, the normal range for the nasofrontal angle is:

A. 85 to 99 degrees.
B. 100 to 114 degrees.
C. 115 to 130 degrees.
D. 131 to 145 degrees.

Answer: C

Rationale:
The nasofrontal angle is the angle between a line connecting the nasal bridge and nasion and a
line connecting nasion and glabellA. Nasion is ideally located at the upper eyelid margin and
4-6mm behind the glabellA.

Reference:
Nasal-Facial Analysis: McGraw-Wall BL, in Atlas of Head and Neck Surgery-
Otolaryngology: Baily BJ, Calhoun KH, Coffey AR, Neely JG, Lippencott-Raven,
Philadelphia, pp. 442-443, 1996.

The American Board of Oral and Maxillofacial Surgery 201

Вам также может понравиться